You are on page 1of 200

2nd Edition

Chemistry Matters for GCE 'O' Level Workbook (2nd Edition)


is the companion workbook to the revised best-selling Chemistry Matters for
GCE ‘O’ Level textbook. It is written in line with the 2013 GCE Ordinary Level
Chemistry syllabus from the Ministry of Education, Singapore. This workbook 2nd Edition

2nd Edition
is designed to promote learning with understanding. Its comprehensive range
of exercises builds foundational knowledge and develops critical higher-order
thinking skills. Coupled with tips on answering techniques and diagnostic self-
tests, this workbook will help students tackle questions with success and achieve
excellence in Chemistry.

Strengthens conceptual understanding


• Graded multiple-choice, structured and free-response questions reinforce learning

Workbook (Teacher’s Edition)


while building confidence and competency.

Develops independent learning and critical thinking skills


• Every worksheet ends with a diagnostic checklist that encourages students to
evaluate their learning and take steps to secure their understanding.
• A variety of context-based and data-based questions challenge students to train
up their higher-order thinking skills.

Sharpens interpretive and answering skills


• Tips to guide students’ understanding of the questions they might be asked help
them to hit the mark with their answers.
• Accompanying worked examples of sample questions ensure students articulate
their answers effectively.

Builds exam confidence

John Sadler • Emily Sadler


Tan Yin Toon • Chen Ling Kwong
• A Trial Examination gives further practice and boosts exam confidence by helping
students familiarise themselves with the exam format.

Other components
• Practical Book
• Textbook

Tan Yin Toon • Chen Ling Kwong


John Sadler • Emily Sadler
ISBN 978-981-01-1702-3

Workbook
Teacher's Edition
(S)CMWB(TE)_Cover.indd 1 9/11/12 11:22 AM
(S)CMWB_IFC&IBC(TE).indd 1
Acknowledgements

Front cover crystals © Yurok/Dreamstime.com


Private Limited

Reprinted 2017

All rights reserved.

Printed in Malaysia
First published 2007
Second edition 2013

ISBN 978-981-01-1702-3
addressed to the Publisher.
Website: www.mceducation.com
E-mail: tmesales@mceducation.com
© 2002 Times Media Private Limited
© 2001 Oxford University Press Pte Ltd

Customer Service Hotline: (65) 6213 9444

welcome any information which would enable us to contact the copyright holders/owners involved.
© 2014 Marshall Cavendish Education Pte Ltd

Published by Marshall Cavendish Education


Times Centre, 1 New Industrial Road, Singapore 536196

Level Theory Workbook by Oxford University Press Pte Ltd

of the copyright owner. Any requests for permission should be

The publisher would like to acknowledge the following who have kindly given permission for use of copyright material:
© 2003, 2007, 2013 Marshall Cavendish International (Singapore)

photocopying, recording or otherwise, without the prior permission


or transmitted, in any form or by any means, electronic, mechanical,

While every effort has been made to contact copyright holders of material reproduced, we have been unsuccessful in some
Original edition first published in 2001 as Chemistry Matters for GCE ‘O’

Marshall Cavendish is a registered trademark of Times Publishing Limited.


No part of this publication may be reproduced, stored in a retrieval system

instances. To these copyright holders, we offer our sincere apologies and hope they will take our liberty in good faith. We would
The Periodic Table of Elements
Period Group →

I II III IV V VI VII 0
1 4

1 H He
Hydrogen Helium
1 2
7 9 11 12 14 16 19 20

2 Li Be B C N O F Ne
Lithium Beryllium Boron Carbon Nitrogen Oxygen Fluorine Neon
3 4 5 6 7 8 9 10
23 24 27 28 31 32 35.5 40

3 Na Mg Al Si P S Cl Ar
Sodium Magnesium Aluminium Silicon Phosphorus Sulfur Chlorine Argon
11 12 13 14 15 16 17 18
39 40 45 48 51 52 55 56 59 59 64 65 70 73 75 79 80 84

4 K Ca Sc Ti V Cr Mn Fe Co Ni Cu Zn Ga Ge As Se Br Kr
Potassium Calcium Scandium Titanium Vanadium Chromium Manganese Iron Cobalt Nickel Copper Zinc Gallium Germanium Arsenic Selenium Bromine Krypton
19 20 21 22 23 24 25 26 27 28 29 30 31 32 33 34 35 36
85 88 89 91 93 96 99 101 103 106 108 112 115 119 122 128 127 131

5 Rb Sr Y Zr Nb Mo Tc Ru Rh Pd Ag Cd In Sn Sb Te I Xe
Rubidium Strontium Yttrium Zirconium Niobium Molybdenum Technetium Ruthenium Rhodium Palladium Silver Cadmium Indium Tin Antimony Tellurium Iodine Xenon
37 38 39 40 41 42 43 44 45 46 47 48 49 50 51 52 53 54
133 137 139 * 178 181 184 185 190 192 195 197 201 204 207 209

6 Cs Ba La Hf Ta W Re Os Ir Pt Au Hg Tl Pb Bi Po At Rn
Caesium Barium Lanthanum Hafnium Tantalum Tungsten Rhenium Osmium Iridium Platinum Gold Mercury Thallium Lead Bismuth Polonium Astatine Radon
55 56 57 72 73 74 75 76 77 78 79 80 81 82 83 84 85 86
226 227 †
7 Fr Ra Ac
Francium Radium Actinium
87 88 89
140 141 144 150 152 157 159 162 165 167 169 173 175
*58-71 Lanthanoid series
†90-103 Actinoid series Ce Pr Nd Pm Sm Eu Gd Tb Dy Ho Er Tm Yb Lu
Cerium Preseodymium Neodymium Promethium Samarium Europium Gadolinium Terbium Dysprosium Holmium Erbium Thulium Ytterbium Lutetium
58 59 60 61 62 63 64 65 66 67 68 69 70 71
a a = relative atomic mass 232 238

Key X X = atomic symbol Th Pa U Np Pu Am Cm Bk Cf Es Fm Md No Lr


Thorium Protactinium Uranium Neptunium Plutonium Americium Curium Berkelium Californium Einsteinium Fermium Mendelevium Nobelium Lawrencium
b = proton (atomic) number
b 90 91 92 93 94 95 96 97 98 99 100 101 102 103

The volume of one mole of any gas is 24 dm3 at room temperature and pressure.
3/10/16 9:23 AM
2nd Edition

Workbook
Teacher's Edition

Tan Yin Toon • Chen Ling Kwong


John Sadler • Emily Sadler

(S)CMWB_TP(TE).indd 1 8/30/12 2:17 PM


Preface
Chemistry Matters for GCE ‘O’ Level Workbook (2nd Edition) is designed to meet the
aims and objectives of the latest GCE ‘O’ Level Chemistry syllabus requirements set by
the Ministry of Education (MOE). All aspects of this new edition have been re-designed
to strengthen understanding of concepts, provide ample practice and equip you with the
edge and confidence to excel in Chemistry.

(e) The distil


late containin
layers were obse g only bromoben
rved in the conic zene was cont
aminated with
al flask. an unknown liquid
How can the liquid , E. Two
s be separated
? Explain your
answer.

WORK ON
IT!
In the table

Self-directed learning
below, circle
relevant sect the question
ion(s) in the (s) that you
textbook to stre have answered
ngthen your unde incorrectly. Revi
rstanding of sit the
the key conc
ept(s).

Work On It! is an enhanced self-


Learning outco Question numb
mes er(s)
Multiple- Textbook
(a) Name apparatus choice Structured Free- section(s)

diagnostic tool at the end of each time, temperatu for measuring mass, response to revise
liquids and gasere and volumes of
s. 1
(b) –
Select suitable

worksheet, that will help you identity



a sample of gas.apparatus for collecting 2.1
2
(c) Understand the –
pure substance difference between –

concepts that still need working on.


s and mixtures. 2.2
(d) 3
Describe meth 2(c)(i)
purifying mixtu ods of separating and –
3.1
res of a solid

As you revise the relevant Textbook


liquid. and a
6
(e) –
Describe meth –
purifying mixtu ods of separating and 3.2
res of solids.

sections, your understanding will be (f) Describe 4


a method to sepa –
purify a solve rate and 1
nt from a solut 3.3
ion. 7, 8, 9

strengthened.
(g) Describe –
meth
purifying mixtu ods of separating and –
3.4
res of liquids.
(h) Describe –
chrom –
chromatograms.atography and interpret 2
3.5
5 1(b), 1(c),
(i) Explain
the use of a locat 1(d) –
in the chromatogr ing agen 3.6
compounds. aphy of colou t
rless

(j) Explain 1(a)
the impo –
the purity of subsrtance of deter mining 3.6
tances in daily
life. –
(k) Identify 2(c)(ii)
substance –
given their melti s and their purity, 3.7
ng and boiling
points. 3 2(a), 2(b),
2(c)(i) –
3.7

16 Worksheet 2

© 2013 Marsh
all Cavendish
International (Singa
pore) Private Limite
d

Answer It Right!
Answer It Right! guides you on how to interpret questions, so you understand exactly
what a question is asking for. As you go through the pointers and accompanying
worked examples, you will pick up the secrets to answering questions well.
Worksheet 11

Name:
( ) ( )
Worksheet 4

Class:
Name: Date:
Date:
Class: Atomic Structure
Electrolysis
for Chapter 15

for Chapter 5

ANSWER IT RIGHT!

RIGHT! ering the Go through the worked example.


ANSWER IT you in understan
ding and answ
the questions.
Use the tips to guide you in understan
ding and answering
tips to guide
ple. Use the
worked exam
Go through the s. The gases The figure below shows the electronic
questions. inert electrode structures of five atoms, P, Q, R,
carried out with S and T.
ide solution was
e sodium chlor
rolysis of dilut s.
(a) The elect colle cted . place at the electrode
produced were tions that take
ions for the reac
ionic half-equat œÊÜÀˆÌiÊޜÕÀÊ>˜Ã
ÜiÀ° 2p
(i) Construct vœÀ“>̈œ˜Ê}ˆÛi˜ÊÌ ÌœÊÜÀˆÌiÊ̅iʈœ˜
ˆVÊ 14p 9p 10p
ʜvʈ˜ Ê܏Ṏœ˜½Ê 2n 14n
œÕÊ̜ʓ>ŽiÊÕÃi œ˜Ê¼`ˆÕÌiÊ Ãœ`ˆÕ“ÊV…œÀˆ`i 10n 14p
ÌÀÕV̽ÊÀiµÕˆÀiÃÊÞ À“>̈ 10n 16n
UÊ /…iÊÌiÀ“ʼVœ˜Ã Ê܈Ê ˜ii`Ê̜ÊÕÃiÊ̅iʈ˜vœ
]ÊޜÕ
UÊ Ê˜Ê̅ˆÃÊV>Ãi
half-equations. nt. experime
apparatus used for this P Q
led diagr am to show the R S T
(ii) Draw a label +

Ê" . Which of the following statements
Na ]Ê
l ]Ê Ê>˜`
+ – is correct?
r.
ess ide solution are en gas and wate 1
Thought Proc e sodium chlor rons to form oxyg gen gas. P and S are Group 0 elements in
present –in dilut d by losing elect
the Periodic Table.
(i) z The ions  ions are discharge ng elect rons to form hydro 2 Q and T are isotopes.
z ÌÊ̅iÊ>˜œ`i]Ê" + ions are discharged by gaini 3 R and T have the same nucleon number.
Ê
z ÌÊ̅iÊV>̅œ`i] A 1 and 2

+ 4e B 1 and 3
Answer – O2­}®Ê³ÊÓ2O(l)
œ`i\Ê{" +(aq) C 2 and 3
(i) ÌÊ̅iÊ>˜ \ÊÓ (aq) + 2e
– Ê2(g) dilute NaCl]Ê
ÌÊ̅iÊV>̅œ`i electrolysis of D 1, 2 and 3
Ê Ê the products of
electrolysed —
ion that is being Þ}i˜Ê}>ÃÊ܈ÊLiÊ
the type of solut NaCl are different. >Ãʈ˜ÃÌi>`ʜvÊœÝ Not all Group 0 elements have eight
z Take note of n Ãi`]ÊV…œÀˆ˜iÊ} electrons in their outer shell. Helium
NaCl and molte >
lÊ>ÀiÊiiVÌÀœÞ has only two outer electrons.
concentrated `ʜÀʓœÌi˜Ê
i]ʈvÊVœ˜Vi˜ÌÀ>Ìi
zÊ ÊœÀÊiÝ>“« the anode. Thought Process
the produ ct at
UÊ Ê
…iVŽÊ̅iʘՓLiÀʜvʜÕÌiÀÊiiVÌÀœ˜
Ãʈ˜Êi>V…Ê>̜“°Ê˜Ê}i˜iÀ>]Êii“i
hydrogen gas same number of outer electrons. ˜ÌÃʈ˜Ê̅iÊÃ>“iÊ}ÀœÕ«Ê…>ÛiÊ̅iÊ
oxygen gas Helium in Group 0 is an exception.
(ii) UÊ Ü̜«iÃÊ>ÀiÊ>̜“ÃÊ܈̅Ê̅iÊÃ>“
iʘՓLiÀʜvÊ«ÀœÌœ˜ÃÊLÕÌÊ`ˆvviÀi˜Ì
UÊ /…iʘÕViœ˜Ê˜Õ“LiÀʈÃÊ̅iÊÃÕ“Ê Ê˜Õ“LiÀÃʜvʘiÕÌÀœ˜Ã°
œvÊ̅iʘՓLiÀʜvÊ«ÀœÌœ˜ÃÊ>˜`ʘiÕ
ÌÀœ˜Ãʈ˜Ê>˜Ê>̜“°
carbon cathod
e Answer
A
carbon anode dilute sodium
n
chloride solutio

batter y

Worksheet 11 85 © 2013 Marshall Cavendish International


(Singapore) Private Limited
d Worksheet 4 23
pore) Private Limite
International (Singa
all Cavendish
© 2013 Marsh

ii

(S)CMWB_Frontmatter.indd ii 8/16/12 9:48 AM


Comprehensive exercises
Each worksheet comprises a complete range of exercises — multiple-choice,
structured and free-response — to give you all the practice you need. The
questions are designed to support learning with understanding and give you a firm
grounding in conceptual understanding. Challenging questions are indicated with an
asterisk(*). A Trial Examination at the end will further build your exam confidence.

TEST IT!
ns
Stru ctured Questio
Section A: Multiple-C Section B: s in the spac
es provided.
below
hoice Questions wing question tes. The table
Answer the follo rent metal nitra reaction.
Choose the correct answe solutions of diffe () indicates no
A cross
r and write its letter ss to aqueous ates a reaction.
in the brackets provide added in exce A tick () indic
d. Metals were
1. Which of the following
1. rvations obtained.
processes is exother mic? records the obse Zinc nitrate
Nickel(II) Silver nitrate
A Decomposition by heat l Magn esium (colourless)
of zinc carbonate into Aqueous meta Copper(II)
nitrate
nitrate (colourless)
B Formation of glucose and zinc oxide and carbon nitrate
oxygen during photosy dioxide nitrate s) (green)
C Conversion of glucose nthesis (colo urles
to carbon dioxide and (blue) 
D Sublimation of dry ice water during respiration 

Metal  

2. Which of the following ( ) Copper 
does not involve an endothe 
rmic change?  
A An iron nail rusting in Magnesium  
air 
B A few drops of ethanol Nickel 
drying up 
C A potted plant making  
food in sunlight Silver 
D A candle melting 

Zinc mos t reac tive metal.
with the
3. A thermometer is placed ( ) tivity, starting
r of their reac
in water and the tempera metals in orde
(Figure 1). A liquid, X,
is then added to the
ture is measured at
room temperature (a) Place the
(Figure 2). water and the tempera
ture is measured again r changes?
following colou
react to give the
meta l nitrate would
l and aqueous
34
28
(b) Which meta
32 n:
30
26 (i) Blue to gree
24
28
22 ical
26
(ii) Colourles
s to blue: ate that a chem
20 that would indic
24
r observations
18
state two othe
colour changes,
Figure 1
Figure 2 (c) Apar t from n place .
reaction has take
What conclusion can be
drawn from Figures 1 and
2?
A The process of mixing
liquid X and water is exother
B The temperature increas mic.
ed by 3.5°C.
C The temperature decreas
ed by 4.0°C.
D The graph that shows
the temperature change
s after X is added to water should be: to rust?
can cause iron
the conditions that
Temperature/°C 2. (a) What are

Name: tion.
room example of oxida
rusting is an
(b) Explain why
temperature ( )

Trial Examination
Class:
Date:

Time/min
( )
Trial Examination
PAPER 1 d
104 Workshee
orksheett 13 pore) Private Limite
International (Singa
Answer all questions. all Cavendish
© 2013 Marshall Cavendish © 2013 Marsh
International (Singapore
) Private Limited
Worksheet 10 1. An isotope of element
78 Z forms an ion, Z3–. The
(S)CMWB_13.indd 104
What is element Z? Z3– ion has 18 electron
s and 17 neutrons.

-Response Que stion A Argon


Section C: Free B Nitrogen
ing question. C Oxygen
Answer the follow
facturing ammonia. D Phosphorus
process for manu
shows the Haber
1. The diagram below 2. Benzoic acid occurs naturall ( )
y in some plants. It is
preservative. The melting widely used in the food
and boiling points of benzoic industry as a food
acid are shown below.
nitrogen liquid z Melting point: 122°C
process X
condenser ammonia z Boiling point: 249°C
converter
N2 : H2 = 1 : 3
Which of the following
occurs when benzoic acid
at 2000°C is cooled to
Distance between particle 100°C?
s Energy of particles
hydrogen A increases
process Y B increases
increases
C decreases
decreases
D decreases
and decreases
unreacted nitrogened increases
hydrogen recycl 3. A student is asked ( )
to confirm the presenc
hydrogen. e of Ca2+ and I– ions in
ing nitrogen and Which of the following a salt solution.
X and Y for obtain observations is incorrec
(a) (i) Name the processes t?
Test
A add ammonia solution Observation
B add sodium hydroxid no precipitate
e solution white precipitate, insolub
into C add barium chloride solution le in excess
hy gen
hydro g gases is converted D yellow precipitate
re of nitrogen and add dilute sulfuric acid
ibe how the mixtu white precipitate
(ii) Briefly descr rter. ( )
4. A substance, X, has
ammonia in the conve the following properties:
z Melting point: 3550°C
z Does not conduct electric
ity
*2. (a) In an experiment, 25.0 cm3 of hydrochloric acid was placed in an insulated container. Aqueous Which of the following
sodium hydroxide of concentration 1.0 mol/dm3 was then added to the acid, 2.0 cm3 at a substances is likely to
be X?
time. The mixture was then stirred and the highest temperature reached was recorded after A Diamond
rea on.
in the reacti
each addition. The graph below shows the results of the experiment. en and hydrogen B Graphite
ing unreacted nitrog
advantage of recycl C Copper
(iii) Suggest one
D Poly(ethene)
Temperature/°C
( )
© 2013 Marshall Cavendish
3
of nitrogen International (Singapore
ced when 36 dm
) Private Limited
35
ted to be produ at room
ammonia is expec es are measured Paper 1 171
What volume of 3 reaction? (All volum
(b) (i) of hydrogen in the
30 reacts with 72 dm
pressure.)
temperature and
25

20

15

10

5 Worksheet 15 127
Private Limited
ional (Singapore)
Cavendish Internat
© 2013 Marshall
0 2 4 6 8 10 12 14 16 18
Volume of NaOH added/cm3 7/22/12 7:18 PM

(i) State why the reaction is exothermic.

(ii) From the graph, what was the maximum temperature reached and volume of sodium
hydroxide added?

(iii) Calculate the concentration, in mol/dm3, of hydrochloric acid used in the experiment.
Handling information
and solving problems
Data-based questions in the worksheets present
information and problems in real-life contexts.
You will evaluate, interpret data and solve
© 2013 Marshall Cavendish International (Singapore) Private Limited Worksheet 13 109 interesting problems applicable to everyday life.

iii

(S)CMWB_Frontmatter.indd iii 8/16/12 9:48 AM


Contents

Preface ii

Worksheet 1 Kinetic Particle Theory 1

Worksheet 2 Experimental Techniques 9

Worksheet 3 Elements, Compounds and Mixtures 17

Worksheet 4 Atomic Structure 23

Worksheet 5 Chemical Bonding 31

Worksheet 6 Chemical Calculations 41

Worksheet 7 Acids and Bases 51

Worksheet 8 Salts 59

Worksheet 9 Oxidation and Reduction 67

Worksheet 10 Metals 75

Worksheet 11 Electrolysis 85

Worksheet 12 The Periodic Table 93

Worksheet 13 Energy Changes 103

Worksheet 14 Speed of Reaction 111

Worksheet 15 Ammonia 123

Worksheet 16 The Atmosphere and Environment 129

Worksheet 17 An Introduction to Organic Chemistry 135

Worksheet 18 Alkanes and Alkenes 141

Worksheet 19 Alcohols and Carboxylic Acids 153

Worksheet 20 Macromolecules 163

Trial Examination 171

The Periodic Table of Elements

iv

(S)CMWB_Frontmatter.indd iv 8/16/12 9:48 AM


Name: ( )

Worksheet 1
Class: Date:

Kinetic Particle Theory

for Chapter 1
ANSWER IT RIGHT!

Go through the worked example. Use the tips to guide you in understanding and answering
the questions.

The diagram below shows four balloons containing the gases, methane (CH4), oxygen (O2), nitrogen (N2)
and carbon dioxide (CO2) at a constant temperature of 25°C. Each balloon contains the same volume
of gas. When left in the air for some time, the balloons deflate and become smaller.

CH4 O2 N2 CO2

(a) Explain why the balloons deflate over a period of time.

• You need to familiarise yourself with the glossary of terms used in Chemistry papers.
• The term ‘explain’ implies that you will need to give reasons or refer to a particular theory to support
your answer.
• In this case, refer to the kinetic particle theory to explain why the balloons deflate.

Thought Process
Using the kinetic particle theory, consider the movement of gas particles.

Answer
According to the kinetic particle theory, gas particles are in constant motion. The rubber that is
used to make balloons contains millions of small pores. The gas particles can move through these
small pores. Hence, the balloons deflate over a period of time.

(b) State which balloon will deflate at the (i) fastest rate; (ii) slowest rate. Give a reason for your
answer.

‘State’ requires a concise answer. There is no need to give supporting reasons. However, if an explanation
is required, the question will ask for it.

Thought Process
At a constant temperature, the rate of movement of gas particles depends on the molecular mass
of the gas particles. The lighter the gas particles, the higher the rate of movement.

© 2013 Marshall Cavendish International (Singapore) Private Limited Worksheet 1 1

(S)CMWB_01.indd 1 8/14/12 11:29 AM


Answer
(i) Balloon containing methane. Methane is the lightest (smallest molecular mass). Hence, the
particles of methane will pass through the small pores of the balloon at the fastest rate.
(ii) Balloon containing carbon dioxide. Carbon dioxide is the heaviest (largest molecular mass).
Hence, the particles of carbon dioxide will pass through the small pores of the balloon at the
slowest rate.

It is insufficient to say the gas is ‘light’ or ‘heavy’ because you need to compare it to the other gases. Use
the words ‘lightest’ and ‘heaviest’.

TEST IT!

Section A: Multiple-Choice Questions


Choose the correct answer and write its letter in the brackets provided.

1. Which of the following statements is not true?


A Matter can exist in three states.
B A solid has a fixed volume.
C A gas cannot be compressed.
D A liquid takes the shape of its container. ( C )

2. Which statement about the kinetic particle theory is not true?


A It describes the different states of matter.
B It explains the changes of state.
C It explains the difference in the properties of solids, liquids and gases.
D It explains why substances have different solubilities. ( D )

3. The diagrams below show the arrangement of particles in different states of matter.

1 2 3
Which diagrams represent the correct arrangement of particles in a solid, liquid and gas
respectively?
Solid Liquid Gas
A 1 2 3
B 2 1 3
C 2 3 1
D 3 2 1 ( D )

2 Worksheet 1 © 2013 Marshall Cavendish International (Singapore) Private Limited

(S)CMWB_01.indd 2 8/14/12 11:29 AM


4. The rate of diffusion of a substance depends on its .
1 relative molecular mass
2 smell and colour
3 temperature
A 1 and 2
B 1 and 3
C 2 and 3
D 1, 2 and 3 ( B )

Use the table below to answer Questions 5 and 6.

Metal Melting point/°C Boiling point/°C


A caesium 29 671
B potassium 63 774
C rubidium 39 688
D sodium 98 883

5. Which metal remains a liquid over the widest range of temperatures? ( D )

6. Which metal is most likely to be in the liquid state at 30°C? ( A )

7. A substance changes from a state where its particles vibrate in fixed positions to a state where
the particles move about freely but are still close to one another.
What change has taken place?
A Condensation
B Evaporation
C Freezing
D Melting ( D )

8. The diagram below shows an apparatus for studying the effect of heat on substance X. What is X?

cold
water in

cold
water out

solid X

heat

A Silicon B Carbon
C Iodine D Oxygen ( C )

© 2013 Marshall Cavendish International (Singapore) Private Limited Worksheet 1 3

(S)CMWB_01.indd 3 3/10/16 9:25 AM


9. The diagrams below show the particles of hexane at atmospheric pressure but at two different
temperatures. Hexane melts at –95°C and boils at 70°C.

x°C y°C

What could be the values of x and y?


x y
A –100 0
B –50 10
C 0 100
D 10 –50 ( C )

Section B: Structured Questions


Answer the following questions in the spaces provided.

1. The interconversions between the three states of matter are represented by the diagram below.

melting X

Y Z

(a) Name the process in

(i) X; Boiling/ Evaporation (ii) Y; Freezing (iii) Z. Condensation

(b) In which of the change(s) of state (X, Y and/or Z) is energy being given out?

Y and Z

2. In terms of the kinetic particle theory, explain the following.

(a) A liquid can take the shape of its container.

The particles in a liquid are not held in fixed positions and can move freely throughout the liquid.
________________________________________________________________________________

________________________________________________________________________________

(b) A tub of ice is heavier than a tub filled with water vapour.

The particles of ice, a solid, are very closely packed together. The particles of water vapour, a gas,
________________________________________________________________________________

are spread far apart from one another. Thus, ice has a higher density than water vapour of the same
________________________________________________________________________________
volume.
________________________________________________________________________________

4 Worksheet 1 © 2013 Marshall Cavendish International (Singapore) Private Limited

(S)CMWB_01.indd 4 8/14/12 11:29 AM


(c) A spray can containing gas should be kept away from fire.

If a spray can is heated, the gas particles inside it will gain energy and spread out further apart very
________________________________________________________________________________

quickly. This may cause the can to explode.


________________________________________________________________________________

3. The boiling point of a liquid depends on the surrounding pressure. The atmospheric pressure at
sea level is 1 atm.
The atmospheric pressure at a base camp on a mountain is 0.40 atm. A mountain climber at the
base camp gently heated several chunks of ice in a saucepan until the water boiled for several
minutes.
(a) At 0.40 atm, ice melts at 0°C and water boils at 68°C. Sketch a graph of temperature against
time for the heating of ice and the boiling of water.
Temperature/°C
(water and water vapour)
68

(water)

(ice and water)


0 Time

(b) On your graph, label where there is


(i) water only;
(ii) a mixture of ice and water;
(iii) a mixture of water and water vapour.

4. Methanal (HCHO) kills most bacteria. The heating curve of methanal is shown in Figure 1.
Selenium (Se) has both metallic and non-metallic properties. It can convert light energy directly
into electrical energy and is used in solar cells. The cooling curve of selenium is shown in
Figure 2.

Temperature/°C
Temperature/°C
1000
Time/min
0 900
liquid + gas 800
C D liquid + gas
700
E F
–50 600

500

400
liquid + solid
–100 A B 300
liquid + solid
200 G H
100
0 Time/min

Figure 1 Figure 2

© 2013 Marshall Cavendish International (Singapore) Private Limited Worksheet 1 5

(S)CMWB_01.indd 5 8/14/12 11:29 AM


(a) Name the process that occurs from
(i) A–B; Melting

(ii) C–D; Boiling

(iii) E–F; Condensation

(iv) G–H. Freezing

(b) From Figures 1 and 2, infer the physical states of the substances at room temperature
(30°C).
(i) Physical state of methanal: Gas

(ii) Physical state of selenium: Solid

(c) Solar cells can generate such a large amount of heat energy that the cells heat up to more
than 1000°C. Such cells spoil when they vaporise. At what temperature will a selenium cell
start to spoil?
About 700°C. Selenium will turn into a gas (i.e. vaporise) at around this temperature.

Section C: Free-Response Question


Answer the following question.

*1. The table below compares the rate of diffusion of some gases to that of helium.

Gas Relative molecular mass Rate of diffusion with respect to helium


helium 4 1.0
methane 16 0.5
oxygen 32 0.35
sulfur dioxide 64 0.25
heptane 100 0.2

(a) Define the term ‘diffusion’.

Diffusion is the process by which particles move freely to fill up any available space.

6 Worksheet 1 © 2013 Marshall Cavendish International (Singapore) Private Limited

(S)CMWB_01.indd 6 8/14/12 11:29 AM


(b) (i) Plot a graph of rate of diffusion with respect to helium against the relative molecular
mass.

1.2

1.0
Rate of diffusion with respect to helium

0.8

0.6

0.4

0.2

0 20 40 60 80 100 120 140


Relative molecular mass

(ii) What conclusion can you make from your graph?


The rate of diffusion of a gas is inversely proportional to its relative molecular mass.

(iii) How much faster does helium diffuse compared to a gas with a relative molecular mass
of 40?
From the graph,
Rate of diffusion of gas with relative molecular mass of 40 = 0.28

Number of times rate of diffusion of helium is as fast as that of the gas = 1


0.28
= 3.571

© 2013 Marshall Cavendish International (Singapore) Private Limited Worksheet 1 7

(S)CMWB_01.indd 7 8/14/12 11:29 AM


(iv) What is the relative molecular mass of a gas that diffuses 2.5 times as slow as helium?

Rate of diffusion of the gas : rate of diffusion of helium = 1 : 2.5

Rate of diffusion of the gas = 1


Rate of diffusion of helium 2.5
Rate of diffusion of the gas with respect to helium = 1 × 1 = 0.4
2.5
From the graph, relative molecular mass of the gas = 22

(v) Name a gas that diffuses faster than helium.


Hydrogen (relative molecular mass = 2.0)

(vi) Explain, with a reason, how the results might differ if the experiment is performed at a
higher temperature.

The rate of diffusion of all the gases will increase. An increase in temperature will cause the gas

particles to gain kinetic energy and move faster.

(c) Give an everyday example of diffusion.

The aroma of food being detected from a distance when someone is cooking
(Accept any possible answers.)

WORK ON IT!
In the table below, circle the question(s) that you have answered incorrectly. Revisit the
relevant section(s) in the textbook to strengthen your understanding of the key concept(s).

Question number(s) Textbook


Learning outcomes Multiple- Free- section(s)
Structured to revise
choice response

(a) Describe the solid, liquid and gaseous


1 – – 1.1
states of matter.

(b) State the kinetic particle theory. 2 – – 1.2

(c) Describe the states of matter using the


3 2 – 1.2
kinetic particle theory.

(d) Explain the interconversion of states of


matter in terms of the kinetic particle 5, 6, 7, 8, 9 1, 3, 4 – 1.3
theory and energy changes.

(e) Use diffusion to account for the


1(a)
movement of particles in liquids and – – 1.4
gases.

(f) State examples of daily effects of


– – 1(c) 1.4
diffusion.

(g) Explain qualitatively the effect of


molecular mass and temperature on 4 – 1(b) 1.4
the rate of diffusion.

8 Worksheet 1 © 2013 Marshall Cavendish International (Singapore) Private Limited

(S)CMWB_01.indd 8 8/14/12 11:29 AM


Name: ( )

Worksheet 2
Class: Date:

Experimental Techniques

for Chapters 2 and 3


ANSWER IT RIGHT!
Go through the worked example. Use the tips to guide you in understanding and answering
the questions.

A gas sample contains a mixture of three gases: ammonia, carbon dioxide and methane. The properties
of these gases are shown below.

Gas Solubility in water Solubility in sodium hydroxide solution


ammonia extremely soluble insoluble
carbon dioxide slightly soluble extremely soluble
methane insoluble insoluble

(a) What are the steps needed to obtain methane from this gas sample? Explain the purpose of
each step.

You will need to state the steps for separating the gases in the correct sequence (i.e. which gas should
be removed first, second and last).

Thought Process
There are three methods of collecting gases — displacement of water, downward delivery and
upward delivery. The method of collecting a gas will depend on its solubility and density. In order
to obtain methane, ammonia and carbon dioxide must first be removed from the gas mixture.

Answer
Step 1: Pass the gas sample through water.
Purpose: To remove ammonia, which is extremely soluble in water
Step 2: Pass the remaining gases (carbon dioxide and methane) through sodium hydroxide
solution.
Purpose: To remove carbon dioxide, which is extremely soluble in sodium hydroxide solution
Step 3: Collect methane by displacement of water.
Purpose: To remove any residual gases that are slightly soluble in water

(b) Draw a labelled diagram to illustrate your answer in (a).

Your diagram should include all the pieces of apparatus needed for the experiment.

© 2013 Marshall Cavendish International (Singapore) Private Limited Worksheet 2 9

(S)CMWB_02.indd 9 8/14/12 4:10 PM


Answer
gas jar
delivery tube methane collected
mixture of ammonia,
carbon dioxide and
methane

water trough
conical flask
water
water

sodium hydroxide
solution

• You must label your diagram and make sure that it is large enough to fill up the space given.
• Make sure the apparatus is set up correctly.
– The tube introducing the gases must be immersed in the water or sodium hydroxide solution.
– The exiting tube must not be immersed.
– The apparatus must be air-tight, without any places for the gas to escape.

TEST IT!

Section A: Multiple-Choice Questions


Choose the correct answer and write its letter in the brackets provided.

1. The diagram shows three pieces of apparatus cm3

used for measuring volumes of liquids. Which


25 cm3 50 cm3
0

apparatus can be used to measure 25.0 cm3 10


50 cm3
of distilled water? (Note: Diagrams are not 20

drawn to scale.) 30

A 1 only 40

B 2 only 50

C 1 and 2
D 2 and 3
1 2 3 ( C )

gas P
2. Gas P can be collected using the apparatus shown.
delivery tube
Based on the diagram only, what can you infer about
gas P?
A It is denser than air. gas jar
B It is insoluble in water.
C It is less dense than air.
D It is soluble in water. ( A )

10 Worksheet 2 © 2013 Marshall Cavendish International (Singapore) Private Limited

(S)CMWB_02.indd 10 8/14/12 4:10 PM


3. Which property indicates that a solid is pure?
A It burns completely in oxygen leaving no residue.
B It has a fixed melting point.
C It has a fixed volume.
D It is soluble in water. ( B )

4. Which of the following methods is used to obtain iodine from a mixture of iodine and sodium
chloride?
A Crystallisation B Evaporation
C Magnetic separation D Sublimation ( D )

5. Which statement has to be true in order for two substances to be separated by chromatography?
A They are soluble in the same solvent.
B They have different boiling points.
C They have different colours.
D They have different densities. ( A )

6. Sodium nitrate decomposes when heated. Which is the best method to obtain crystals of sodium
nitrate from sodium nitrate solution?
A Chromatography
B Distillation
C Evaporation to dryness
D Crystallisation ( D )

Use the following information to answer Questions 7 and 8.

The diagram shows the apparatus used to obtain water from a solution of blue copper(II) sulfate.

water out

Y
Z

X
water in

7. At which part of the apparatus will the temperature be greater than 100°C?
A W B X C Y D Z ( B )

© 2013 Marshall Cavendish International (Singapore) Private Limited Worksheet 2 11

(S)CMWB_02.indd 11 8/14/12 4:10 PM


8. Which statement about the separation process shown is true?
A Liquid W changes from colourless to blue.
B Liquid X gets darker blue in colour.
C The temperature at Y is 100°C at first and then steadily rises as liquid W is collected.
D The vapour only starts to condense at Z. ( B )

9. A substance, Q, has the following properties:

Melting point Boiling point Solubility in water Reaction with water


85°C 130°C High None

Which set of apparatus can be used to separate water from a mixture of Q and water?
A B C D

( B )

Section B: Structured Questions


Answer the following questions in the spaces provided.

1. Six athletes, A, B, C, D, E and F, took part in a race. cm


Athlete E won the race easily and was suspected 10 solvent front
to have taken banned drugs. A sample of his urine
was analysed by chromatography to determine 9

whether it contained any of the two known banned 8


drugs, X and Y, which are used to increase physical
performance. The diagram on the right shows the 7
chromatogram obtained.
6
Key:
(a) The spots on the chromatogram obtained were X – banned drug X
5
colourless. What step was carried out to make Y – banned drug Y
the spots visible? 4 E – urine sample of
athlete E
A locating agent was applied onto the 3

2
chromatogram.
1

0
starting line
X Y E

(b) State one factor that determines how far a substance travels on a chromatogram.
Solubility of the substance in the solvent

12 Worksheet 2 © 2013 Marshall Cavendish International (Singapore) Private Limited

(S)CMWB_02.indd 12 8/14/12 4:10 PM


(c) What are the Rf values of X and Y?

Rf of X = 7.5 = 0.75
10
Rf of Y = 3.0 = 0.3
10

(d) Has athlete E taken either drug X or Y? Explain your answer.


No. None of the Rf values of the substances in his urine matches those of X and Y.

2. A scientist carried out an experiment to synthesise a new drug. On his first attempt, he obtained a
white solid, which he named substance S. He decided to examine substance S further by heating
it. The table below shows how the temperature of substance S changed with time during heating.

Time/min 0 1 2 3 4 5 6 7 8 9 10 11
Temperature/°C 30 40 50 60 70 80 90 92 94 96 98 110

(a) Using the data above, plot a graph of temperature against time on the grid provided below.

Temperature/°C

120

100

80

60

40

20

0
1 2 3 4 5 6 7 8 9 10 11 12
Time/min

(b) From your graph, deduce the melting point of substance S.

90–98°C

(c) (i) The scientist decided to purify substance S further. Explain why the scientist concluded
that substance S was not pure.
Substance S melted over a range of temperatures. It has no fixed melting point.

(ii) Explain why it is important for drugs to be pure.


Impurities in drugs can cause undesirable side effects.

© 2013 Marshall Cavendish International (Singapore) Private Limited Worksheet 2 13

(S)CMWB_02.indd 13 8/14/12 4:10 PM


Section C: Free-Response Questions
Answer the following questions.

1. Caliche is made up mainly of sodium nitrate, which is soluble in water. You may assume that all the
other substances present in caliche are insoluble in water. Describe the processes for obtaining
pure, dry sodium nitrate crystals from caliche. Explain the purpose of each process.
Step 1: Add water to caliche and stir. This is to dissolve the soluble sodium nitrate to form sodium nitrate

solution.

Step 2: Filter the mixture and collect the filtrate. This is to separate the insoluble substances from the

sodium nitrate solution.

Step 3: Evaporate the filtrate collected. This is to obtain a saturated solution of sodium nitrate.

Step 4: Allow the saturated solution to cool. This is to allow sodium nitrate crystals to form.

Step 5: Filter the mixture and collect the residue. This is to separate the sodium nitrate crystals from the

solution.

Step 6: Press the crystals between a few sheets of filter paper. This is to dry the pure sodium nitrate

crystals obtained.

*2. Chlorobenzene and bromobenzene are


miscible liquids. Their boiling points are
132°C and 156°C respectively. A student
water out
set up the apparatus shown on the right to
separate chlorobenzene from a mixture of P
C
chlorobenzene and bromobenzene.
(a) Name the method used to separate B

chlorobenzene and bromobenzene in


this experiment. water in
A
Fractional distillation
mixture of
chlorobenzene and
bromobenzene distillate
(b) (i) The student made an error in
setting up the apparatus. On the boiling chips
diagram, circle the part in the
apparatus where the error has
heat from
occurred. electric heater

14 Worksheet 2 © 2013 Marshall Cavendish International (Singapore) Private Limited

(S)CMWB_02.indd 14 8/14/12 4:10 PM


(ii) What would happen if the student does not correct his mistake? Why?
The apparatus would blow up. Air pressure would build up in the apparatus.

(c) Identify the apparatus labelled A, B and C. In each piece of apparatus, at least one change of
state has occurred. Name these processes.

Apparatus Process(es)
A: Round-bottomed flask Evaporation/boiling
B: Fractionating column Evaporation/boiling and condensation
C: Condenser Condensation

(d) Assume the apparatus is now set up correctly.


(i) Sketch a graph to show the temperature at point P against time during the experiment.

Temperature at point P/°C

156

132

0 Time/min

(ii) Explain the temperature changes in the graph you obtained in (d)(i).
When the mixture of chlorobenzene and bromobenzene is heated, its temperature increases

until it reaches 132°C, the boiling point of chlorobenzene. The temperature remains at 132°C

until all the chlorobenzene has distilled over. The temperature then rises until it reaches

156°C, the boiling point of bromobenzene. The temperature remains at 156°C until all the

bromobenzene has distilled over.

© 2013 Marshall Cavendish International (Singapore) Private Limited Worksheet 2 15

(S)CMWB_02.indd 15 8/14/12 4:10 PM


(e) The distillate containing only bromobenzene was contaminated with an unknown liquid, E. Two
layers were observed in the conical flask.

How can the liquids be separated? Explain your answer.

By using a separating funnel. Bromobenzene and E are immiscible and do not dissolve in each other.

WORK ON IT!
In the table below, circle the question(s) that you have answered incorrectly. Revisit the
relevant section(s) in the textbook to strengthen your understanding of the key concept(s).

Question number(s) Textbook


Learning outcomes Multiple- Free- section(s)
Structured to revise
choice response

(a) Name apparatus for measuring mass,


time, temperature and volumes of 1 – – 2.1
liquids and gases.

(b) Select suitable apparatus for collecting 2


– – 2.2
a sample of gas.

(c) Understand the difference between


3 2(c)(i) – 3.1
pure substances and mixtures.

(d) Describe methods of separating and


purifying mixtures of a solid and a 6 – – 3.2
liquid.

(e) Describe methods of separating and


4 – 1 3.3
purifying mixtures of solids.

(f) Describe a method to separate and


7, 8, 9 – – 3.4
purify a solvent from a solution.

(g) Describe methods of separating and


– – 2 3.5
purifying mixtures of liquids.

(h) Describe chromatography and interpret 1(b), 1(c),


5 – 3.6
chromatograms. 1(d)

(i) Explain the use of a locating agent


in the chromatography of colourless – 1(a) – 3.6
compounds.

(j) Explain the importance of determining


– 2(c)(ii) – 3.7
the purity of substances in daily life.

(k) Identify substances and their purity, 2(a), 2(b),


3 – 3.7
given their melting and boiling points. 2(c)(i)

16 Worksheet 2 © 2013 Marshall Cavendish International (Singapore) Private Limited

(S)CMWB_02.indd 16 8/14/12 4:10 PM


Name: ( )

Worksheet 3
Class: Date:

Elements, Compounds and Mixtures

for Chapter 3
ANSWER IT RIGHT!
Go through the worked example. Use the tips to guide you in understanding and answering
the questions.

(a) Complete the table below for comparing the characteristics of compounds and mixtures.

Compounds Mixtures
Separation
Energy change
Composition

The term ‘compare’ implies that you need to provide both similarities and differences between things or
concepts.

Answer
Compounds Mixtures
Separation The components in a compound can The components in a mixture can be
only be separated by chemical separated by physical processes.
processes (e.g. electrolysis).
Energy change A chemical reaction occurs when There is no reaction when a mixture
a compound is formed. There is is formed. Hence, there is little or no
usually an energy change. energy change.
Composition The elements in a compound are The components can be mixed in
always combined in a fixed any proportion.
proportion.

(b) Is tap water a pure compound? Explain your answer.

Answer
A pure compound is made up of only one substance. Tap water is not a pure compound. It contains
other elements or compounds, such as minerals, dissolved in it.

It is a common mistake to think that tap water is pure. It is a mixture of water and other compounds
such as rainwater and dissolved minerals.

© 2013 Marshall Cavendish International (Singapore) Private Limited Worksheet 3 17

(S)CMWB_03.indd 17 8/14/12 4:14 PM


TEST IT!

Section A: Multiple-Choice Questions


Choose the correct answer and write its letter in the brackets provided.

1. Ammonium carbonate is a salt with the chemical formula, (NH4)2CO3. What is the number of
elements in ammonium carbonate?
A 2
B 3
C 4
D 5 ( C )

2. Which statement is not true about an alloy?


A An alloy is a mixture.
B An alloy may contain a non-metal.
C The chemical properties of an alloy are similar to the elements that form it.
D The physical properties of an alloy are similar to the elements that form it. ( D )

3. Which statements are true about compounds?


1 They can be made from two metals.
2 They can be made from two non-metals.
3 They can be made from a metal and a non-metal.
4 They can be made from another compound.
A 1, 2 and 3
B 1, 2 and 4
C 1, 3 and 4
D 2, 3 and 4 ( D )

4. The diagram below shows two electrical circuits. What could X and Y be if the bulbs light up in both
circuits?
battery battery

bulb bulb

solid X liquid Y

X Y
A copper molten sulfur
B sand molten silver
C silver molten copper
D sulfur molten sand ( C )

18 Worksheet 3 © 2013 Marshall Cavendish International (Singapore) Private Limited

(S)CMWB_03.indd 18 3/10/16 9:31 AM


5. PCB is a compound that was used as a coolant. It was banned in 2001 because it is extremely
poisonous. PCB can be made harmless by reacting it with calcium oxide to form calcium chloride,
carbon dioxide and water. What elements must PCB contain?
A Calcium, carbon and oxygen
B Calcium, chlorine and oxygen
C Carbon, chlorine and hydrogen
D Chlorine, hydrogen and oxygen ( C )

6. Which diagram below represents nitrogen (N2) at room temperature and pressure?

A B C D ( A )

7. When mercury(II) oxide is heated, it decomposes to give the elements mercury and oxygen. What
can you deduce from this experiment?
A Mercury is a liquid element.
B Mercury(II) oxide is a compound.
C Oxygen is a gaseous element.
D When mercury is heated in oxygen, it will form mercury(II) oxide. ( B )

8. Which diagram below represents an incomplete reaction between two elements?

A B C D ( C )

9. Shoe polish consists of wax, turpentine (a volatile liquid that does not mix with water) and dyes.
Once applied, shoe polish hardens as the turpentine in it evaporates.
Which of the following statements is true?
A Shoe polish melts over a range of temperatures.
B A clear solution is formed when water is added to shoe polish.
C The components of shoe polish can be separated by evaporation to dryness.
D The components of shoe polish are always combined in a fixed proportion. ( A )

© 2013 Marshall Cavendish International (Singapore) Private Limited Worksheet 3 19

(S)CMWB_03.indd 19 8/14/12 4:14 PM


Section B: Structured Questions
Answer the following questions in the spaces provided.

1. Eight elements are listed in the table below. All of them are non-metals.

Number of atoms
Element Chemical symbol
in one molecule

argon Ar (noble gas)

hydrogen H 2

fluorine F 2

neon Ne (noble gas)

nitrogen N 2

oxygen O 2

phosphorus P 4

sulfur S 8

(a) Explain what ‘an element’ is.


An element is a pure substance that cannot be broken down into two or more simpler substances by

chemical processes.

(b) Complete the table by filling in the chemical symbols of the eight elements.

(c) Tabulate the non-metals from the table as monatomic, diatomic or polyatomic.

Monatomic Diatomic Polyatomic


argon hydrogen phosphorus
neon fluorine sulfur
nitrogen
oxygen

2. The following describes an experiment in which magnesium was heated in air.

1. Magnesium is a grey metal. A piece of magnesium ribbon was placed in a crucible and
weighed.
2. The crucible was heated and the magnesium ribbon burnt with a very bright flame.
3. A white solid consisting of magnesium oxide and a small amount of magnesium nitride
was formed.
4. The white solid and crucible were weighed.

20 Worksheet 3 © 2013 Marshall Cavendish International (Singapore) Private Limited

(S)CMWB_03.indd 20 8/14/12 4:14 PM


(a) In this experiment, identify
Magnesium
(i) an element;

Magnesium oxide and magnesium nitride


(ii) two compounds;

The white solid consisting of magnesium oxide and magnesium nitride


(iii) a mixture.

(b) State the evidence to indicate that a chemical change has taken place.
Energy was given off — a bright flame was produced when magnesium was burnt. When magnesium

was heated, new substances such as magnesium oxide and magnesium nitride were formed.

(c) Name two elements that magnesium reacted with.


Oxygen and nitrogen

(d) Magnesium nitride is made up of three atoms of magnesium and two atoms of nitrogen. What
is the formula of magnesium nitride?
Mg3N2

Section C: Free-Response Question


Answer the following question.

1. Read the article below about methods of food preservation.

Micro-organisms cause food to decay. They require water, and sometimes oxygen, to grow and
reproduce. Many micro-organisms can be killed by heat or chemicals such as sodium chloride,
sodium nitrate, sodium nitrite and sulfur dioxide. Thus, these methods are used for preserving
food.

In preparing canned food, food is first placed in metal cans. These metal cans are usually
made of steel, an alloy, and coated with tin on the inside and the outside. To preserve the food,
air is sucked out of the cans before they are sealed. The cans are then heated under pressure
to kill the micro-organisms inside.

Another method of preservation is salting. Salt (sodium chloride) is added to meat to kill micro-
organisms. Salting can be done in several ways. Dry-salting involves burying meat in granular
salt, while brine-curing involves soaking meat in concentrated salt solution.

(a) State two differences between compounds and mixtures.


The components in a mixture can be separated by physical methods, while the components of a

compound can only be separated by chemical reactions. The components of a mixture are not fixed in

proportion, while those of a compound are fixed.

© 2013 Marshall Cavendish International (Singapore) Private Limited Worksheet 3 21

(S)CMWB_03.indd 21 8/14/12 4:14 PM


(b) Tabulate the substances mentioned in the article as elements, compounds or mixtures.

Element Compound Mixture


oxygen sodium nitrate air
tin sodium nitrite steel
sulfur dioxide salt solution
water
salt (sodium chloride)

(c) Air contains nitrogen, oxygen, carbon dioxide and noble gases (mostly argon). Draw a diagram
to represent the components of air.

WORK ON IT!
In the table below, circle the question(s) that you have answered incorrectly. Revisit the relevant
section(s) in the textbook to strengthen your understanding of the key concept(s).

Question number(s) Textbook


Learning outcomes Multiple- Free- section(s)
Structured to revise
choice response
(a) Define an element. 1(a), 1(c),
4, 6 – 4.1
2(a)(i)
(b) State the chemical symbols for a number
– 1(b) – 4.1
of elements.
(c) Define a compound. 2(a)(ii),
3, 5, 7 – 4.2
2(b), 2(c)
(d) Describe the differences between elements
8 – 1(b), 1(c) 4.2
and compounds.
(e) Deduce the formula of a compound from
1 2(d) – 4.2
the ratio of elements present.
(f) Define a mixture. 2 2(a)(iii) 1(c) 4.3
(g) Differentiate between compounds and
9 – 1(a), 1(b) 4.3
mixtures.
(h) Identify substances and their purity, given
9 – – 3.7
their melting and boiling points.

22 Worksheet 3 © 2013 Marshall Cavendish International (Singapore) Private Limited

(S)CMWB_03.indd 22 8/14/12 4:14 PM


Name: ( )

Worksheet 4
Class: Date:

Atomic Structure

for Chapter 5
ANSWER IT RIGHT!
Go through the worked example. Use the tips to guide you in understanding and answering
the questions.

The figure below shows the electronic structures of five atoms, P, Q, R, S and T.

2p 14p 9p 10p 14p


2n 14n 10n 10n 16n

P Q R S T

Which of the following statements is correct?


1 P and S are Group 0 elements in the Periodic Table.
2 Q and T are isotopes.
3 R and T have the same nucleon number.
A 1 and 2
B 1 and 3
C 2 and 3
D 1, 2 and 3

Not all Group 0 elements have eight electrons in their outer shell. Helium has only two outer electrons.

Thought Process
• Check the number of outer electrons in each atom. In general, elements in the same group have the
same number of outer electrons. Helium in Group 0 is an exception.
• Isotopes are atoms with the same number of protons but different numbers of neutrons.
• The nucleon number is the sum of the number of protons and neutrons in an atom.

Answer
A

© 2013 Marshall Cavendish International (Singapore) Private Limited Worksheet 4 23

(S)CMWB_04.indd 23 8/30/12 9:32 AM


Section A: Multiple-Choice Questions
Choose the correct answer and write its letter in the brackets provided.

1. What is the relative mass and charge of a neutron compared to a proton?


Relative mass Relative charge
A 1 0
1840
B 1 –1
1840
C 1 1
1840
D 1 0 ( D )

2. What particles are found in the nucleus of an atom?


A Protons only
B Electrons and protons
C Protons and neutrons
D Electrons and neutrons ( C )

3. Which statement is true?


A The nucleon number is the number of neutrons in the nucleus.
B The nucleon number is the sum of the number of protons and neutrons.
C The proton number is the number of neutrons in the nucleus.
D The proton number is the sum of the number of protons and electrons. ( B )

4. How many neutrons are there in an atom of radium, 226


88 Ra?
A 88
B 138
C 226
D 314 ( B )

5. Phosphorus has an atomic number of 15. Which statement is true?


A A phosphorus atom has 15 electrons.
B A phosphorus atom has 15 neutrons.
C A phosphorus atom has 15 protons, neutrons and electrons.
D A phosphorus atom has a mass number of 15. ( A )

6. Which statement is not true about isotopes of an element?


A They have the same arrangement of electrons.
B They have the same chemical properties.
C They have the same number of protons.
D They have the same physical properties. ( D )

24 Worksheet 4 © 2013 Marshall Cavendish International (Singapore) Private Limited

(S)CMWB_04.indd 24 8/30/12 9:32 AM


7. Which of the following is correct about the two isotopes of oxygen, 16
8O and 18
8 O?

16
8 0 18
8 O

A Electronic configuration 2, 8, 6 2, 8, 6
B Number of valence electrons 8 8
C Chemical formula of oxygen gas formed O2 O3
D Number of neutrons 8 10 ( D )

8. Elements W, X, Y and Z have proton numbers of 10, 12, 14 and 16 respectively. Which element
does not have the same number of electron shells as the other three?
A W B X
C Y D Z ( A )

9. Which diagram represents the electronic structure of an oxygen atom?

O O O O

A B C D ( B )

10. The number of valence electrons for the first 12 elements in the Periodic Table, hydrogen to
magnesium, was plotted against their proton numbers. Which of the following is the correct graph?
A B
14 14
12 12
valence electrons

valence electrons

10 10
Number of

Number of

8 8
6 6
4 4
2 2

1 2 3 4 5 6 7 8 9 10 11 12 1 2 3 4 5 6 7 8 9 10 11 12
Proton number Proton number

C D
14 14
12 12
valence electrons

valence electrons

10 10
Number of

Number of

8 8
6 6
4 4
2 2

1 2 3 4 5 6 7 8 9 10 11 12 1 2 3 4 5 6 7 8 9 10 11 12
Proton number Proton number

( C )

© 2013 Marshall Cavendish International (Singapore) Private Limited Worksheet 4 25

(S)CMWB_04.indd 25 8/30/12 9:32 AM


Section B: Structured Questions
Answer the following questions in the spaces provided.

1. The table below shows the relative mass and relative charge of three types of subatomic particles.

M N O
Relative mass 1 1 x
Relative charge +1 y –1

(a) Identify the subatomic particles M, N and O.

M: Proton N: Neutron O: Electron

(b) What are the values of x and y?


1
x: 1840 y: 0

2. Lithium (Li), sodium (Na) and potassium (K) are in the same group of the Periodic Table. Complete
the table below.

Electronic
Element Number of electrons Number of neutrons
configuration
7
3 Li 2, 1 3 4
Na
23
11 2, 8, 1 11 12
39
19 K 2, 8, 8, 1 19 20

3. The electronic structure of a beryllium atom is shown in the diagram below.

Key:
electron
+ proton
+ neutron
+ +
+

(a) Write down the symbol of this element, including its proton number and nucleon number.
9
Be
4

(b) Write down the electronic configuration of beryllium.

2, 2

(c) How many valence electrons are there in an atom of beryllium?

Two

26 Worksheet 4 © 2013 Marshall Cavendish International (Singapore) Private Limited

(S)CMWB_04.indd 26 8/30/12 9:32 AM


(d) Deduce the group that beryllium belongs to in the Periodic Table.

Group II

4. Correct each of the following statements.


(a) The nucleus of a fluorine atom 冸 F冹 contains 19 neutrons and 9 protons.
The nucleus of a fluorine atom 冸 F冹 contains 10 neutrons and 9 protons.

(b) The first shell in an atom can hold a maximum of eight electrons.
The first shell in an atom can hold a maximum of two electrons.

(c) A helium atom has eight electrons in its outer shell.


A helium atom has two electrons in its outer shell.

(d) Nitrogen has an atomic number of 7. Therefore, there are seven electrons in the outer shell
of its atom.
Nitrogen has an atomic number of 7. Therefore, there are five electrons in the outer shell of its atom.

5. The figures below show the electronic structures of six atoms, A to F.

18p 12p 3p
22n 14n 4n

A B C

6p 20p 12p
6n 20n 12n

D E F
(a) Which atoms
(i) are isotopes; B and F

(ii) have the same nucleon number; A and E

(iii) are in the same group of the Periodic Table? B and F

© 2013 Marshall Cavendish International (Singapore) Private Limited Worksheet 4 27

(S)CMWB_04.indd 27 8/30/12 9:32 AM


(b) (i) Which atom has a mass twice that of atom D?
F

(ii) Using the Periodic Table, identify element D.


Carbon

(c) Atom A has an isotope, Q, with a nucleon number of 38. In the space below, draw the full
electronic structure of Q.

18p
20n

6. At airports, security officers fire neutrons at luggage to detect any hidden explosives. Most
explosives contain nitrogen-14. When neutrons collide with nitrogen-14, a radioisotope, nitrogen-15,
is formed and high-energy rays are given off. These rays, when detected, indicate the presence of
explosives in the luggage.

(a) (i) Write down the electronic arrangement of an atom of nitrogen-15.


2, 5

(ii) Using the Periodic Table, name another element that has similar chemical properties as
nitrogen-15.
Phosphorus/ Arsenic/ Antimony

(iii) Explain your answer in (ii).


Phosphorus/ Arsenic/ Antimony is in the same group as nitrogen. Elements in the same group

have the same number of valence electrons and thus share similar chemical properties.

(b) State the similarity in the atomic structures of nitrogen-15 and oxygen-16.

Both nitrogen-15 and oxygen-16 have the same number of neutrons.

28 Worksheet 4 © 2013 Marshall Cavendish International (Singapore) Private Limited

(S)CMWB_04.indd 28 8/30/12 9:32 AM


Section C: Free-Response Questions
Answer the following questions.

1. A sample of volcanic rock consists of two isotopes of helium, helium-3 and helium-4, in the following
mass ratio:
3
2 He : 42He = 1 : 99

(a) Explain what is meant by the term ‘isotopes’.


Isotopes are atoms of the same element with the same number of protons but different numbers of

neutrons.

(b) (i) State the difference between the atomic structures of helium-3 and helium-4.
A helium-3 atom has two protons and one neutron, while a helium-4 atom has two protons and

two neutrons.

(ii) Explain why helium-3 and helium-4 have similar chemical properties.
The chemical properties of an element depend on its outer electrons which are involved in

chemical reactions. Both helium-3 and helium-4 have two outer electrons. Hence, they have

similar chemical properties.

(c) This sample of volcanic rock also contains hydrogen-3.


(i) Write the symbol of hydrogen-3, showing the atomic and mass numbers.
3
1H

(ii) List one similarity and one difference between hydrogen-3 and helium-3.
Both hydrogen-3 and helium-3 have the same nucleon number. The difference is that hydrogen-3

has two neutrons and one proton, while helium-3 has one neutron and two protons.

2. The table below shows the nucleon numbers and proton numbers of atoms W, X, Y and Z.
(Note: W, X, Y and Z are not chemical symbols of elements.)

Atom Nucleon number Proton number


W 2 1
X 14 6
Y 19 9
Z 37 17

© 2013 Marshall Cavendish International (Singapore) Private Limited Worksheet 4 29

(S)CMWB_04.indd 29 8/30/12 9:32 AM


Draw the electronic structures of atoms W, X, Y and Z.

Atom W Atom X

1p 6p
1n 8n

Atom Y Atom Z

9p 17p
10n 20n

WORK ON IT!
In the table below, circle the question(s) that you have answered incorrectly. Revisit the
relevant section(s) in the textbook to strengthen your understanding of the key concept(s).

Question number(s) Textbook


Learning outcomes Multiple- Free- section(s)
Structured to revise
choice response
(a) State the relative charges and relative
masses of a proton, a neutron and an 1 1(b) – 5.1
electron.
(b) Describe the structure of an atom. 2 1(a) – 5.1
(c) Define proton (atomic) number and 5(a)(ii), 5(b),
3 – 5.2
nucleon (mass) number. 5(c)
(d) Deduce the numbers of protons, 2, 4(a), 5(c),
neutrons and electrons in an atom. 4, 5 1(c)(ii) 5.2
6(b)
(e) Interpret and use symbols that
represent an element’s nucleon and 1(c)(i)
4, 10 2, 3(a), 4(a) 5.2
proton numbers  X .

(f) Define isotopes. 6, 7 5(a)(i), 5(c) 1(a), 1(b) 5.3


(g) Use diagrams to describe atoms as
2, 3(b), 3(c),
containing
3(d), 4(b),
• protons and neutrons in the
8, 9, 10 4(c), 4(d), 2 5.4
nucleus;
5(a)(iii), 5(c),
• electrons arranged in the electron
6(a)
shells (energy levels).

30 Worksheet 4 © 2013 Marshall Cavendish International (Singapore) Private Limited

(S)CMWB_04.indd 30 3/10/16 9:33 AM


Name: ( )

Worksheet 5
Class: Date:

Chemical Bonding

for Chapters 6 and 7


ANSWER IT RIGHT!
Go through the worked example. Use the tips to guide you in understanding and answering
the questions.

Aluminium oxide is a white solid that contains ionic bonds. Due to its hardness and strength, it is
commonly used as an abrasive such as sandpaper.
(a) What do you understand by ‘ionic bond’? Give an example in your answer.

The phrase ‘what do you understand by’ or ‘what is meant by’ implies that a definition should be given.

Answer
An ionic bond is the force of attraction between a positive ion and a negative ion in an ionic
compound. For example, the forces of attraction between aluminium ions and oxide ions make up
the ionic bonds in aluminium oxide.

(b) Draw the electronic structure of an aluminium ion and an oxide ion.

• When asked to draw the electronic structure of an atom or ion, read the question carefully to see whether
you are required to show all the shells or only the outer shell.
• Unless otherwise stated in the question, draw the full electronic structure, showing all the shells.
• When drawing the electronic structure of an ion, write the charge of the ion at the top right-hand corner
of the structure.

Thought Process
• Aluminium is a metal. It loses three valence electrons to attain an octet electronic configuration.
• Oxygen is a non-metal. It gains two electrons to attain an octet electronic configuration.

Answer
3+ 2–
Aluminium ion: Oxide ion:

Al O

(c) Explain the electrical conductivity of aluminium oxide in the solid and molten states.

Thought Process
Ionic compounds conduct electricity when molten due to the movement of free ions.

© 2013 Marshall Cavendish International (Singapore) Private Limited Worksheet 5 31

(S)CMWB_05.indd 31 8/14/12 4:22 PM


Answer
Aluminium oxide does not conduct electricity in the solid state because the Al3+ and O2– ions
cannot move freely. In the molten state, the ions are free to move about. Hence, aluminium oxide
conducts electricity when molten.

• A common mistake is to state that an ionic compound conducts electricity when molten because ‘the
molecules or ions can move’.
• Remember that a molten ionic compound contains positive and negative ions only, not molecules. It is
these mobile ions (ions which move about) that conduct electricity.
• Another common mistake is to state that an ionic compound conducts electricity when molten because
‘the ions and electrons can move’.
• Remember that there are no free electrons in ionic compounds.

TEST IT!

Section A: Multiple-Choice Questions


Choose the correct answer and write its letter in the brackets provided.

1. Which ion has the same number of electrons as the neon atom?

Ion O2– Na+ Cl –


Atomic number 8 11 17

A Na+ only
B Na+ and Cl –
C Na+ and O2–
D O2– and Cl – ( C )

2. Which of the following is true about the iron(II) ion, Fe2+, and the iron(III) ion, Fe3+?
A The compounds they form with the chloride ion, Cl –, have the same chemical formula.
B They have the same number of electrons.
C They have the same number of protons.
D They are formed from the iron atom by losing the same number of electrons. ( C )

3. Two elements, Z and Y, react to form a compound.

Z Y

What is the chemical formula of the compound?


A Z2Y
B Z2Y3
C ZY
D ZY2 ( A )

32 Worksheet 5 © 2013 Marshall Cavendish International (Singapore) Private Limited

(S)CMWB_05.indd 32 8/14/12 4:22 PM


4. Indium forms the indium(III) ion with the formula In3+. The sulfate ion has the formula SO42–. What
is the chemical formula of indium(III) sulfate?
A In(SO4)2
B In2(SO4)3
C In2SO4
D InSO4 ( B )

5. How many electrons are shared between the carbon and oxygen atoms in a carbon dioxide
molecule?
A 4 B 6 C 8 D 10 ( C )

6. The ‘dot and cross’ diagram below shows the arrangement of electrons in the compound XY3. (Only
the outer electrons are shown.)

Y X Y

Which elements are likely to be X and Y?


X Y
A sulfur chlorine
B phosphorus chlorine
C nitrogen oxygen
D carbon oxygen ( B )

7. Which compound contains both ionic and covalent bonds?


A Ammonia
B Ammonium chloride
C Hydrogen chloride
D Sodium chloride ( B )

8. Which of the following substances is likely to be potassium chloride?

Electrical conductivity
Substance Melting point/°C Solubility in water
when molten
A –114 nil soluble
B 180 nil insoluble
C 772 good soluble
D 1535 good insoluble
( C )

© 2013 Marshall Cavendish International (Singapore) Private Limited Worksheet 5 33

(S)CMWB_05.indd 33 8/14/12 4:22 PM


9. Which substance in the table below could be methanol?

Melting Boiling Electrical conductivity


Substance
point/°C point/°C in the liquid state in aqueous solution
A –114 –85 nil good
B –98 65 nil nil
C 180 218 nil nil
D 420 970 good good
( B )

10. Silicon carbide, SiC, is a very hard solid that melts at 2730°C. Which diagram best represents
its structure?

A B C D ( C )

11. A student carried out tests to find out the electrical conductivity of four substances. The results are
shown below.

Substance P conducts electricity only in aqueous solution.


Substance Q conducts electricity when molten and in aqueous solution.
Substance R conducts electricity in the solid state.
Substance S does not conduct electricity.

Which substances are likely to be P, Q, R and S?

P Q R S
A sulfur sodium iodide graphite hydrogen chloride
B hydrogen chloride sodium iodide graphite sulfur
C sodium iodide hydrogen chloride graphite sulfur
D hydrogen chloride sodium iodide sulfur graphite
( B )

34 Worksheet 5 © 2013 Marshall Cavendish International (Singapore) Private Limited

(S)CMWB_05.indd 34 3/10/16 10:29 AM


Section B: Structured Questions
Answer the following questions in the spaces provided.

1. (a) The chemical formula of ammonia is NH3. Draw a ‘dot and cross’ diagram to show the
arrangement of the electrons in ammonia. (Only the outer electrons need to be shown.)

N
H H

(b) The structural formula of phosphoric acid is shown on the right. From the O
structural formula, deduce the total number of electrons that are shared
P
between the phosphorus and oxygen atoms.
H O O H
O
There are three single bonds and one double bond between the phosphorus
H
and oxygen atoms in phosphoric acid. Each bond is formed by the sharing of

two electrons. Thus, the total number of electrons shared between the phosphorus and oxygen atoms

is 5 × 2 = 10.

2. The table below gives information about the compounds formed between chlorine and some
elements in Periods 2 and 3 of the Periodic Table.

Compound Boiling point/°C Compound Boiling point/°C


LiCl 1380 NaCl 1415
beryllium chloride 480 MgCl2 1410
BCl3 12 AlCl3 120
CCl4 80 silicon tetrachloride 60
NCl3 70 PCl5 80
Cl2O 2 SCl2 60
ClF –100 Cl2 –34

(a) Write the chemical formula of

(i) beryllium chloride; BeCl2

(ii) silicon tetrachloride. SiCl4

(b) Explain why neon and argon do not form compounds.


They have a stable octet electronic configuration and thus do not form compounds.

© 2013 Marshall Cavendish International (Singapore) Private Limited Worksheet 5 35

(S)CMWB_05.indd 35 8/14/12 4:22 PM


(c) Describe how bonds are formed in the compounds below.
(i) LiCl
A lithium atom loses an electron to a chlorine atom. The lithium and chloride ions formed are held

together by ionic bonds.

(ii) ClF
A chlorine atom shares a pair of electrons with a fluorine atom to form a single covalent bond.

(d) Explain, in terms of structure and bonding, the difference in the boiling points of NaCl and
CCl4.
In NaCl, the Na+ and Cl – ions are arranged in a giant lattice structure and the oppositely charged ions

are held together by strong forces of attraction (ionic bonds). A large amount of energy is needed to

break the strong ionic bonds in NaCl. In CCl4, the CCl4 molecules are held together loosely by weak

intermolecular forces. These intermolecular forces can be easily overcome. Thus, NaCl has a much

higher boiling point than CCl4.

Section C: Free-Response Questions


Answer the following questions.

1. (a) Magnesium oxide is used as a construction material because of its strength and fire
resistance. Its structure and physical properties are similar to those of sodium chloride. The
lattice structure of magnesium oxide is shown here.

Key:
Mg2+ ion
O2– ion

(i) Deduce that the chemical formula of magnesium oxide is MgO.


Each Mg2+ ion is surrounded by six O2– ions and vice versa. The overall ratio of Mg2+ ions to

O2– ions is 1 : 1. Thus, the chemical formula of magnesium oxide is MgO.

36 Worksheet 5 © 2013 Marshall Cavendish International (Singapore) Private Limited

(S)CMWB_05.indd 36 8/14/12 4:22 PM


(ii) Draw a ‘dot and cross’ diagram to show the bonding in magnesium oxide.
2+ 2–

Mg O

(b) The diagram below shows how aqueous sodium chloride can be tested for electrical
conductivity. State the result you would expect to see. Explain your answer.

battery bulb
switch

carbon rod carbon rod

aqueous sodium chloride beaker

The bulb lights up, showing that aqueous sodium chloride conducts electricity. This is because the

sodium and chloride ions are free to move about in aqueous solution.

2. Silicon and carbon are in Group IV of the Periodic Table. They react with oxygen to form compounds.

Silicon dioxide (SiO2), a white solid, is abundantly found in the Earth’s crust. It is insoluble in
water and has a melting point of 1713°C . It is used in the manufacture of glass and bricks. Carbon
dioxide is a colourless gas present in the atmosphere. It is soluble in water and has a boiling point
of –57°C.
(a) Draw a ‘dot and cross’ diagram to show the bonding in a carbon dioxide molecule. (Only the
outer electrons need to be shown.)

O C O

© 2013 Marshall Cavendish International (Singapore) Private Limited Worksheet 5 37

(S)CMWB_05.indd 37 8/14/12 4:22 PM


(b) By comparing the structures of silicon dioxide and carbon dioxide, explain why silicon dioxide
exists as a solid and carbon dioxide as a gas at room temperature.

Silicon dioxide has a giant molecular structure. The silicon and oxygen atoms are held together by

strong covalent bonds. A large amount of energy is required to break these strong bonds. Hence,

silicon dioxide has high melting and boiling points and exists as a solid at room temperature. Carbon

dioxide exists as simple molecules, which are held together by weak intermolecular forces. These

forces are easily overcome. Hence, carbon dioxide has low melting and boiling points and exists as a

gas at room temperature.

(c) Explain why silicon dioxide does not conduct electricity.

All the outer electrons of the silicon and oxygen atoms in silicon dioxide are used for bonding. There

are therefore no free electrons that move through the structure. Hence, silicon dioxide does not

conduct electricity.

3. Diamond and graphite are allotropes of carbon.

Explain the following in terms of bonding and structure.


(a) Diamond is hard but graphite is soft and slippery.

Diamond forms a three-dimensional structure in which each carbon atom is covalently bonded to four

other carbon atoms, which are in turn bonded to four more carbon atoms. These covalent bonds are

very strong and difficult to break. Thus, diamond is hard. Graphite is made up of layers of carbon

atoms which are held loosely by weak intermolecular forces of attraction. These layers of carbon

atoms slide over each other easily when a force is applied. Thus, graphite is soft and slippery.

(b) Graphite conducts electricity but not diamond.


Graphite is made up of layers of carbon atoms. Each carbon atom has one outer electron that is not

used for bonding. These electrons move freely along the layers from one carbon atom to the next, i.e.

they are delocalised, allowing graphite to conduct electricity. In diamond, all the outer electrons of

the carbon atoms are used to form covalent bonds. There are no free electrons that move through its

structure. Thus, diamond does not conduct electricity.

38 Worksheet 5 © 2013 Marshall Cavendish International (Singapore) Private Limited

(S)CMWB_05.indd 38 8/14/12 4:22 PM


4. Aluminium is a malleable metal with a silvery appearance. It has a melting point of 660°C. Due to
its good electrical conductivity, aluminium is used to make electrical transmission lines.
(a) With the aid of a diagram, describe the structure and bonding in aluminium.

In aluminium, the aluminium atoms are held strongly to each other by metallic bonds to form a
giant lattice structure. In the structure, a lattice of positive ions is surrounded by a ‘sea of mobile
electrons’.

(b) Based on your answer in (a), explain why aluminium


(i) is malleable;

In the aluminium metal lattice, the outer electrons do not belong to any aluminium atom, i.e.

they are delocalised. When sufficient force is applied, one layer of aluminium atoms can slide

over another without disrupting the metallic bonds.

(ii) conducts electricity;


The outer electrons within the aluminium metal lattice are delocalised and able to move freely

within the metal lattice.

(iii) has a high melting point.

A large amount of energy is needed to overcome the strong forces of attraction between

aluminium ions and the 'sea of mobile electrons' in the metal lattice structure.

(c) Draw a ‘dot and cross’ diagram to show the bonding in aluminium fluoride, AlF3. (Only the
outer electrons need to be shown.)
3+ –

Al 3 F

© 2013 Marshall Cavendish International (Singapore) Private Limited Worksheet 5 39

(S)CMWB_05.indd 39 8/14/12 4:22 PM


WORK ON IT!
In the table below, circle the question(s) that you have answered incorrectly. Revisit the
relevant section(s) in the textbook to strengthen your understanding of the key concept(s).

Question number(s) Textbook


Learning outcomes Multiple- Free- section(s)
Structured to revise
choice response
(a) Describe the stable electronic structure
– 2(b) – 6.1
of a noble gas.
(b) Describe the formation of positive ions
(cations) and negative ions (anions) to 1, 2 – – 6.2
achieve the noble gas configuration.
(c) Describe how ionic bonds are formed
7 2(c)(i) 1(a)(ii), 4(c) 6.3
between metals and non-metals.
(d) Deduce the formula of an ionic
compound from the charges on the ions 3, 4 2(a) – 6.4
and vice versa.
(e) State that ionic compounds form giant
lattice structures. – – 1(a)(i) 6.5
(f) Deduce the formulae of ionic
compounds from their lattice – – 1(a)(i) 6.5
structures.
(g) Relate the physical properties of ionic
8, 11 2(d) 1(b), 4(b) 6.5
compounds to their lattice structures.
(h) Describe the formation of a covalent
5, 6, 7 2(c)(ii) 2(a) 7.1
bond by the sharing of electrons.
(i) Describe the formation of covalent
bonds between non-metallic elements – 1(a) 2(a) 7.1
using ‘dot and cross’ diagrams.
(j) Deduce the arrangement of electrons in
– 1(a), 1(b) – 7.1
covalent molecules.
(k) Relate the physical properties of
covalent substances to their structure 9, 10, 11 2(d) 2(c), 3 7.2
and bonding.
(l) Compare the structures of simple
molecular substances and giant
– – 2(b) 7.2
molecular substances to deduce their
properties.
(m) Compare the bonding and structures of
diamond and graphite to deduce their – – 3 7.2
physical properties.
(n) Describe the structure of metals as
a lattice of positive ions in a ‘sea of – – 4(a) 7.3
electrons’.
(o) Relate the physical properties of metals
11 – 4(b) 7.3
to their structure.

40 Worksheet 5 © 2013 Marshall Cavendish International (Singapore) Private Limited

(S)CMWB_05.indd 40 3/10/16 9:49 AM


Name: ( )

Worksheet 6
Class: Date:

Chemical Calculations

for Chapters 8–10


ANSWER IT RIGHT!
Go through the worked example. Use the tips to guide you in understanding and answering
the questions.

Step 1: Excess iron(III) oxide, Fe2O3, was added to dilute hydrochloric acid, HCl. Iron(III) chloride, FeCl3,
and water were formed as products. The reaction mixture was filtered to remove unreacted
iron(III) oxide and the filtrate was collected.
Step 2: The filtrate obtained in step 1 was added in excess to 30 cm3 of 2.0 mol/dm3 sodium hydroxide
solution, NaOH. A brown precipitate of iron(III) hydroxide, Fe(OH)3, was formed.
(a) Write an ionic equation, including state symbols, to show the reaction in step 1.

• In writing state symbols, do not confuse (l) with (aq).


• The state symbol (l) refers to a pure liquid. In contrast, the symbol (aq) refers to a substance dissolved
in water. For example, molten sodium hydroxide is represented as NaOH(l), while an aqueous solution
of sodium hydroxide is represented as NaOH(aq).

Thought Process
• The first step in writing an ionic equation is to write the balanced chemical equation.
Fe2O3(s) + 6HCl(aq) 2FeCl3(aq) + 3H2O(l)
• Then, rewrite an equation in terms of ions and cancel out the spectator ions.
Fe2O3(s) + 6H+(aq) + 6Cl –(aq) 2Fe3+(aq) + 6Cl –(aq) + 3H2O(l)

Answer
Fe2O3(s) + 6H+(aq) 2Fe3+(aq) + 3H2O(l)

(b) (i) Write an ionic equation for the reaction in step 2.


(ii) Calculate the mass of iron(III) hydroxide precipitated in step 2.

• ‘Calculate’ means that a numerical answer is required.


• When doing calculations, you should show your working.

Thought Process
(ii) Since iron(III) chloride is in excess, sodium hydroxide solution is the limiting reactant. The
amount of product(s) formed is determined by the amount of limiting reactant used.

© 2013 Marshall Cavendish International (Singapore) Private Limited Worksheet 6 41

(S)CMWB_06.indd 41 8/23/12 11:04 AM


Answer
(i) Fe3+(aq) + 3OH–(aq) Fe(OH)3(s)
(ii) Number of moles of NaOH = concentration in mol/dm3 × volume in dm3
= 2.0 × 30
1000
= 0.06 mol
From the equation, 3 mol of OH– ions react to form 1 mol of Fe(OH)3.
Number of moles of Fe(OH)3 produced = 1 × 0.06
3
= 0.02 mol
Relative formula mass (Mr) of Fe(OH)3 = 56 + (3 × 16) + (3 × 1)
= 107
Mass of Fe(OH)3 = number of moles × molar mass
= 0.02 × 107
= 2.14 g

When calculating relative molecular mass or relative formula mass, make sure that you use relative atomic
masses and not atomic numbers.

TEST IT!

Section A: Multiple-Choice Questions


Choose the correct answer and write its letter in the brackets provided.

1. Which pair of standards is used to measure the relative molecular mass and the relative atomic
mass of a substance?
Relative molecular mass Relative atomic mass
A 1 mass of a carbon-12 atom 1 mass of a carbon-12 atom
12 12
B 1 mass of a carbon-12 atom mass of a hydrogen-1 atom
12
C mass of a hydrogen-1 atom 1 mass of a carbon-12 atom
12
D mass of a hydrogen-1 atom mass of a hydrogen-1 atom ( A )

2. What is the mass of oxygen in 74 g of calcium hydroxide, Ca(OH)2?


A 8g
B 16 g
C 32 g
D 64 g ( C )

3. Urea, CO(NH2)2, is used as a fertiliser. What is the percentage by mass of nitrogen in urea?
A 12%
B 23%
C 40%
D 47% ( D )

42 Worksheet 6 © 2013 Marshall Cavendish International (Singapore) Private Limited

(S)CMWB_06.indd 42 3/10/16 9:50 AM


4. Which of the following statements about ethanoic acid (CH3COOH) and methyl methanoate
(HCOOCH3) are true?
1 They contain the same percentage by mass of oxygen.
2 They have the same empirical formula.
3 They have the same relative molecular mass.
A 1 and 2
B 1 and 3
C 2 and 3
D 1, 2 and 3 ( D )

5. One of the reactions that takes place during the manufacture of iron is as shown:
2CO(g) + O2(g) 2CO2(g)
Which statement about this reaction is true?
A 2 g of carbon monoxide reacts with 1 g of oxygen.
B 2 mol of carbon dioxide can be obtained from 2 mol of oxygen.
C 28 kg of carbon monoxide produces 44 kg of carbon dioxide.
D 88 kg of carbon dioxide can be obtained from 16 kg of oxygen. ( C )

6. When calcium carbonate is heated strongly, it decomposes into calcium oxide and carbon
dioxide gas.
CaCO3(s) CaO(s) + CO2(g)
What is the volume of carbon dioxide gas produced at r.t.p. when 10 g of calcium carbonate is
heated strongly?
A 0.10 dm3
B 0.24 dm3
C 1.00 dm3
D 2.40 dm3 ( D )

7. A student carried out titration of sodium hydroxide with an unknown acid. He found that 20.0 cm3
of 1.5 mol/dm3 of sodium hydroxide exactly reacted with 10.00 cm3 of 1.0 mol/dm3 of the acid.
How many moles of sodium hydroxide would react with one mole of the acid?
A 2
B 3
C 4
D 5 ( B )

*8. Tin occurs naturally as tin(IV) oxide, SnO2, in the ore cassiterite. It is extracted by heating the ore
with carbon. The equation for the reaction is:
SnO2(s) + C(s) Sn(s) + CO2(g)
357 g of tin was obtained from 30.2 kg of cassiterite. What is the percentage yield of tin?
A 0.015%
B 0.15%
C 1.5%
D 15% ( C )

© 2013 Marshall Cavendish International (Singapore) Private Limited Worksheet 6 43

(S)CMWB_06.indd 43 8/23/12 11:04 AM


Use the following information to answer Questions 9 and 10.
A sample of seawater contains bromide ions with a concentration of 0.1 g/dm3. When chlorine gas is
bubbled through the sample, bromine gas is obtained. The equation of the reaction is:
2Br–(aq) + Cl2(g) Br2(g) + 2Cl –(aq)

9. What volume of chlorine gas is required to produce 0.008 tonne of bromine gas at room
temperature and pressure? (1 tonne = 106 g)
A 50 dm3
B 100 dm3
C 1200 dm3
D 2400 dm3 ( C )

*10. What volume of seawater is required to produce 0.008 tonne of bromine gas? (1 tonne = 106 g)
A 20 000 dm3
B 40 000 dm3
C 80 000 dm3
D 160 000 dm3 ( C )

Section B: Structured Questions


Answer the following questions in the spaces provided.

1. When aqueous sodium sulfate, Na2SO4, was added to aqueous lead(II) nitrate, Pb(NO3)2, lead(II)
sulfate was precipitated.
(a) Write a chemical equation for the reaction. Include state symbols.
Na2SO4(aq) + Pb(NO3)2(aq) 2NaNO3(aq) + PbSO4(s)

(b) Hence, write the ionic equation for the reaction in (a).

Pb2+(aq) + SO42–(aq) PbSO4(s)

(c) 200 cm3 of 0.1 mol/dm3 lead(II) nitrate solution was mixed with 100 cm3 of 0.1 mol/dm3
sodium sulfate solution.
(i) What is the limiting reactant in this reaction?
Number of moles of Pb(NO3)2 = volume (dm3) × concentration (mol/dm3)
= 200 × 0.1
1000
= 0.02 mol

Number of moles of Na2SO4 = 100 × 0.1


1000
= 0.01 mol

From the equation, 1 mol of Pb(NO3)2 reacts with 1 mol of Na2SO4.


0.02 mol of Pb(NO3)2 will react with 0.02 mol of Na2SO4.
However, only 0.01 mol of Na2SO4 was used.
Therefore, sodium sulfate is the limiting reactant.

44 Worksheet 6 © 2013 Marshall Cavendish International (Singapore) Private Limited

(S)CMWB_06.indd 44 9/11/12 6:07 PM


(ii) Calculate the mass of lead(II) sulfate precipitated.
From the equation, 1 mol of PbSO4 is produced from 1 mol of Na2SO4.
Number of moles of PbSO4 = 0.01 mol
Mr of PbSO4 = 207 + 32 + (4 × 16)
= 303
Mass of PbSO4 = number of moles × molar mass
= 0.01 × 303
= 3.03 g

2. A student was provided with the following solutions:


A: Unknown concentration of sodium hydroxide solution
B: 0.25 mol/dm3 of dilute sulfuric acid

The student titrated 25.0 cm3 portions of A in a conical flask against B in a burette. The results of
the experiment are shown below.

Titration number 1 2 3
3
Final burette reading/cm 19.10 18.70 18.70
Initial burette reading/cm3 0.00 0.00 0.00
3
Volume of B used/cm 19.10 18.70 18.70

(a) Using suitable sets of the titration results, calculate the average volume of the acid used for
complete neutralisation.
18.70 + 18.70
Average volume =
2
= 18.70 cm3

(b) (i) Write a chemical equation for the reaction between sodium hydroxide solution and dilute
sulfuric acid.
H2SO4(aq) + 2NaOH(aq) Na2SO4(aq) + 2H2O(l)

(ii) Hence, write the ionic equation for the reaction in (i).
H+(aq) + OH–(aq) H2O(l)

(c) Calculate the concentration, in mol/dm3, of the sodium hydroxide solution.

Number of moles of H2SO4 = 0.25 × 18.70


1000
= 4.68 × 10–3 mol
From the equation, 1 mol of H2SO4 reacts with 2 mol of NaOH.
Number of moles of NaOH = 2 × 4.68 × 10–3
= 9.36 × 10–3 mol

Concentration of NaOH = number of moles


3
volume in dm
= 9.36 × 10–3
0.025
= 0.374 mol/dm3

© 2013 Marshall Cavendish International (Singapore) Private Limited Worksheet 6 45

(S)CMWB_06.indd 45 3/10/16 9:53 AM


3. Oxygen is the third most abundant element in the universe by mass. The relative atomic mass of
oxygen is 16. The most stable form of oxygen is the diatomic form, known as dioxygen. The other
form of oxygen, known as ozone, has a relative molecular mass of 48. Ozone at the ground level
is harmful to the human respiratory system. In some countries, the recommended limit of
concentration of ground-level ozone is 1.57 × 10–4 g/m3.
(a) Define ‘relative atomic mass’.
The relative atomic mass of an atom is the mass of one atom of that element compared to 1 of the
12

mass of one carbon-12 atom.

(b) Based on the relative molecular mass of ozone, suggest the molecular formula of ozone.
03

(c) An environmental scientist found that there are 1 × 1018 molecules of ozone in 1 m3 of air.
Has the limit of 1.57 × 10–4 g/m3 been exceeded? Support your answer with relevant
calculations. (One mole of a substance contains 6 × 1023 particles.)
18
Number of moles of ozone in 1 m3 of air = 1 × 1023
6 × 10
= 1.667 × 10–6 mol
Mass of ozone in 1 m3 of air = 1.667 × 10–6 × 48
= 8.00 × 10–5 g
Hence, the limit has not been exceeded.

*4. Hydrogen fluoride attacks glass and is used to draw on glass. It is prepared by adding concentrated
sulfuric acid to calcium fluoride. The reaction takes place at room temperature.
CaF2(s) + H2SO4(l) 2HF(g) + CaSO4(s)
(a) When 15.6 g of a sample of calcium fluoride was reacted with 12.5 cm3 of 20.0 mol/dm3
sulfuric acid at r.t.p., 7.68 dm3 of hydrogen fluoride was formed.
(i) What is the limiting reactant in this reaction?
Mr of CaF2 = 40 + (19 × 2)
= 78
Number of moles of CaF2 used = 15.6
78
= 0.2 mol

Number of moles of H2SO4 used = 20.0 × 12.5


1000
= 0.25 mol
From the equation, 1 mol of H2SO4 reacts with 1 mol of CaF2.
0.25 mol of H2SO4 will react with 0.25 mol of CaF2.
However, only 0.2 mol of CaF2 was used.
∴ CaF2 is the limiting reactant.

46 Worksheet 6 © 2013 Marshall Cavendish International (Singapore) Private Limited

(S)CMWB_06.indd 46 8/23/12 11:04 AM


(ii) Calculate the theoretical volume of hydrogen fluoride that was formed.
From (a)(i), number of moles of CaF2 used = 0.2 mol
From the equation, 2 mol of HF are produced from 1 mol of CaF2.
Number of moles of HF formed = 2 × 0.20
= 0.40 mol
Theoretical volume of HF formed = number of moles × 24 dm3
= 0.40 × 24
= 9.6 dm3

(b) Calculate the percentage purity of calcium fluoride.


3
Number of moles of HF formed = volume in 3dm
24 dm
= 7.68
24
= 0.32 mol
From the equation, 1 mol of CaF2 produces 2 mol of HF.

Number of moles of pure CaF2 = 0.32


2
= 0.16 mol
Mass of pure CaF2 = number of moles × molar mass
= 0.16 × 78
= 12.48 g

Percentage purity of CaF2 = mass of pure CaF2 in sample × 100%


mass of sample
= 12.48 × 100%
15.6
= 80%

*5. The recommended dosage of paracetamol for a child weighing 10–16 kg is 160 mg. A brand of
medicine contains 0.20 mol of paracetamol in 1 dm3 of solution. What is the maximum number of
2.5 cm3 spoonfuls of the medicine a parent can safely feed her child, if her child weighs 15 kg?
(Mr: paracetamol = 151)
Number of moles of paracetamol in 1 spoonful (2.5 cm3) of the medicine
= volume (dm3) × concentration (mol/dm3)
= 2.5 × 0.20
1000
= 0.0005 mol
Mass of paracetamol in 1 spoonful of the medicine = number of moles × molar mass
= 0.0005 × 151
= 0.0755 g
= 75.5 mg
Number of spoonfuls of medicine that contains the recommended dosage of paracetamol
= 160
75.5
= 2.12
∴ The maximum number of 2.5 cm3 spoonfuls of the medicine that the parent can feed her child is 2.

© 2013 Marshall Cavendish International (Singapore) Private Limited Worksheet 6 47

(S)CMWB_06.indd 47 8/23/12 11:04 AM


Section C: Free-Response Questions
Answer the following questions.

1. Hydrated compounds are compounds that contain water of crystallisation. The information of some
hydrated compounds is given in the table below.

Percentage by
Compound Formula
mass of water/%
hydrated magnesium sulfate MgSO4.7H2O 51.2
hydrated sodium carbonate Na2CO3.10H2O 62.9
hydrated zinc nitrate Zn(NO3)2.xH2O 36.5

(a) Given that one mole of a substance contains 6 × 1023 particles, calculate the number of water
molecules present in 5.00 g of hydrated magnesium sulfate.

Mass of water in hydrated magnesium sulfate = 51.2 × 5.00


100
= 2.56 g
Number of moles of water molecules = mass in g
molar mass in g/mol
= 2.56
18
= 0.142 mol
Number of water molecules = 0.142 × 6 × 1023
= 8.52 × 1022

(b) When solid hydrated sodium carbonate is heated, the water of crystallisation is given off as
steam. Write a balanced chemical reaction, with state symbols, to represent this change.
Na2CO3.10H2O(s) Na2CO3(s) + 10H2O(g)

(c) Calculate the value of x in Zn(NO3)2.xH2O.


Mr of Zn(NO3)2.xH2O = 65 + 2(14) + 6(16) + x(18)
= 189 + 18x

Percentage by mass of H2O = (number of H2O molecules) × (Mr of H2O) × 100%


Mr of Zn(NO3)2.xH2O
36.5 = 18x × 100
(189 + 18x)
18x = 0.365(189 + 18x)
x = 6 (round off to the nearest whole number)

48 Worksheet 6 © 2013 Marshall Cavendish International (Singapore) Private Limited

(S)CMWB_06.indd 48 8/23/12 11:04 AM


*2. The following results were obtained in an experiment to determine the formula of an oxide of
mercury. The oxide is a red solid. It decomposed into its elements when heated at r.t.p. (One mole
of a gas occupies 24 dm3 at r.t.p.)
Mass of empty test tube = 15.45 g
Mass of test tube + oxide of mercury = 17.61 g
Volume of oxygen collected at r.t.p. = 120 cm3
(a) Draw a labelled diagram of the apparatus that can be used to perform this experiment.
gas syringe

oxygen collected

test tube

mercury

oxide of mercury

Bunsen burner

(b) State a precaution to take when performing this experiment.

Conduct the experiment in the fume cupboard.

(c) What observation (other than colour change) indicates that the reaction is complete?

No more oxygen is given off (reading on the gas syringe stays constant).

(d) From the results of the experiment, determine the molecular formula of this oxide of mercury.
(One mole of this oxide of mercury weighs 217 g.)
Mass of oxide of mercury used = 17.61 – 15.45 = 2.16 g
Mass of oxygen produced = ( 120 ÷ 24 dm3) × 32 = 0.16 g
1000
Mass of mercury produced = 2.16 – 0.16 = 2.00 g

Element Mercury Oxygen


Mass/g 2.00 0.16
Relative atomic mass 201 16
Number of moles/mol 2.00 = 0.01 0.16 = 0.01
201 16
Mole ratio 1 1

∴ The empirical formula of the oxide of mercury is HgO.


Let the molecular formula of the oxide of mercury be (HgO)n.
Relative mass of the oxide of mercury from empirical formula = 201 + 16 = 217
n= relative molecular mass of oxide of mercury = 217 = 1
relative mass of oxide of mercury from empirical formula 217
∴ The molecular formula of the oxide of mercury is HgO.

© 2013 Marshall Cavendish International (Singapore) Private Limited Worksheet 6 49

(S)CMWB_06.indd 49 3/17/16 11:09 AM


WORK ON IT!
In the table below, circle the question(s) that you have answered incorrectly. Revisit the
relevant section(s) in the textbook to strengthen your understanding of the key concept(s).

Question number(s) Textbook


Learning outcomes Multiple- Free- section(s)
Structured to revise
choice response
(a) Interpret chemical equations with state
5 – – 8.1
symbols.
(b) Write balanced chemical equations
– 1(a), 2(b)(i) 1(b) 8.1
with state symbols.
(c) Write ionic equations with state
– 1(b), 2(b)(ii) – 8.2
symbols.
(d) Define relative atomic mass (Ar). 1 3(a) – 9.1
(e) Define relative molecular mass (Mr). 1 – – 9.2
(f) Calculate relative molecular mass or
2, 4 – – 9.2
relative formula mass of a substance.
(g) Convert number of particles into
number of moles of particles and vice – – 1(a) 9.3
versa.
(h) Perform calculations involving the
number of moles, mass and molar 2 3(c) – 9.3
mass of a substance.
(i) Determine the percentage composition
3, 4 – – 9.4
of compounds from given data.
(j) Determine the empirical and molecular
formulae of a compound from given 4 3(b) 1(c), 2 9.5
data.
(k) Perform calculations involving molar
gas volume and the number of moles – 4(a)(ii) – 9.6
of a gas.
(l) Perform calculations involving the
concentration of a solution (g/dm3 or 10 – – 9.7
mol/dm3).
(m) Calculate the masses of reactants and
products by using the mole ratio in a 5 1(c)(ii) – 10.1
chemical equation.
(n) Calculate the volumes of gaseous
reactants and products using the mole 6, 9 – – 10.2
ratio in a chemical equation.
(o) Perform stoichiometric calculations
– 1(c)(i), 4(a)(i) – 10.3
involving the idea of limiting reactants.
(p) Process the results of volumetric
– 2(a), 2(c) – 10.4
experiments.
(q) Perform stoichiometric calculations
7 2(c), 5 – 10.4
involving concentrations of solutions.
(r) Calculate the percentage yield and
8 4(b) – 10.5
percentage purity of a substance.

50 Worksheet 6 © 2013 Marshall Cavendish International (Singapore) Private Limited

(S)CMWB_06.indd 50 8/23/12 11:04 AM


Name: ( )

Worksheet 7
Class: Date:

Acids and Bases

for Chapter 11
ANSWER IT RIGHT!
Go through the worked example. Use the tips to guide you in understanding and answering
the questions.

The diagram below shows the set-ups of two experiments.

hydrogen chloride in aqueous solution of


dry organic solvent hydrochloric acid

magnesium ribbon magnesium ribbon

Experiment I: Magnesium ribbon is Experiment II: Magnesium ribbon is


added to HCl in dry organic solvent. added to an aqueous solution of HCl.

Explain the likely observations for both experiments. Construct an equation for any reaction that occurs.

• Look out for the term ‘explain’ in the question. Your answer must make use of some reasoning or reference
to a theory to explain why something happens.
• In this case, you need to refer to the factors that affect the properties of acids to explain the difference in
the observations for both experiments.

Thought Process
Recall the role of water in acidity. Acids only display their properties when they are dissolved in water.

Answer
In Experiment I, no change is observed. In organic solvents, HCl exists as covalent molecules. It does
not ionise to form hydrogen ions (H+), which are responsible for the acidic properties. Therefore, HCl in
an organic solvent does not react with magnesium.

In Experiment II, effervescence is observed. In the presence of water, HCl ionises to form hydrogen
ions (H+).

HCl(aq) H+(aq) + Cl –(aq)

Aqueous HCl thus reacts with magnesium to form magnesium chloride and hydrogen gas.

2HCl(aq) + Mg(s) MgCl2(aq) + H2(g)

© 2013 Marshall Cavendish International (Singapore) Private Limited Worksheet 7 51

(S)CMWB_07.indd 51 8/14/12 4:38 PM


TEST IT!

Section A: Multiple-Choice Questions


Choose the correct answer and write its letter in the brackets provided.

1. Which statement is true for all aqueous solutions of acids?


A They have a pH value greater than 7.
B They react with ammonium chloride to give ammonia gas.
C They react with copper to give hydrogen.
D They react with sodium carbonate to give carbon dioxide. ( D )

2. Sulfuric acid has many uses in daily life. Which of the following is not a use of sulfuric acid?
A Acid in car batteries
B Bleaching agent
C Manufacture of detergents
D Manufacture of fertilisers ( B )

3. Which statement about sodium hydroxide is incorrect?


A It is a soluble base.
B It reacts with ammonium salts to produce ammonia gas.
C It reacts with copper(II) sulfate to form two salts.
D It reacts with dilute nitric acid to form a sodium salt. ( C )

4. Which statement is true about a weak acid and a strong acid?


A A strong acid has a higher pH value than a weak acid.
B A weak acid has a higher concentration of hydrogen ions in solution than a strong acid.
C A strong acid is completely ionised in solution while a weak acid is only partially ionised in
solution.
D A weak acid reacts faster with metals than a strong acid. ( C )

5. Which compound should a farmer add to neutralise acidic soil?


A Ammonium nitrate
B Calcium hydroxide
C Potassium nitrate
D Sodium hydroxide ( B )

6. An oxide, Z, reacts with both dilute hydrochloric acid and sodium hydroxide solution to form a salt
and water. What type of oxide is Z?
A Acidic B Amphoteric C Basic D Neutral ( B )

7. Which statement is true at the point where hydrochloric acid is exactly neutralised by sodium
hydroxide solution?
A The concentration of hydrogen ions remains unchanged.
B The concentration of hydrogen ions is equal to the concentration of hydroxide ions.
C The concentration of hydrogen ions is greater than the concentration of hydroxide ions.
D The concentration of hydrogen ions is less than the concentration of hydroxide ions. ( B )

52 Worksheet 7 © 2013 Marshall Cavendish International (Singapore) Private Limited

(S)CMWB_07.indd 52 8/14/12 4:38 PM


8. The tables below show the colours of methyl orange and phenolphthalein at different pH ranges.

Methyl orange Phenolphthalein


pH range 1–4 4.1–14 pH range 1–10 10.1–14
Colour red yellow Colour colourless pink

What would be the colour of each indicator in pure water?


Methyl orange Phenolphthalein
A red colourless
B red pink
C yellow colourless
D yellow pink ( C )

9. A 1.0 mol/dm3 solution of an acid T has a pH value of 5. Which of the following statements best
describes T?
A It is a poor conductor of electricity.
B It is a weak acid.
C It reacts with water to produce a higher concentration of hydrogen ions.
D The acid is only slightly soluble in water. ( B )

Section B: Structured Questions


Answer the following questions in the spaces provided.
1. Below are some reactions of dilute sulfuric acid.

no reaction

+ metal M
+ sodium
hydroxide
Solution T solution + metal N Colourless gas P
dilute sulfuric acid
is formed. is given off.

+ sodium
carbonate solution

Colourless gas Q is given


off. Q produces a white
precipitate with limewater.

© 2013 Marshall Cavendish International (Singapore) Private Limited Worksheet 7 53

(S)CMWB_07.indd 53 8/14/12 4:38 PM


(a) Suggest the identities of
(i) metal M;

Copper/ silver/ gold (any metal below hydrogen in the reactivity series)

(ii) metal N;

Magnesium/ zinc/ iron (any reactive metal except alkali metals)

(iii) gas P;
Hydrogen

(iv) gas Q.
Carbon dioxide

(b) When solution T is evaporated, a white crystalline solid is obtained. Solid T contains 19.17%
sodium, 0.83% hydrogen, 26.67% sulfur and 53.33% oxygen. The relative molecular mass of
T is 120. What is the molecular formula of T?

Element Na H S O

Percentage/% 19.17 0.83 26.67 53.33

Ar 23 1 32 16

Number of 19.17 = 0.83 0.83 = 0.83 26.67 = 0.83 53.33 = 3.33


moles/mol 10 1 32 16

Mole ratio 1 1 1 4

The empirical formula of T is NaHSO4.


Let the molecular formula of T be (NaHSO4)n.
Relative mass of T from empirical formula = 23 + 1 + 32 + (4 × 16)
= 120
n = relative molecular mass of T
relative mass of T from empirical formula
= 120
120
=1
∴ The molecular formula of of T is NaHSO4.

2. Milk of magnesia is a type of medicine which works as an antacid to treat stomach problems.
Milk of magnesia is a mixture of magnesium hydroxide and water. The solubility of magnesium
hydroxide in water is low.
(a) Suggest a reason why a mixture of magnesium hydroxide and water is known as milk of
magnesia.

Magnesium hydroxide does not dissolve easily in water. Hence, magnesium hydroxide and water form

a suspension (and not a solution). The appearance of this suspension is similar to that of milk.

54 Worksheet 7 © 2013 Marshall Cavendish International (Singapore) Private Limited

(S)CMWB_07.indd 54 8/14/12 4:38 PM


(b) Suggest how milk of magnesia works as an antacid. Write a chemical equation to support
your explanation.
The magnesium hydroxide in milk of magnesia is alkaline. It reacts with H+ ions in stomach acid

(neutralisation reaction).

Mg(OH)2 + 2H+ Mg2+ + 2H2O or Mg(OH)2 + 2HCl MgCl2 + 2H2O

(c) Other than the reaction in (b), suggest another reaction which milk of magnesia can undergo.
Write a chemical equation for this reaction.
Milk of magnesia also reacts with ammonium salts (e.g. ammonium chloride) to produce ammonia gas.

Mg(OH)2 + 2NH4+ Mg2+ + 2H2O + 2NH3 or Mg(OH)2 + 2NH4Cl MgCl2 + 2H2O + 2NH3

Section C: Free-Response Questions


Answer the following questions.

1. The pH values of solutions can be determined by using Universal Indicator or a pH sensor attached
to a data logger.

Hydrochloric acid (HCl) of different concentrations have different pH values. The table below shows
the pH values and colours of Universal Indicator in hydrochloric acid of different concentrations.

Concentration of HCl pH value measured by pH sensor Colour of Universal


/(mol/dm3) attached to data logger Indicator
0.1 1.0 red
0.001 3.0 orange
0.00001 5.0 yellow

(a) Using the data in the table, briefly describe how the pH value varies with the concentration of
hydrochloric acid.
The pH value increases as the concentration of hydrochloric acid decreases.

(b) Bernice tried to determine the pH value of 0.1 mol/dm3 sulfuric acid using both the pH sensor
and Universal Indicator. When she measured the pH of the acid using a pH sensor, the data
logger attached showed a value of 0.7.
(i) Explain why the pH value of 0.1 mol/dm3 sulfuric acid differed from that of 0.1 mol/dm3
hydrochloric acid.

The pH value of a solution is dependent on the concentration of H+ ions. The higher the

concentration of H+ ions, the lower the pH value. 0.1 mol/dm3 hydrochloric acid ionises to give

0.1 mol/dm3 H+ ions. 0.1 mol/dm3 sulfuric acid ionises to give 0.2 mol/dm3 H+ ions. Hence, the

pH value observed for 0.1 mol/dm3 sulfuric was smaller than that for 0.1 mol/dm3 hydrochloric

acid.

© 2013 Marshall Cavendish International (Singapore) Private Limited Worksheet 7 55

(S)CMWB_07.indd 55 8/14/12 4:38 PM


(ii) Bernice concluded that Universal Indicator cannot be used to differentiate between
0.1 mol/dm3 sulfuric acid and 0.1 mol/dm3 hydrochloric acid. Do you agree with her?
Why?
Yes. Both 0.1 mol/dm3 sulfuric acid and 0.1 mol/dm3 hydrochloric acid will give a red colour with

Universal Indicator.

(iii) Bernice also measured the pH of 0.00001 mol/dm3 of ethanoic acid using the pH
sensor. Ethanoic acid is a weak acid. Suggest a likely pH value that she would observe
and explain your answer.
The pH value is likely to be between 5 to 7. Hydrochloric acid is a strong acid which fully

ionises in water. Ethanoic acid is a weak acid which does not fully ionise in water. Hence, the

concentration of H+ ions in 0.00001 mol/dm3 ethanoic acid will be lower than that in

0.00001 mol/dm3 hydrochloric acid. Therefore, the pH value of ethanoic acid will be greater

than 5.

*2. Barium hydroxide reacts with sulfuric acid to form barium sulfate precipitate. The equation for this
reaction is as follows:
Ba(OH)2(aq) + H2SO4(aq) BaSO4(s) + 2H2O(l)
In an experiment, 1.0 mol/dm3 sulfuric acid was gradually added to 50 cm3 of barium hydroxide
solution. The mixture was continually stirred with an iron stirrer covered in plastic. The mixture was
connected to an ammeter. The reading was taken after the addition of every 1 cm3 of sulfuric acid.
The graph below shows the results obtained from the experiment.

Current/A 0.50
0.45
0.40
0.35
0.30
0.25
0.20
0.15
0.10
0.05 End-point

0 1 2 3 4 5 6 7 8 9 10
3
Volume of sulfuric acid added/cm

(a) Write an ionic equation, including state symbols, for


(i) the formation of the barium sulfate precipitate;
Ba2+(aq) + SO42–(aq) BaSO4(s)

(ii) the neutralisation reaction.


H+(aq) + OH–(aq) H2O(l)

56 Worksheet 7 © 2013 Marshall Cavendish International (Singapore) Private Limited

(S)CMWB_07.indd 56 8/14/12 4:38 PM


(b) State the ions that were present when the following volumes of sulfuric acid were added
to the barium hydroxide solution:
(i) 0 cm3

Ba2+ and OH– ions

(ii) 8 cm3
H+ and SO42– ions

(c) (i) Mark, on the graph, the end-point of the neutralisation reaction.
(ii) Suggest why the current decreased initially and increased after 5 cm3 of the acid was
added.
The current is directly proportional to the concentration of ions in the solution. As sulfuric acid

was added to barium hydroxide, barium sulfate precipitate was formed, and the concentration

of ions decreased. When exactly 5 cm3 of sulfuric acid was added, neutralisation was complete.

The amount of barium sulfate precipitate formed was at the maximum and the concentration of

ions was at the minimum. When excess acid was added, the concentration of ions increased

causing the current to increase.

(iii) Calculate the concentration, in mol/dm3, of barium hydroxide solution used.


5.0 cm3 of H2SO4 was required for complete neutralisation.

Number of moles of H2SO4 = 1.0 × 5.0


1000
= 0.005 mol
From the equation, 1 mol of Ba(OH)2 reacts with 1 mol of H2SO4.
Number of moles of Ba(OH)2 = 0.005 mol

Concentration of Ba(OH)2 in mol/dm3 = number of moles


3
volume in dm
= 0.005 ÷ 50
1000
= 0.1 mol/dm3

(d) Suggest why the iron stirrer was covered in plastic.

To prevent it from reacting with the acid

© 2013 Marshall Cavendish International (Singapore) Private Limited Worksheet 7 57

(S)CMWB_07.indd 57 8/14/12 4:38 PM


WORK ON IT!
In the table below, circle the question(s) that you have answered incorrectly. Revisit the
relevant section(s) in the textbook to strengthen your understanding of the key concept(s).
Question number(s) Textbook
Learning outcomes Multiple- Free- section(s)
Structured to revise
choice response
(a) Define acids as substances that produce
– – 2(b)(ii) 11.1
hydrogen ions in aqueous solution.
(b) Describe the properties of acids and
their reactions with metals, bases and 1 1(a) 2(d) 11.1
carbonates.
(c) State the uses of sulfuric acid. 2 – – 11.1
(d) Define alkalis as substances that
produce hydroxide ions in aqueous 7 – 2(b)(i) 11.2
solution.
(e) Describe what is meant by
2(a)(ii),
neutralisation and write the ionic – 2(b) 11.2
2(c)(i), 2(c)(ii)
equation for the neutralisation reaction.
(f) Describe the properties of alkalis
and their reactions with acids and 3 2 – 11.2
ammonium salts.
(g) Distinguish between strong and
weak acids in terms of their extent of 4 – 1(b)(iii) 11.3
ionisation.
(h) Describe the effects of acids and
– – 1(b)(ii) 11.4
alkalis on Universal Indicator.
(i) Describe the use of Universal Indicator
1(a), 1(b)(i),
and the pH scale to test hydrogen ion 8, 9 – 11.4
1(b)(iii)
concentration and relative acidity.
(j) Describe the reasons for controlling
the pH of soil and the methods used to 5 – – 11.4
reduce excess acidity in soil.
(k) Classify oxides as acidic, basic,
amphoteric or neutral based on their 6 – – 11.5
metallic or non-metallic properties.
(l) Write balanced chemical equations
– 2(b), 2(c) – 8.1
with state symbols.
(m) Write ionic equations with state
– – 2(a) 8.2
symbols.
(n) Determine the empirical and molecular
formulae of a compound from given – 1(b) – 9.5
data.
(o) Perform stoichiometric calculations
– – 2(c)(iii) 10.4
involving concentrations of solutions.

58 Worksheet 7 © 2013 Marshall Cavendish International (Singapore) Private Limited

(S)CMWB_07.indd 58 8/14/12 4:38 PM


Name: ( )

Worksheet 8
Class: Date:

Salts

for Chapter 12
ANSWER IT RIGHT!
Go through the worked example. Use the tips to guide you in understanding and answering
the questions.

Describe the preparation of a pure, dry sample of barium sulfate from barium carbonate.

The term ‘describe’ means that you will need to state the main points of the topic, which may be an experiment
or a phenomenon. You will have to provide a diagram if the question asks for it.

Thought Process
There are three methods of preparing salts:
• Acid + insoluble metal carbonate or base
• Acid + alkali (titration)
• Precipitation

When preparing a salt, there are two questions you must consider:
• Is the salt to be prepared soluble in water?
• Are the starting materials soluble in water?

Barium sulfate is insoluble in water. Thus, you need to use the precipitation method to prepare the salt.
Since the starting material, barium carbonate, is also insoluble in water, the first step is to convert the
starting material to a soluble salt.

Answer
Step 1: Add excess barium carbonate to dilute nitric acid to produce barium nitrate solution.
BaCO3(s) + 2HNO3(aq) Ba(NO3)2(aq) + CO2(g) + H2O(l)
Step 2: Filter to remove unreacted barium carbonate. Collect the filtrate (barium nitrate solution).
Step 3: Add dilute sulfuric acid (or any soluble sulfate) to the filtrate. A precipitate of barium sulfate
is formed.
Ba(NO3)2(aq) + H2SO4(aq) BaSO4(s) + 2HNO3(aq)
Step 4: Filter to collect the precipitate. Wash the precipitate with a small amount of distilled water to
remove impurities and any traces of nitric acid.
Step 5: Dry the precipitate between a few sheets of filter paper.

© 2013 Marshall Cavendish International (Singapore) Private Limited Worksheet 8 59

(S)CMWB_08.indd 59 8/14/12 4:51 PM


TEST IT!

Section A: Multiple-Choice Questions


Choose the correct answer and write its letter in the brackets provided.

1. Which statement is not true for all salts?


A They contain a metal ion or an ammonium ion.
B They contain a negative ion that comes from an acid.
C They contain water of crystallisation.
D Their names end with ‘–ate’ if they contain oxygen. ( C )

2. Which salt is best prepared by titration?


A Barium sulfate
B Calcium carbonate
C Lead(II) nitrate
D Potassium chloride ( D )

3. Lead(II) chloride is insoluble in water. It can be prepared by reacting dilute hydrochloric acid with
aqueous lead(II) nitrate. Which step is involved in the preparation of lead(II) chloride?
A Titrate dilute hydrochloric acid against aqueous lead(II) nitrate.
B Evaporate the filtrate until crystals of lead(II) chloride form.
C Collect the precipitate of lead(II) chloride by filtration.
D Add dilute hydrochloric acid until no more gas is evolved. ( C )

4. Dilute nitric acid was added to a colourless solution S. The acidified solution of S reacted with
silver nitrate to give a white precipitate. What is S most likely to be?
A Copper(II) carbonate
B Iron(II) sulfate
C Lead(II) nitrate
D Potassium chloride ( D )

5. As shown below, a student performed a number of tests on an aqueous solution of lead(II) nitrate.
Which test must be repeated because the student’s observation was wrong?

Test Observation
A Add aqueous sodium hydroxide A gas was given off which turned moist
followed by aluminium foil and boil. red litmus paper blue.
B Add dilute hydrochloric acid and A yellow precipitate was formed.
aqueous barium chloride.
C Add aqueous sodium hydroxide. A white precipitate was formed.
D Add aqueous zinc sulfate. A white precipitate was formed. ( B )

60 Worksheet 8 © 2013 Marshall Cavendish International (Singapore) Private Limited

(S)CMWB_08.indd 60 8/14/12 4:51 PM


6. Solution X contains two anions. The scheme below shows some reactions of solution X.

+ dilute HNO3(aq) + BaCl2(aq)


solution X colourless solution white precipitate

+ AgNO3(aq)

white precipitate

What anions are present in X?


A CO32– and SO42–
B CO32– and NO3–
C Cl – and SO42–
D Cl – and NO3– ( C )

7. The reaction scheme below shows some reactions of compound Y.

+ NaOH(aq) white precipitate


colourless solution which dissolves in
+ excess HCl(aq) excess NaOH(aq)
compound Y
carbon dioxide gas

What is compound Y?
A Ammonium carbonate B Calcium carbonate
C Iron(II) carbonate D Zinc carbonate ( D )

Section B: Structured Questions


Answer the following questions in the spaces provided.

1. The salts of magnesium have many uses. For example, magnesium nitrate is a common fertiliser
and magnesium chloride is used as a dietary supplement for magnesium.
(a) Magnesium nitrate can be prepared by the procedure described below.

1. Pour 25 cm3 of reagent X into a beaker.


2. Add solid Y in small portions to X with stirring until no more effervescence is observed.
3. Heat the mixture and then filter to remove excess Y.
4. Transfer the filtrate to an evaporating dish and heat the filtrate until it is saturated.
5. Allow the hot saturated solution to cool and crystallise.
6. Filter the mixture and collect the magnesium nitrate crystals.
7. Dry the magnesium nitrate crystals between a few sheets of filter paper.

(i) Identify X and Y.

X: Dilute nitric acid

Y: Magnesium (or magnesium carbonate)

© 2013 Marshall Cavendish International (Singapore) Private Limited Worksheet 8 61

(S)CMWB_08.indd 61 8/14/12 4:51 PM


(ii) A gas is produced in the preparation of magnesium nitrate described in (a). Write the
formula of this gas.
H2 (or CO2)

(iii) Describe a test that can be used to identify the gas produced.

H2: Insert a lighted splint. The splint will be extinguished with a ‘pop’ sound.

(Or CO2: Bubble the gas through limewater. A white precipitate will be formed.)

(b) Describe a chemical test that can be used to distinguish a solution of magnesium nitrate
from a solution of magnesium chloride.

Add dilute nitric acid followed by silver nitrate to each solution. The solution of magnesium chloride

will form a white precipitate upon reaction with acidified silver nitrate, while magnesium nitrate will not

have any observable change.

*2. Calcium carbonate is insoluble in water. Calcium sulfate and calcium hydroxide are both sparingly
soluble in water. Four students were each asked to suggest a method for making calcium sulfate
from calcium carbonate:
Method 1: Add dilute nitric acid to calcium carbonate, then add dilute sulfuric acid to the calcium
nitrate solution formed.
Method 2: Add dilute sulfuric acid to calcium carbonate.
Method 3: Heat calcium carbonate strongly to give calcium oxide, then react the calcium oxide with
dilute sulfuric acid.
Method 4: Heat calcium carbonate strongly to give calcium oxide. React the calcium oxide with
water to form calcium hydroxide solution and then add dilute sulfuric acid.

(a) Which method is the most suitable for preparing calcium sulfate? Write the equation(s) for
any reaction(s) involved. Include state symbols.
Method 1. The reactions involved are:

CaCO3(s) + 2HNO3(aq) Ca(NO3)2(aq) + H2O(l) + CO2(g)

Ca(NO3)2(aq) + H2SO4(aq) CaSO4(s) + 2HNO3(aq)

(b) Explain why the other three methods are less suitable.
In methods 2 and 3, dilute sulfuric acid and calcium carbonate or calcium oxide react to form an

insoluble layer of calcium sulfate, which prevents further reaction.

In methods 3 and 4, calcium carbonate has to be heated to a very high temperature before it

decomposes. This makes the process difficult and time-consuming.

62 Worksheet 8 © 2013 Marshall Cavendish International (Singapore) Private Limited

(S)CMWB_08.indd 62 8/14/12 4:51 PM


*3. Study the flow chart below.

pungent gas which


turns moist red
on heating litmus paper blue
+ aqueous sodium hydroxide
green solution W

green precipitate
+ dilute nitric acid

+ aqueous + aqueous
silver nitrate barium nitrate

white precipitate white precipitate

Identify the two cations and two anions present in green solution W. Quote evidence from the flow
chart to support your answers.
2+
(a) (i) First cation: Fe

Evidence: Upon addition of aqueous sodium hydroxide, a green precipitate is formed. This
suggests the presence of Fe2+ ions.

+
(ii) Second cation: NH4

Evidence: Upon addition of aqueous sodium hydroxide and on heating, ammonia gas is
given off. This suggests the presence of NH4+ ions.

2–
(b) (i) First anion: SO4

Evidence: A white precipitate is formed with the addition of acidified barium nitrate. This
suggests the presence of SO42– ions.


(ii) Second anion: Cl

Evidence: A white precipitate is formed with the addition of acidified silver nitrate. This
suggests the presence of Cl – ions.

© 2013 Marshall Cavendish International (Singapore) Private Limited Worksheet 8 63

(S)CMWB_08.indd 63 8/14/12 4:51 PM


Section C: Free-Response Questions
Answer the following questions.

1. One of the steps in the preparation of magnesium sulfate crystals is to react excess magnesium
oxide with dilute sulfuric acid.
(a) (i) Why is magnesium oxide added in excess?
To make sure all the sulfuric acid has reacted

(ii) The apparatus shown below was used to filter the mixture. Identify residue A and
solution B.

A: Unreacted magnesium oxide

B: Magnesium sulfate solution

(b) After removing residue A, solution B was heated until it became saturated, and then cooled.
The crystals formed were collected and dried between a few sheets of filter paper.

How would you test if the solution is saturated?

Put a clean, dry, and cold glass rod into the solution and then remove it. If crystals form on the glass

rod, the solution is saturated.

(c) Suggest, with a reason, why the crystals should be dried with filter paper and not by heating.
The crystals contain water of crystallisation. If they are heated, the water of crystallisation will be

removed from the salt.

64 Worksheet 8 © 2013 Marshall Cavendish International (Singapore) Private Limited

(S)CMWB_08.indd 64 8/14/12 4:51 PM


(d) It was found that when 25.0 cm3 of 1.0 mol/dm3 sulfuric acid was used, the mass of
magnesium sulfate obtained was 1.23 g.
(i) Write the equation for the reaction between magnesium oxide and sulfuric acid.
MgO(s) + H2SO4(aq) MgSO4(aq) + H2O(l)

(ii) Calculate the percentage yield of magnesium sulfate obtained.

Number of moles of H2SO4 = 25 × 1.0


1000
= 0.025 mol
From the reaction, 1 mol of H2SO4 produces 1 mol of MgSO4.
Number of moles of magnesium sulfate obtained = 0.025 mol
Mr of MgSO4 = 24 + 32 + (4 × 16)
= 120
Theoretical yield of MgSO4 = number of moles × molar mass
= 0.025 × 120
= 3.00 g
Percentage yield of MgSO4 = actual yield in g × 100%
theoretical yield in g
= 1.23 × 100%
3.00
= 41%

2. The solubilities of some salts are shown in the table below.

Salt Chemical formula Solubility in water

magnesium chloride MgCl2 soluble

magnesium fluoride MgF2 insoluble

magnesium phosphate Mg3(PO4)2 insoluble

silver chloride AgCl insoluble

silver nitrate AgNO3 soluble

silver sulfate Ag2SO4 insoluble

(a) State the method for preparing silver chloride. Explain your answer.
Precipitation can be used to prepare silver chloride as it is an insoluble salt.

(b) From the table above, identify two salts that can be used to prepare silver chloride.
The salts are magnesium chloride and silver nitrate.

© 2013 Marshall Cavendish International (Singapore) Private Limited Worksheet 8 65

(S)CMWB_08.indd 65 8/14/12 4:51 PM


(c) Describe the preparation of silver chloride from the two salts named in (b).

The magnesium chloride solution and silver nitrate solution are mixed in a beaker. The two solutions

react to give a precipitate, silver chloride. The mixture is then filtered and the precipitate (silver

chloride) is collected. The precipitate is washed with a little distilled water and dried on a piece of

filter paper.

(d) Write a balanced equation, including state symbols, for the reaction in (c).
MgCl2(aq) + 2AgNO3(aq) Mg(NO3)2(aq) + 2AgCl(s)

WORK ON IT!
In the table below, circle the question(s) that you have answered incorrectly. Revisit the
relevant section(s) in the textbook to strengthen your understanding of the key concept(s).

Question number(s) Textbook


Learning outcomes Multiple- Free- section(s)
Structured to revise
choice response

(a) Define a salt. 1 – – 12.1

(b) Describe the general rules for the


– – 2(a), 2(b) 12.1
solubility of common salts in water.

(c) Suggest a suitable method and


2 1(a)(i), 2 2(a), 2(b) 12.2
starting materials for preparing a salt.

(d) Describe the methods of separation 1(a), 1(b),


3 – 12.2
and purification in preparing salts. 1(c), 2(c)

(e) Describe the tests to identify cations. 5, 7 3(a) – 12.3

(f) Describe the tests to identify anions. 4, 6, 7 1(b), 3(b) – 12.3

(g) Describe the tests to identify gases. – 1(a)(iii) – 12.3

(h) Write balanced chemical equations


– 2(a) 1(d)(i), 2(d) 8.1
with state symbols.

(i) Calculate the percentage yield and


– – 1(d)(ii) 10.5
percentage purity of a substance.

(j) Describe the properties of acids and


their reactions with metals, bases and – 1(a)(ii) – 11.1
carbonates.

66 Worksheet 8 © 2013 Marshall Cavendish International (Singapore) Private Limited

(S)CMWB_08.indd 66 8/14/12 4:51 PM


Name: ( )

Worksheet 9
Class: Date:

Oxidation and Reduction

for Chapter 13
ANSWER IT RIGHT!
Go through the worked example. Use the tips to guide you in understanding and answering
the questions.

(a) The reaction between ammonium perchlorate, NH4ClO4, and aluminium metal is used to propel
space shuttles. The equation for the reaction is shown below.
6NH4ClO4(s) + 10Al(s) 4Al2O3(s) + 2AlCl3(s) + 12H2O(l) + 3N2(g)
(i) State the oxidation numbers of nitrogen and chlorine in ammonium perchlorate.
(ii) Identify the oxidising agent in the above reaction. Give a reason for your answer.

Oxidation state is also known as oxidation number.

Thought Process
(i) Ammonium perchlorate consists of the polyatomic ions NH4+ and ClO4–. The sum of the
oxidation states or numbers of the atoms in a polyatomic ion is equal to the charge on the ion.
To calculate the oxidation number of nitrogen, look at the NH4+ ion. NH4+ carries a +1 charge
and the oxidation number of hydrogen is +1.
To calculate the oxidation state of chlorine, look at the ClO4– ion. ClO4– carries a –1 charge
and the oxidation number of oxygen is –2.
(ii) An oxidising agent causes another substance to be oxidised and is itself reduced.

Answer
(i) Let the oxidation number of nitrogen be x.
Since NH4+ has a +1 charge and the oxidation number of hydrogen is +1,
x + 4(+1) = +1
x = –3
Thus, the oxidation number of nitrogen in NH4ClO4 is –3.
Let the oxidation number of chlorine be y.
Since ClO4– has a –1 charge and the oxidation number of oxygen is –2,
y + 4(–2) = –1
y = +7
Thus, the oxidation number of chlorine in NH4ClO4 is +7.

• When writing the oxidation state or number, the sign ‘+’ or ‘–’ should be included. For example, the
oxidation state (number) of hydrogen is +1 and not 1.
• Note that oxidation state is not written in the same way as the charge on an ion. For example, the oxide
ion is written as ‘O2–’ where ‘2–’ indicates the charge on the ion, but the oxidation state of oxygen is
written as ‘–2’.

© 2013 Marshall Cavendish International (Singapore) Private Limited Worksheet 9 67

(S)CMWB_09.indd 67 8/14/12 5:25 PM


(ii) Ammonium perchlorate is the oxidising agent. It oxidises aluminium to aluminium oxide and
aluminium chloride. The oxidation number of aluminium increases from 0 in Al to +3 in Al2O3
and AlCl3.

(b) Air bags have saved thousands of lives since their introduction in the early 1980s. When a car
fitted with air bags brakes suddenly and rapidly, sodium azide, NaN3, in the air bag decomposes
to produce nitrogen gas. The nitrogen gas causes the air bag to inflate. The equation for the
decomposition of sodium azide is shown below.
2NaN3(s) 2Na(s) + 3N2(g)
(i) State, with a reason, what has been reduced in this reaction.

• ‘State’ requires a concise answer. There is no need to give supporting reasons unless the question
asks for it.
• In this case, the question is accompanied by the phrase ‘with a reason’. This means that an explanation
is required.

(ii) Calculate the volume, in cm3, of nitrogen gas produced when 6.5 g of sodium azide
decomposes at r.t.p.

Thought Process
(i) A substance is reduced if its oxidation state decreases after the reaction.

Answer
(i) Sodium in sodium azide, NaN3, has been reduced because there is a decrease in the oxidation
number from +1 to 0.
(ii) Mr of NaN3 = 23 + 3(14) = 65
mass in g
Number of moles of NaN3 = = 6.5 = 0.1 mol
molar mass in g/mol 65
From the equation, 3 mol of N2 is produced from 2 mol of NaN3.
Number of moles of N2 produced = 3 × 0.1 = 0.15 mol
2
Volume of nitrogen produced = number of moles × 24 dm3
= 0.15 × 24
= 3.6 dm3
= 3600 cm3

TEST IT!

Section A: Multiple-Choice Questions


Choose the correct answer and write its letter in the brackets provided.

1. The equation below shows the reaction that occurs between iron(III) chloride and hydrogen sulfide.
2FeCl3(aq) + H2S(g) 2FeCl2(aq) + 2HCl(aq) + S(s)
Which element is oxidised in the reaction?
A Iron B Chlorine
C Hydrogen D Sulfur ( D )

68 Worksheet 9 © 2013 Marshall Cavendish International (Singapore) Private Limited

(S)CMWB_09.indd 68 8/14/12 5:25 PM


2. The following equation shows the reaction that takes place on photographic film:
2AgBr(s) 2Ag(s) + Br2(g)
What can be deduced from the equation?
A The bromide ions are reduced to bromine molecules.
B Light is needed for the reaction to take place.
C The silver ions are oxidised to silver atoms.
D Electrons are transferred from the bromide ions to the silver ions. ( D )

3. In which pair of substances does the named element have the same oxidation state?
A Copper in Cu2O and Cu
B Iron in FeO and Fe2O3
C Manganese in MnO2 and KMnO4
D Sulfur in SO3 and H2SO4 ( D )

4. In which reaction does the oxidation state of chlorine increase by one?


A 2KClO3(s) 2KCl(s) + 3O2(g)
B Cl2(g) + 2NaOH(aq) NaCl(aq) + NaClO(aq) + H2O(l)
C Cl2(g) + H2(g) 2HCl(g)
D NaCl(aq) + AgNO3(aq) AgCl(s) + NaNO3(aq) ( B )

5. Which reaction is a redox reaction?


A Barium chloride + potassium sulfate barium sulfate + potassium chloride
B Potassium hydroxide + hydrochloric acid potassium chloride + water
C Potassium iodide + chlorine potassium chloride + iodine
D Silver nitrate + potassium chloride silver chloride + potassium nitrate ( C )

6. What colour changes occur when sulfur dioxide is passed through aqueous potassium iodide and
acidified potassium manganate(VII) separately?
Aqueous potassium iodide Acidified potassium manganate(VII)
A colourless to brown no change
B colourless to brown purple to colourless
C no change no change
D no change purple to colourless ( D )

7. Why does the colour of aqueous potassium iodide change to brown when chlorine gas is bubbled
through it?
A A compound is formed between chlorine and potassium iodide.
B Chlorine oxidises iodide ions to iodine.
C Iodine is reduced by chlorine.
D Potassium chloride, which is brown, is formed. ( B )

8. Hydrogen peroxide reacts with acidified potassium iodide to form iodine as one of the products. It
also turns acidified potassium manganate(VII) from purple to colourless.
Which statement is true about hydrogen peroxide in both reactions?
A Hydrogen peroxide acts as both an oxidising agent and a reducing agent.
B Hydrogen peroxide acts only as a reducing agent.
C Hydrogen peroxide acts only as an oxidising agent.
D Hydrogen peroxide is neither an oxidising agent nor a reducing agent. ( A )

© 2013 Marshall Cavendish International (Singapore) Private Limited Worksheet 9 69

(S)CMWB_09.indd 69 8/14/12 5:25 PM


9. Which reactions are oxidation reactions?

C2H5OH
2
1 3
CH3CHO CH3CO2H CH3CO2Na

A 1 and 2 B 1 and 3
C 2 and 3 D 1, 2 and 3 ( A )

Section B: Structured Questions


Answer the following questions in the spaces provided.

1. Gold (Au) can form ions with the oxidation states of +1 and +3.
(a) Write down the chemical formulae of two chlorides that gold can form.

AuCl and AuCl3

(b) (i) When copper is added to gold(III) chloride solution, gold in the chloride solution is
displaced by copper. Write the chemical and ionic equations for the reaction between
copper and gold(III) chloride.

Chemical equation: 3Cu(s) + 2AuCl3(aq) 3CuCl2(aq) + 2Au(s)

3+
Ionic equation: 3Cu(s) + 2Au (aq) 3Cu2+(aq) + 2Au(s)

(ii) State, with a reason, which substance has been oxidised in this reaction.

Copper has been oxidised. It has lost two electrons to form Cu2+. The oxidation state of copper

increased from 0 to +2.

(c) (i) When a solution containing gold(I) ions is gently heated, solid gold is precipitated
together with gold(III) ions. Write an ionic equation for this reaction.

3Au+(aq) 2Au(s) + Au3+(aq)

(ii) State whether the gold(I) ions have been oxidised or reduced in this reaction.

The Au+ ions have been both oxidised and reduced.

(d) Gold(III) ions are powerful oxidising agents. How do you test for the presence of an oxidising
agent?

Add aqueous potassium iodide to the solution to be tested. The solution changes from colourless to

brown if an oxidising agent is present.

70 Worksheet 9 © 2013 Marshall Cavendish International (Singapore) Private Limited

(S)CMWB_09.indd 70 8/14/12 5:25 PM


2. (a) The diagram below shows the oxidation states of sulfur as an element and in the compounds
sulfuric acid, sulfur trioxide and sulfur dioxide. Identify the substances A, B, C and D.

Oxidation state

+6
+5
+4
+3
+2
+1

0 Substance
A B C D

A: Sulfur B: Sulfur dioxide

C: Sulfur trioxide/sulfuric acid D: Sulfuric acid/sulfur trioxide

(b) The equations below represent the stages in the manufacture of nitric acid from nitrogen.
N2(g) + 3H2(g) 2NH3(g)
4NH3(g) + 5O2(g) 4NO(g) + 6H2O(l)
2NO(g) + O2(g) 2NO2(g)
4NO2(g) + 2H2O(l) + O2(g) 4HNO3(aq)
(i) Sketch a diagram, similar to the one in (a), to show the oxidation states of nitrogen in
the substances N2, NH3, NO, NO2 and HNO3.
Oxidation state
+6
+5

+4
+3
+2

+1
0 Substance
N2 NH3 NO NO2 HNO3
–1
–2

–3

–4

(ii) From your diagram in (i), state if nitrogen has been oxidised or reduced in each stage of
the manufacturing process.

( reduced ) ( oxidised ) ( oxidised ) ( oxidised )


N2 NH3 NO NO2 HNO3

© 2013 Marshall Cavendish International (Singapore) Private Limited Worksheet 9 71

(S)CMWB_09.indd 71 3/10/16 10:05 AM


3. Study each pair of equations and determine whether each statement is true or false. Explain your
answer.
(a) C(s) + O2(g) CO2(g)
2H2S(g) + O2(g) 2H2O(l) + 2S(s)
Oxygen acts as an oxidising agent in both reactions.
True. Oxygen causes C and H2S to be oxidised to CO2 and S respectively.

(b) 2FeCl2(aq) + Cl2(g) 2FeCl3(aq)


2NaCl(aq) + F2(g) 2NaF(aq) + Cl2(g)
The oxidation state of chlorine has decreased to –1 in both reactions.

False. In the second reaction, the oxidation state of chlorine has increased from –1 in NaCl to 0

in Cl2.

(c) N2(g) + 3H2(g) 2NH3(g)


2Na(s) + H2(g) 2NaH(s)
Hydrogen acts as a reducing agent in both reactions.
False. Hydrogen acts as a reducing agent in the first reaction. It causes nitrogen to be reduced to

ammonia due to the gain of hydrogen. On the other hand, hydrogen acts as an oxidising agent in the

second reaction. It causes sodium to be oxidised to sodium ions due to the loss of electrons.

Section C: Free-Response Questions


Answer the following questions.

1. Ethanoic acid, CH3COOH, is the main compound found in vinegar. It has a sour taste and a
distinctive pungent smell. Ethanoic acid undergoes many different reactions. When ethanoic acid
reacts with a reducing agent, lithium aluminium hydride (LiAlH4), ethanol is formed. The chemical
equation for this reaction is shown below.
LiAlH4
CH3COOH(aq) CH3CH2OH(aq) Reaction I

When ethanoic acid reacts with magnesium metal, the products obtained are a salt (magnesium
ethanoate) and hydrogen gas. The chemical equation for this reaction is shown below.
2CH3COOH(aq) + Mg(s) (CH3COO)2Mg(aq) + H2(g) Reaction II

When ethanoic acid reacts with sodium hydroxide, the products are a salt (sodium ethanoate) and
water. The chemical equation for this reaction is shown below.
CH3COOH(aq) + NaOH(aq) CH3COONa(aq) + H2O(l) Reaction III

72 Worksheet 9 © 2013 Marshall Cavendish International (Singapore) Private Limited

(S)CMWB_09.indd 72 8/14/12 5:25 PM


(a) Explain, in terms of gain or loss of oxygen and hydrogen, why the conversion of ethanoic acid
to ethanol is a reduction reaction.

It is a reduction reaction because ethanoic acid, CH3COOH, has lost oxygen and gained hydrogen to

form ethanol, CH3CH2OH.

(b) Identify the elements that are oxidised and reduced in Reaction II. State the changes in the
oxidation states of these elements during the reaction.
Hydrogen is reduced. The oxidation state of H decreases from +1 in CH3COOH to 0 in H2.

Magnesium is oxidised. The oxidation state of Mg increases from 0 in Mg to +2 in (CH3COO)2Mg.

(c) Is Reaction III a redox reaction? Explain your answer.

No. None of the elements in the reaction are reduced or oxidised.

(d) When ethanoic acid reacts with potassium carbonate, the products obtained are a salt
(potassium ethanoate), carbon dioxide and water.
(i) Write a balanced chemical equation for this reaction. Include state symbols.
2CH3COOH(aq) + K2CO3(aq) 2CH3COOK(aq) + H2O(l) + CO2(g)

(ii) State the oxidation state of carbon in potassium carbonate and in carbon dioxide.
The oxidation state of carbon is +4 in both K2CO3 and CO2.

*2. (a) Explain, with an example, why there cannot be an oxidation reaction without a corresponding
reduction reaction.

Take the reaction between copper and silver nitrate as an example:

Cu(s) + 2AgNO3(aq) Cu(NO3)2(aq) + 2Ag(s). Copper has been oxidised as it has lost two electrons

to form copper(II) ions. The electrons lost must be gained by another substance. The silver ions in

silver nitrate have gained the electrons to form silver atoms. The silver ions have been reduced. This

example shows that oxidation and reduction always take place together.

© 2013 Marshall Cavendish International (Singapore) Private Limited Worksheet 9 73

(S)CMWB_09.indd 73 8/14/12 5:25 PM


(b) Explain why solutions of strong reducing agents, such as iron(II) chloride, have a short shelf
life when exposed to the air. Write an equation to support your explanation.

Since iron(II) chloride is a strong reducing agent, it is easily oxidised. When iron(II) chloride is exposed

to the air, it is oxidised by oxygen in the air. In the reaction, iron(II) ions lose electrons to form iron(III)

ions. The equation for the reaction is Fe2+ Fe3+ + e–.

WORK ON IT!
In the table below, circle the question(s) that you have answered incorrectly. Revisit the
relevant section(s) in the textbook to strengthen your understanding of the key concept(s).

Question number(s) Textbook


Learning outcomes Multiple- Free- section(s)
Structured to revise
choice response
(a) Define
• oxidation as the gain of oxygen; 9 3(a) 1(a) 13.1
• reduction as the loss of oxygen.
(b) Define redox reactions. – – 2(a) 13.1
(c) Define
• oxidation as the loss of hydrogen; 1, 9 3(c) 1(a) 13.2
• reduction as the gain of hydrogen.
(d) Define
1(b)(ii),
• oxidation as the loss of electrons; 2 2(b) 13.3
1(c)(ii), 3(c)
• reduction as the gain of electrons.
(e) Define
• oxidation as an increase in oxidation 1(b)(ii),
state; 1, 3, 4 1(c)(ii), 2, 1(b), 1(d)(ii) 13.4
• reduction as a decrease in oxidation 3(b)
state.
(f) Identify redox reactions. 5 – 1(c) 13.5
(g) Test for oxidising agents and reducing
1(d), 3(a),
agents using aqueous potassium iodide 6, 7, 8 – 13.6
3(c)
and acidified potassium manganate(VII).
(h) Deduce the formula of an ionic
compound from the charges on the – 1(a) – 6.4
ions and vice versa.
(i) Write balanced chemical equations
– 1(b)(i) 1(d)(i) 8.1
with state symbols.
(j) Write ionic equations with state
– 1(b)(i), 1(c)(i) – 8.2
symbols.

74 Worksheet 9 © 2013 Marshall Cavendish International (Singapore) Private Limited

(S)CMWB_09.indd 74 8/14/12 5:25 PM


Name: ( )

Worksheet 10
Class: Date:

Metals

for Chapter 14
ANSWER IT RIGHT!
Go through the worked example. Use the tips to guide you in understanding and answering
the questions.

(a) Brass is an alloy of copper and zinc. Explain why brass is harder than pure copper.

Thought Process
The physical properties of a substance depend on its structure. The difference in hardness
between brass (an alloy) and copper (a metal) is due to their different atomic arrangements.

Answer
The atoms in pure copper are packed regularly in layers. When a force is applied, the layers of
atoms can slide over one another easily. Thus, pure copper is soft. In brass, the copper and zinc
atoms are of different sizes. This breaks up the regular arrangement of atoms in pure copper,
which makes it harder for the layers to slide over one another when a force is applied. Hence,
brass is harder than pure copper.

• It is a common mistake to think that brass (an alloy) is harder and stronger than copper (a pure metal)
because copper forms strong bonds with zinc.
• The hardness of an alloy is due to the different sizes of the constituent atoms of the alloy.

(b) Explain the electrical conductivity of zinc in the solid state.

Thought Process
The electrical conductivity of a substance depends on its type of chemical bonding and its physical
state.

Answer
Zinc is a metal. Zinc atoms are held by metallic bonds, which consist of positive zinc ions
surrounded by a ‘sea of mobile electrons’. This means that the outer electrons are free to move
through the entire metal structure. Thus, zinc can conduct electricity in the solid state.

• It is too vague to write ‘metals conduct electricity because the electrons move through the entire metal
structure’.
• Note that metals conduct electricity due to the movement of their outer electrons only.

© 2013 Marshall Cavendish International (Singapore) Private Limited Worksheet 10 75

(S)CMWB_10.indd 75 8/14/12 5:27 PM


(c) Carbon steels are alloys of iron and carbon. How does the carbon content of steel affect the
properties of steel?

Thought Process
Whether a steel is malleable or brittle will depend on the proportion of carbon in the steel.

Answer
Mild steel has low carbon content. It is strong and malleable. High-carbon steel has high carbon
content. It is strong but brittle. Thus, the higher the content of carbon, the more brittle the steel.

This is a question on comparing the different physical properties of high-carbon steel and mild steel
(low-carbon steel).

TEST IT!

Section A: Multiple-Choice Questions


Choose the correct answer and write its letter in the brackets provided.

1. What is an alloy?
A A mixture of a metal with one or a few other elements
B A mixture of a non-metal with one or a few other elements
C A mixture that must contain carbon
D A mixture that must contain iron ( A )

2. Which diagram shows the structure of an alloy?


A B

C D

( B )

3. What are the advantages of recycling metals?


1 Conserves fossil fuels
2 Reduces air and water pollution
3 Always requires less time and money
A 1 and 2 B 1 and 3
C 2 and 3 D 1, 2 and 3 ( A )

76 Worksheet 10 © 2013 Marshall Cavendish International (Singapore) Private Limited

(S)CMWB_10.indd 76 8/14/12 5:27 PM


4. The positions of some metals in the reactivity series are shown below.
Most reactive Least reactive
iron > tin > lead > silver
In which beaker will there be no reaction?
A Sn B Pb C Sn D Fe

Pb(NO3)2(aq) SnCl2(aq) AgNO3(aq) SnCl2(aq) ( B )

5. Which statement about the production of iron from iron(III) oxide in the blast furnace is correct?
A Calcium carbonate (limestone) removes the basic impurities.
B Carbon is oxidised to carbon monoxide by carbon dioxide.
C Carbon monoxide oxidises iron(III) oxide to iron(II) oxide.
D Molten iron floats on the molten slag. ( B )

6. Which of the following uses is incorrect?


A Cast iron for woks
B Galvanised steel for cutlery
C Stainless steel for hospital equipment
D Steel for cans and containers ( B )

7. A metal, M, takes part in the reactions shown by the following chemical equations:
2M + O2 2MO
M + 2HCl MCl2 + H2
heat
MO + C no reaction
Which metal could M be?
A Iron
B Lead
C Magnesium
D Zinc ( C )

*8. Hydrogen was passed over 1.0 g of heated copper(II) oxide. What is the colour and mass of the
product obtained?
Colour of product Mass of product
A black more than 1.0 g
B black less than 1.0 g
C reddish-brown more than 1.0 g
D reddish-brown less than 1.0 g ( D )

© 2013 Marshall Cavendish International (Singapore) Private Limited Worksheet 10 77

(S)CMWB_10.indd 77 8/14/12 5:27 PM


Section B: Structured Questions
Answer the following questions in the spaces provided.

1. Metals were added in excess to aqueous solutions of different metal nitrates. The table below
records the observations obtained. A tick (✓) indicates a reaction. A cross (✗) indicates no reaction.

Aqueous metal
Copper(II) Magnesium Nickel(II)
nitrate Silver nitrate Zinc nitrate
nitrate nitrate nitrate
(colourless) (colourless)
(blue) (colourless) (green)
Metal
Copper ✗ ✗ ✓ ✗
Magnesium ✓ ✓ ✓ ✓
Nickel ✓ ✗ ✓ ✗
Silver ✗ ✗ ✗ ✗
Zinc ✓ ✗ ✓ ✓

(a) Place the metals in order of their reactivity, starting with the most reactive metal.
Magnesium, zinc, nickel, copper, silver

(b) Which metal and aqueous metal nitrate would react to give the following colour changes?
(i) Blue to green: Nickel and copper(II) nitrate

(ii) Colourless to blue: Copper and silver nitrate

(c) Apart from colour changes, state two other observations that would indicate that a chemical
reaction has taken place.
A precipitate is observed on the surface of the metal added. The reaction mixture becomes warmer

(due to the increase in temperature during the reaction).

2. (a) What are the conditions that can cause iron to rust?
The presence of air (oxygen) and water

(b) Explain why rusting is an example of oxidation.


During rusting, iron gains oxygen./ The oxidation state of iron increases from 0 to +3.

78 Worksheet 10 © 2013 Marshall Cavendish International (Singapore) Private Limited

(S)CMWB_10.indd 78 8/14/12 5:27 PM


(c) (i) Car bodies can be protected from rusting by painting and sacrificial protection. Explain
how the two methods work.
Painting and sacrificial protection coat the iron of the car body with a layer of paint and a more

reactive metal (e.g. zinc) respectively. These layers protect the iron from being exposed to the

air, thus preventing moisture and oxygen in the air from reacting with the iron. In sacrificial

protection, the more reactive metal corrodes instead of iron.

(ii) What will happen if the paint on a galvanised car body is scratched off? Explain your
answer.
The car body will not rust. Zinc is more reactive than iron, so the zinc that remains will corrode

instead of iron.

3. The following list shows some elements in order of reactivity.


Most reactive Least reactive
potassium > magnesium > carbon > iron > tin > gold
(a) Some reactions of the metals are summarised in the table below.

Metal Reaction with water or steam Reaction with dilute acids


potassium violent reaction with cold water explosive
magnesium violent reaction with steam fast
iron slow reaction with steam slow
tin slow reaction with steam slow
gold no reaction with water or steam no reaction

(i) From the table, deduce the position of hydrogen in the reactivity series.
Between tin and gold

(ii) Based on your answer in (a)(i), deduce a general rule for the reactivity of metals with
water and dilute acids.
Only metals that are more reactive than hydrogen will react with water and dilute acids.

(b) Why does gold occur mostly as pure gold in the Earth’s crust, while potassium only occurs
as potassium compounds?
Gold is very unreactive and therefore occurs naturally uncombined as a metal. Potassium is very

reactive. It reacts easily with substances such as water, air and acids in the surroundings to form

compounds.

© 2013 Marshall Cavendish International (Singapore) Private Limited Worksheet 10 79

(S)CMWB_10.indd 79 8/14/12 5:27 PM


(c) Based on the order of reactivity of the elements, state the method that is most likely used to
extract the following metals from their compounds.
(i) Magnesium from molten magnesium chloride: Electrolysis

(ii) Tin from tin(IV) oxide: Reduction by carbon

4. The table below gives the densities and melting points of substances used in the blast furnace, at
normal atmospheric pressure.

Substance Density/(g/cm3) Melting point/°C


calcium carbonate 2.71 decomposes at 850°C
calcium oxide 3.35 2600
calcium silicate 2.50 1530
carbon 2.25 4000
iron 7.80 1539
iron(III) oxide 5.24 1566
magnesium oxide 3.58 2900

The diagram below shows the temperatures in a blast furnace.

waste gases

700°C

1100°C
1600°C
hot air hot air

B
C

Some elements, including those used in the blast furnace, are shown below in decreasing order of
reactivity.
Ca > Mg > Fe > C > H > Ti

80 Worksheet 10 © 2013 Marshall Cavendish International (Singapore) Private Limited

(S)CMWB_10.indd 80 3/10/16 10:06 AM


(a) Name the substances that are fed into the furnace at A.

Calcium carbonate, carbon and iron(III) oxide

(b) This reaction takes place at the base of the furnace:


C(s) + O2(g) CO2(g)
How is the carbon dioxide formed immediately reduced to carbon monoxide?
Excess carbon (coke) is present in the furnace. This reduces the carbon dioxide to carbon monoxide:

CO2(g) + C(s) 2CO(g)

(c) Write the equation for the reduction of iron(III) oxide by carbon monoxide.
Fe2O3(s) + 3CO(g) 2Fe(l) + 3CO2(g)

(d) (i) Why is the temperature of the slag coming out of B much lower than 1530°C?
It contains impurities that lower its melting point.

(ii) Use the data from the table to explain why slag floats on top of molten iron.
Slag (calcium silicate) is less dense than molten iron.

(iii) Why is it useful for slag to float on top of the molten iron?
This protects the molten iron from oxidation by the hot air.

(e) (i) The furnace is lined with magnesium oxide bricks. Suggest one physical property and
one chemical property of magnesium oxide that make it a suitable material.

Physical property: Magnesium oxide has a high melting point (it is a refractory substance).

Chemical property: It does not react with any of the substances in the furnace.

(ii) Why might the furnace explode if damp substances were added at A?
Iron and carbon would reduce the water (steam) to hydrogen. Hydrogen gas and air form an

explosive mixture.

© 2013 Marshall Cavendish International (Singapore) Private Limited Worksheet 10 81

(S)CMWB_10.indd 81 3/10/16 10:08 AM


Section C: Free-Response Questions
Answer the following questions.

1. Aluminium is the most abundant metal in the Earth’s crust. It has a low density and is corrosion-
resistant.

In a laboratory, some experiments involving aluminium were set up.

aluminium aluminium aluminium aluminium


strip strip strip strip
rubbed with rubbed with
sandpaper sandpaper
aqueous aqueous
iron(II) calcium aqueous aqueous
nitrate nitrate iron(II) nitrate calcium
nitrate
Experiment 1 Experiment 2 Experiment 3 Experiment 4

Changes were only observed in Experiment 3.


(a) Describe the observations you would see in Experiment 3. Write a balanced chemical equation
for this reaction.

A grey deposit (iron) is formed on the aluminium strip. The pale green solution of iron(II) nitrate slowly

turns colourless.

2Al(s) + 3Fe(NO3)2(aq) Al2(NO3)3(aq) + 3Fe(s)

(b) Suggest why a reaction occurred in Experiment 3 but not in Experiment 1.


When aluminium is exposed to air, a thin surface layer of aluminium oxide is formed. This oxide

layer prevents the aluminium from reacting with other reagents. Hence, no reaction took place in

Experiment 1. In Experiment 3, this layer of aluminium oxide was removed by the sandpaper and the

aluminium came into contact with aqueous iron(II) nitrate. Aluminium displaced iron from the solution

because aluminium is more reactive than iron.

(c) Based on the experimental results, arrange aluminium, calcium and iron in increasing order
of reactivity.
Iron, aluminium, calcium

(d) The carbonates of calcium and iron decompose on heating. Based on your answer in (c),
deduce the thermal stability of aluminium carbonate. Explain your answer.
Aluminium carbonate decomposes into aluminium oxide and carbon dioxide upon heating. Its thermal

stability is similar to those of iron(II) carbonate and calcium carbonate, both of which decompose into

the metal oxide and carbon dioxide when heated.

82 Worksheet 10 © 2013 Marshall Cavendish International (Singapore) Private Limited

(S)CMWB_10.indd 82 8/14/12 5:27 PM


2. Steel is an alloy of iron with carbon and/or other metals. It is made from cast iron extracted from
the blast furnace. Steel has many uses and properties.
(a) With the aid of diagrams, explain the difference between steel and iron in terms of structure.

Structure of steel Structure of iron

In iron, atoms are packed regularly in layers. All the atoms are of the same size. In steel, the

arrangement of atoms is not regular due to the different atom sizes of the different elements.

(b) Explain how the difference in structures in (a) accounts for the difference in physical properties
of steel and iron.
The regular arrangement in iron makes it easy for the layers of atoms to slide over each other

when a force is applied. Thus, iron is soft. The different sizes of atoms in steel disrupt the regular

arrangement of atoms. The layers cannot slide over each other easily. Thus, steel is harder and

stronger than iron.

(c) Carbon steel is a main category of steel. There are two types of carbon steel —
high-carbon steel and mild steel. State how the compositions of high-carbon steel and mild
steel account for their properties and uses.
High-carbon steel has high carbon content, which makes it strong but brittle. It is used to manufacture

cutting and boring tools. Mild steel has low carbon content, so it is soft and malleable. It is used to

make car bodies and machinery.

(d) Recycling metals helps to conserve natural resources and has many advantages. Discuss
some problems faced in metal recycling.
Recycling metals can be extremely costly. It is sometimes more expensive than extracting the

metal from the earth. Recycling metals can pollute the environment. For example, air pollutants are

produced in the process of recycling lead–acid batteries. It takes time and effort for communities to

adopt recycling as a lifestyle. Thus, it is not immediately effective.

© 2013 Marshall Cavendish International (Singapore) Private Limited Worksheet 10 83

(S)CMWB_10.indd 83 8/14/12 5:27 PM


WORK ON IT!
Circle the question(s) that you have answered incorrectly. Then, revisit the relevant section(s) in the
textbook to strengthen your understanding of the key concept(s).

Question number(s) Textbook


Learning outcomes Multiple- Free- section(s)
Structured to revise
choice response
(a) Explain what an alloy is. 1 – – 14.1
(b) Identify the diagrams of structures of
metals and alloys. 2 – 2(a) 14.1

(c) Explain the difference in the properties of


alloys and their constituent metals. – – 2(b) 14.1

(d) Determine the order of reactivity of some


metals from the reactions of metals with 7 3(a) – 14.2
water, steam and dilute hydrochloric acid.
(e) Relate the reactivity of metals to their
tendency to form positive ions by
• the displacement of metals from their
salt solutions; 4 – 1(a), 1(b) 14.3
• the reaction of a metal with the oxide
of another metal.
(f) Determine the order of reactivity of
metals from the reduction of metal oxides 7, 8 – – 14.3
with carbon and hydrogen.
(g) Describe the action of heat on metal
carbonates and explain how their thermal – – 1(d) 14.3
stability is related to the reactivity series.
(h) Deduce the order of reactivity of metals
when given a set of experimental results. – 1, 3(a) 1(c) 14.3

(i) Relate the method of obtaining the metal


from its ore to its position in the reactivity – 3(b), 3(c) – 14.4
series.
(j) Describe and explain the reactions
involved in the extraction of iron in the 5 4 – 14.4
blast furnace.
(k) Identify and describe steel as an alloy. – – 2(a) 14.5
(l) Describe how the properties of steel vary
with the amount of carbon or other metals – – 2(c) 14.5
added to iron.
(m) State the uses of steel. 6 – 2(c) 14.5
(n) Describe the conditions necessary for
rusting to occur. – 2(a) – 14.6

(o) State how rusting can be prevented. – 2(c) – 14.6


(p) State why there is a need to recycle metals. 3 – – 14.7
(q) Discuss the issues related to recycling
metals. – – 2(d) 14.7

(r) Define oxidation as the gain of oxygen. – 2(b) – 13.1


(s) Define oxidation as an increase in
oxidation state. – 2(b) – 13.4

84 Worksheet 10 © 2013 Marshall Cavendish International (Singapore) Private Limited

(S)CMWB_10.indd 84 8/14/12 5:27 PM


Name: ( )

Worksheet 11
Class: Date:

Electrolysis

for Chapter 15
ANSWER IT RIGHT!
Go through the worked example. Use the tips to guide you in understanding and answering the
questions.

(a) The electrolysis of dilute sodium chloride solution was carried out with inert electrodes. The gases
produced were collected.
(i) Construct ionic half-equations for the reactions that take place at the electrodes.

• The term ‘construct’ requires you to make use of information given to write your answer.
• In this case, you will need to use the information ‘dilute sodium chloride solution’ to write the ionic
half-equations.

(ii) Draw a labelled diagram to show the apparatus used for this experiment.

Thought Process
(i) • The ions present in dilute sodium chloride solution are Na+, Cl –, H+ and OH–.
• At the anode, OH– ions are discharged by losing electrons to form oxygen gas and water.
• At the cathode, H+ ions are discharged by gaining electrons to form hydrogen gas.

Answer
(i) At the anode: 4OH–(aq) O2(g) + 2H2O(l) + 4e–
At the cathode: 2H (aq) + 2e–
+
H2(g)

• Take note of the type of solution that is being electrolysed — the products of electrolysis of dilute NaCl,
concentrated NaCl and molten NaCl are different.
• For example, if concentrated or molten NaCl are electrolysed, chlorine gas instead of oxygen gas will be
the product at the anode.

(ii)
oxygen gas hydrogen gas

carbon anode carbon cathode

dilute sodium
chloride solution

battery

© 2013 Marshall Cavendish International (Singapore) Private Limited Worksheet 11 85

(S)CMWB_11.indd 85 8/15/12 2:55 PM


(b) A metal object is to be electroplated with copper. Suggest what can be used as the anode, cathode
and electrolyte.

Thought process
• The metal object to be electroplated is made the cathode.
• The anode is the source of the plating metal.
• The electrolyte is an aqueous salt solution of the plating metal.

Answer
Cathode: metal object to be electroplated
Anode: pure copper
Electrolyte: copper(II) sulfate solution

TEST IT!

Section A: Multiple-Choice Questions


Choose the correct answer and write its letter in the brackets provided.

1. In which of the following experiments will the bulb not light up?
A B

sodium chloride dilute sulfuric


solution acid

C D

mercury molten sulfur

( D )

2. Which of the following molten electrolytes requires only one mole of electrons to produce one mole
of metal atoms during electrolysis?
A Calcium chloride B Iron(II) chloride
C Magnesium bromide D Potassium bromide ( D )

3. What changes take place when concentrated potassium chloride solution is electrolysed using
inert electrodes?
Product formed at the cathode Solution produced
A chlorine neutral
B hydrogen alkaline
C oxygen acidic
D potassium neutral ( B )

86 Worksheet 11 © 2013 Marshall Cavendish International (Singapore) Private Limited

(S)CMWB_11.indd 86 8/15/12 2:55 PM


4. Copper(II) sulfate solution is electrolysed using carbon electrodes. Which of the half-equations
below represents the reaction at the cathode?
A 2H+(aq) + 2e– H2(g)
B 4OH–(aq) 2H2O(l) + O2(g) + 4e–
C Cu2+(aq) + 2e– Cu(s)
D SO42–(aq) SO2(g) + O2(g) + 2e– ( C )

5. Cells A and B show the electrolysis of dilute hydrochloric acid and copper(II) chloride solution
respectively.

carbon copper

dilute copper(II)
hydrochloric chloride solution
acid
Cell A Cell B

Which of the statements below is true?


A Oxygen gas is given off at the carbon anode in cell A.
B Hydrogen gas is given off at the copper cathode in cell B.
C The OH– ions in cell A accept electrons from the anode.
D The size of the copper anode in cell B increases. ( A )

6. A spoon to be silver-plated is made the cathode in an electrolytic cell. Which of the following could
be used as the anode and the electrolyte?
Anode Electrolyte
A carbon dilute hydrochloric acid
B carbon silver nitrate solution
C silver dilute hydrochloric acid
D silver silver nitrate solution ( D )

*7. Consider the simple cell shown below. Which of the following changes will occur?

salt bridge
Mg (negative electrode) Cu (positive electrode)

Mg(NO3)2(aq) CuSO4(aq)

A Oxygen gas is produced at the copper electrode.


B The blue colour of the copper(II) sulfate solution deepens.
C Hydrogen gas is produced at the magnesium electrode.
D The copper electrode is coated with a layer of brown solid. ( D )

© 2013 Marshall Cavendish International (Singapore) Private Limited Worksheet 11 87

(S)CMWB_11.indd 87 8/15/12 2:55 PM


Section B: Structured Questions
Answer the following questions in the spaces provided.

1. (a) Explain what is meant by the term ‘electrolysis’.

Electrolysis is the process of using electricity to break down or decompose a compound (usually an

ionic compound in the molten state or in aqueous solution).

(b) The following solutions are electrolysed using carbon electrodes. Predict the products.

Products
Electrolyte
at the cathode at the anode
(i) copper(II) chloride solution copper solid oxygen gas

(ii) silver nitrate solution silver solid oxygen gas

(iii) potassium sulfate solution hydrogen gas oxygen gas

(iv) molten sodium chloride molten sodium chlorine gas

2. The diagrams below show two types of cells.

v
e– e– e– e–
>

>
>

>

+ – – +
Cu electrode X Cu electrode Y Mg Cu
porous
pot

CuSO4(aq) CuSO4(aq)

Cell I Cell II
(a) On the above diagrams,
(i) label the positive and negative electrodes of cells I and II with ‘+’ or ‘–’ signs;
(ii) indicate the direction of electron flow in cells I and II.

(b) Write ionic half-equations for the reactions that occur at


(i) electrodes X and Y in cell I;

At electrode X: Cu(s) Cu2+(aq) + 2e–

At electrode Y: Cu2+(aq) + 2e– Cu(s)

(ii) the magnesium and copper electrodes in cell II.

At the magnesium electrode: Mg(s) Mg2+(aq) + 2e–

At the copper electrode: Cu2+(aq) + 2e– Cu(s)

88 Worksheet 11 © 2013 Marshall Cavendish International (Singapore) Private Limited

(S)CMWB_11.indd 88 8/15/12 2:55 PM


3. Consider three simple cells, A, B and C, as shown below. Each cell contains sodium chloride
solution as the electrolyte.

v v v

Pb Fe Mg Pb Pb Ag

NaCl(aq) NaCl(aq) NaCl(aq)

Cell A Cell B Cell C

(a) Identify the positive and negative terminals of each of these cells. Match the following voltages
to the cells: 0.31 V; 0.93 V; 2.25 V.

Cell Positive terminal Negative terminal Voltage/V


A Pb Fe 0.31
B Pb Mg 2.25
C Ag Pb 0.93

(b) Explain the following observations in cell A.


(i) The iron electrode decreases in size.
Iron is more reactive than lead. Hence, iron atoms lose electrons to form iron(II) ions, which go

into the solution.

(ii) Bubbles of gas are produced at the lead electrode.


Hydrogen ions present in the aqueous solution gain electrons and are reduced to hydrogen gas

at the lead electrode.

(c) What is the direction of electron flow in cell C? Give your reason.
Electron flow in cell C is from the lead electrode to the silver electrode. Electrons flow from the more

reactive metal (lead) to the less reactive metal (silver).

Section C: Free-Response Questions


Answer the following questions.

*1. An iron object is electroplated with chromium as shown in the diagram below.

chromium (anode)

chromium(III) iron object


nitrate solution (cathode)

© 2013 Marshall Cavendish International (Singapore) Private Limited Worksheet 11 89

(S)CMWB_11.indd 89 8/15/12 2:55 PM


(a) Explain the chemical changes that occur at the
(i) anode;

NO3– and OH– ions migrate to the anode. Since chromium is a reactive electrode, the chromium

anode dissolves, producing chromium(III) ions.

(ii) cathode.
Cr3+ and H+ ions migrate to the cathode. Chromium is formed rather than hydrogen gas because

Cr3+ ions accept electrons more readily than hydrogen ions. The iron object at the cathode

becomes coated with a layer of chromium.

(b) Explain why the concentration of the electrolyte remains the same throughout the
electroplating process.
The chromium(III) ions that are discharged at the cathode come mainly from the chromium anode.

There is no net loss of chromium(III) ions from the chromium(III) nitrate solution. Thus, the

concentration of the electrolyte remains the same.

(c) Give two uses of electroplating metals.

It prevents the corrosion of metals and improves their appearance.

*2. The set-up shown was used to compare the reactivity of four metals, P, Q, R and S. Metal strips P,
Q, R and S and a copper plate were first cleaned with sandpaper. Each metal strip was then clipped
onto the wet filter paper and the voltmeter reading was recorded.

metal strip connecting


under test wire

filter paper soaked with


sodium chloride solution copper plate

90 Worksheet 11 © 2013 Marshall Cavendish International (Singapore) Private Limited

(S)CMWB_11.indd 90 8/15/12 2:55 PM


A summary of the voltmeter readings obtained is shown in the table below.

Metal strip Direction of electron flow in the external circuit Voltmeter reading/V
P P to Cu +0.80
Q Cu to Q –2.20
R R to Cu +1.40
S S to Cu +0.30

(a) Give a reason why the metal strips and copper plate must first be cleaned with sandpaper.

To remove any oxide layer formed on the metal surface, to ensure better electrical conductivity

(b) From the results in the table,


(i) arrange the metals in order of increasing reactivity;

Q, S, P, R

(ii) state how the positions of the metals in the reactivity series affect the magnitude of
voltage.

The further apart the metals are from copper in the reactivity series, the greater the magnitude

of voltage.

(c) (i) Given that R is an element in Group II of the Periodic Table, what is observed if it is
placed in copper(II) sulfate solution?

A reddish-brown solid is deposited. The solution turns from blue to colourless.

(ii) Write an ionic equation for the reaction.


R(s) + Cu2+(aq) Cu(s) + R2+(aq)

(d) State and explain any difference in the voltmeter readings if the experiment is repeated using
filter paper soaked with the organic solvent methylbenzene.
The voltmeter reading will be zero for all four metals. This is because methylbenzene is a covalent

compound and does not conduct electricity.

© 2013 Marshall Cavendish International (Singapore) Private Limited Worksheet 11 91

(S)CMWB_11.indd 91 8/15/12 2:55 PM


WORK ON IT!
In the table below, circle the question(s) that you have answered incorrectly. Revisit the
relevant section(s) in the textbook to strengthen your understanding of the key concept(s).
Question number(s) Textbook
Learning outcomes Multiple- Free- section(s)
Structured to revise
choice response
(a) Define electrolysis. – 1(a) – 15.1
(b) Describe electrolysis as evidence of
the structure of ionic compounds in the 1 – – 15.1
solid, molten and aqueous states.
(c) Describe the electrolysis of molten
– 1(b)(iv) – 15.2
sodium chloride using inert electrodes.
(d) Predict the products of the electrolysis
2 – – 15.2
of a molten ionic compound.
(e) Construct ionic equations for the
reactions occurring at the electrodes 4 2(b) – 15.2
during electrolysis.
(f) Predict the products of the electrolysis 1(b)(i),
of an aqueous electrolyte using the 3, 5 1(b)(ii), – 15.3
idea of selective discharge of ions. 1(b)(iii), 2(b)
(g) Describe the purification of copper by
the electrolysis of aqueous copper(II) – 2(a)(i) – 15.4
sulfate using copper electrodes.
(h) Describe the electroplating of metals
6 – 1 15.4
and its uses.
(i) Describe how electricity can be
produced from simple cells, in relation
7 2(a)(ii), 3 2(b) 15.5
to the reactivity series and redox
reactions.
(j) Relate the physical properties of ionic
– – 2(a) 6.5
compounds to their lattice structures.
(k) Relate the physical properties of
covalent substances to their structure – – 2(d) 7.2
and bonding.
(l) Write ionic equations with state
– – 2(c)(ii) 8.2
symbols.
(m) Relate the reactivity of metals to their
tendency to form positive ions by
• the displacement of metals from
– – 2(c)(i) 14.3
their salt solutions;
• the reaction of a metal with the
oxide of another metal.

92 Worksheet 11 © 2013 Marshall Cavendish International (Singapore) Private Limited

(S)CMWB_11.indd 92 8/15/12 2:55 PM


Name: ( )

Worksheet 12
Class: Date:

The Periodic Table

for Chapter 16
ANSWER IT RIGHT!
Go through the worked example. Use the tips to guide you in understanding and answering
the questions.

(a) (i) Write the chemical formula of rubidium carbonate.


(ii) Explain what is observed when rubidium carbonate is added to dilute sulfuric acid.

Thought Process
Rubidium is a Group I element and forms Rb+ ions. It undergoes similar reactions to other Group I
elements.

Answer
(i) Rb2CO3
(ii) Rubidium carbonate dissolves in sulfuric acid and effervescence occurs.

Write down physical observations, i.e. what you see, hear or feel. An answer such as ‘a gas is given off’
is not acceptable.

(b) (i) Based on the position of silicon in the Periodic Table, deduce the chemical formula of silicon
chloride.
(ii) Draw a ‘dot and cross’ diagram to show the bonding in silicon chloride. Show only the outer
electrons.

Thought Process
(i) Silicon is a Group IV element. It has four valence electrons. It will form four covalent bonds
with chlorine atoms to attain the stable octet electronic configuration.

Answer
(i) SiCl4
(ii)
Cl

Cl Si Cl

Cl

© 2013 Marshall Cavendish International (Singapore) Private Limited Worksheet 12 93

(S)CMWB_12.indd 93 8/15/12 2:59 PM


(c) Chromium is a transition element. List two properties of chromium.

• ‘List’ implies that no elaboration is needed on the points you write down.
• You should only list the number of points that is required.

Answer
Chromium has a high density and high melting and boiling points. It forms coloured compounds.

Remember that transition elements are not coloured. It is their compounds (oxides, hydroxides or salts)
that are coloured.

TEST IT!

Section A: Multiple-Choice Questions


Choose the correct answer and write its letter in the brackets provided.

1. A bar chart for some elements is shown below. What should the label for the x-axis be?
Element

Sc

Ca

Ar

Cl

0 5 10 15 20

A Group number of the element


B Number of electrons in the outermost shell of the element
C Nucleon number of the element
D Proton number of the element ( D )

2. Which statement about elements in the Periodic Table is not true?


A Group III elements form negative ions.
B Group II elements form positive ions.
C Group I elements are reactive metals.
D Group 0 elements are gases. ( A )

3. An element has a proton number of 5. Which group of the Periodic Table does the element belong
to?
A II B III
C IV D V ( B )

94 Worksheet 12 © 2013 Marshall Cavendish International (Singapore) Private Limited

(S)CMWB_12.indd 94 8/15/12 2:59 PM


4. What is the similarity between the elements carbon, nitrogen and oxygen?
A They have the same number of electrons.
B They form negative ions with the same charge.
C They are in the same period of the Periodic Table.
D They are in the same group of the Periodic Table. ( C )

5. The diagram below shows the electronic structure of a molecule of YZ. What could Y and Z be?

Y Key:
valence electron
of atom Y
valence electron
Z of atom Z

Y Z
A chlorine chlorine
B hydrogen oxygen
C oxygen oxygen
D oxygen sodium ( A )

6. Which halogens are gases at room temperature and pressure?


A Bromine and fluorine
B Bromine and iodine
C Chlorine and fluorine
D Chlorine and iodine ( C )

7. The table below shows some properties of two elements from Group I.

Element Relative atomic mass Melting point/°C Boiling point/°C Density/(g/cm3)


Sodium 23 883
Potassium 39 63 0.86

Which prediction is most likely to be correct?


A The density of sodium is 1.53 g/cm3.
B The melting point of sodium is 39°C.
C The melting point of sodium is 98°C.
D The boiling point of potassium is 1347°C. ( C )

© 2013 Marshall Cavendish International (Singapore) Private Limited Worksheet 12 95

(S)CMWB_12.indd 95 8/15/12 2:59 PM


*8. R, S, T and V represent some elements in the simplified Periodic Table below. Which statement is
true? (Note: R, S, T and V are not chemical symbols of the elements.)

S T
R V

transition elements

A S and T have the same number of valence electrons.


B R and V react to form an ionic compound.
C The oxide of R is insoluble in water.
D V is the most powerful oxidising agent of the four elements. ( B )

*9. D, E and F are in the same period of the Periodic Table. The oxide of D reacts with water to form a
solution with a pH less than 7. The oxide of E reacts with water to form a solution with a pH greater
than 7. The oxide of F is soluble in both hydrochloric acid and aqueous sodium hydroxide. What is
the order of elements D, E and F in terms of increasing proton number? (Note: D, E and F are not
chemical symbols of elements.)
A D, E, F B D, F, E
C E, D, F D E, F, D ( D )

Section B: Structured Questions


Answer the following questions in the spaces provided.

1. (a) What determines the position of an element in the Periodic Table?

The proton number of the element and the number of valence electrons in the atom of

the element

(b) Indium (In) is in Group III and Period 5 of the Periodic Table.
(i) In the diagram below, which shows a simplified version of the Periodic Table, mark the
symbol of indium (In).

O
Cl

In

transition elements

(ii) How many valence electrons does indium have? Three

(iii) Predict whether indium conducts electricity. Give a reason for your answer.
Indium conducts electricity as it is a metal.

96 Worksheet 12 © 2013 Marshall Cavendish International (Singapore) Private Limited

(S)CMWB_12.indd 96 8/15/12 2:59 PM


(iv) Write the chemical formulae of indium chloride and indium oxide.
InCl3
Indium chloride:
In2O3
Indium oxide:

(v) Indium chloride reacts with water to form insoluble indium hydroxide and hydrogen
chloride gas. Write an equation for the reaction of indium chloride with water. Include
state symbols.
InCl3(aq) + 3H2O(l) In(OH)3(s) + 3HCl(g)

2. Carbon and silicon belong to the same group of the Periodic Table.
(a) Carbon and silicon react with hydrogen to form methane, CH4, and silane, SiH4, respectively.
(i) Without referring to the Periodic Table, state the group in which carbon and silicon are
found. Explain how you arrived at your answer.
Group IV. Both carbon and silicon have the oxidation state +4 in their compounds. This means

that they have four valence electrons and hence belong to Group IV.

(ii) Methane is used as a fuel. It forms carbon dioxide and water when burnt. Predict the
products that are formed when silane burns in oxygen.
Silicon dioxide and water

(b) The chemical formula of sodium carbonate is Na2CO3. Predict the formula of sodium silicate.
Na2SiO3

3. Caesium, an element in Group I of the Periodic Table, is a liquid at room temperature. It is used in
atomic clocks. Atomic clocks are very accurate time-measurement devices.
Caesium reacts explosively with cold water and also reacts with ice at −110°C.
(a) Describe the trend in the melting points of Group I elements down the group.
The melting points of Group I elements decrease down the group.

(b) State the chemical formulae of


(i) caesium chloride; CsCl

Cs2S
(ii) caesium sulfide;

Cs2SO4
(iii) caesium sulfate;
CsOH and H2
(iv) the products of the reaction between caesium and water.

(c) Explain why caesium reacts with ice at −110°C but lithium does not.
The reactivity of Group I metals increases down the group. Caesium is below lithium in the Periodic

Table. Thus, it is more reactive than lithium.

© 2013 Marshall Cavendish International (Singapore) Private Limited Worksheet 12 97

(S)CMWB_12.indd 97 8/15/12 2:59 PM


4. Element X has the electronic configuration 2, 8, 7. Element Y is above X in the Periodic Table.

(a) Which element, X or Y,

(i) has a higher boiling point; X

(ii) has a lighter colour; Y

(iii) will displace the other element from its salt solution? Y

(b) Element Z is an unreactive element. It is found next to X in the Periodic Table.


(i) Which group does Z belong to?

Group 0

(ii) In terms of electronic configuration, explain why X is reactive but not Z.

X is reactive because its outer shell contains seven electrons and it needs to accept only one

electron to form a stable octet electronic configuration. Z is unreactive as its atoms have fully

filled outer shells and thus do not gain or lose electrons.

Section C: Free-Response Questions


Answer the following questions.

1. (a) Sodium, potassium and rubidium are Group I metals. Explain the following properties of the
Group I metals. Write equations, if any, to support your explanation.
(i) Group I metals are powerful reducing agents.
Reducing agents give away electrons. Sodium, potassium and rubidium give away their valence

electrons readily to form ions with a charge of +1: Na Na+ + e–

(ii) Freshly cut potassium rapidly tarnishes in air.

Potassium is a very reactive metal. It reacts with oxygen in the air to form a layer of potassium

oxide: 4K(s) + O2(g) 2K2O(s)

98 Worksheet 12 © 2013 Marshall Cavendish International (Singapore) Private Limited

(S)CMWB_12.indd 98 8/15/12 2:59 PM


(iii) Rubidium reacts more vigorously with water than potassium does.

The reactivity of Group I metals increases down the group. The equation for the reaction of

rubidium with water is: 2Rb(s) + 2H2O(l) 2RbOH(aq) + H2(g)

(b) Group 0 elements are known as noble gases. They are the least reactive elements in the
Periodic Table.
(i) Explain the unreactivity of noble gases.
Noble gases are unreactive because their atoms have fully filled outer shells.

(ii) State one use of a noble gas in daily life and explain its importance.

Argon is used to fill electric bulbs. It provides an inert atmosphere to protect the filament from

oxidation in air.

2. (a) Chlorine, bromine and iodine are halogens. They react in the gaseous state with hydrogen to
form hydrogen halides. Their reactions are summarised in the table below.

Halogen Conditions and rate of reaction

chlorine rapid reaction in the presence of light

bromine slow reaction when heated in the presence of a catalyst

iodine no reaction unless very strongly heated

(i) Write a chemical equation for the reaction between hydrogen and bromine.
H2(g) + Br2(g) 2HBr(g)

(ii) Based on the data given, state whether chlorine, bromine or iodine is the strongest
oxidising agent.

Chlorine

© 2013 Marshall Cavendish International (Singapore) Private Limited Worksheet 12 99

(S)CMWB_12.indd 99 8/15/12 2:59 PM


(b) An experiment was carried out by adding solutions of halogens A, B and C to each of their
halide solutions. The results of the experiment are shown in the table below. A tick (✓)
indicates a reaction. A cross (✗) indicates no reaction.

Halide solution of A Halide solution of B Halide solution of C


Solution of halogen A ✓ ✗
Solution of halogen B ✗ ✗
Solution of halogen C ✓ ✓

The three halogens are chlorine, bromine and iodine. Based on the results given above,
(i) arrange A, B and C according to an increasing order of reactivity and identify A, B and C;

Order of reactivity: B < A < C

A: Bromine B: Iodine C: Chlorine

(ii) write ionic equations for the reactions that took place.
Br2(aq) + 2I–(aq) 2Br –(aq) + I2(aq)

Cl2(aq) + 2Br –(aq) 2Cl –(aq) + Br2(aq)

Cl2(aq) + 2I–(aq) 2Cl –(aq) + I2(aq)

3. The graph below shows the melting points and boiling points of the first 20 elements in the Periodic
Table.
Temperature/°C

5000

4000 boiling point


melting point

3000

2000

1000

1 2 3 4 5 6 7 8 9 10 11 12 13 14 15 16 17 18 19 20

--1000 Proton number

(a) Circle the melting and boiling points of three alkali metals.

100 Worksheet 12 © 2013 Marshall Cavendish International (Singapore) Private Limited

(S)CMWB_12.indd 100 8/15/12 2:59 PM


(b) Compare the melting points of the alkali metals and other metals.
The melting points of the alkali metals are lower than those of other metals.

(c) Compare the boiling points of the alkali metals with those of other metals.
The boiling points of the alkali metals are lower than the boiling points of other metals.

(d) Rubidium is found below potassium in the Periodic Table. Predict whether rubidium has a
higher or lower boiling point than potassium. Explain your answer.
The boiling point of rubidium should be lower than that of potassium. This is because the boiling

points of alkali metals decrease down the group.

*4. A catalyst is a substance which increases the speed of reaction. Iron and its compounds are often
used as catalysts in chemical reactions. One such example is iron(III) chloride, FeCl3. It can be
used to catalyse the reaction between sodium thiosulfate, Na2S2O8, and potassium iodide, KI.
The reaction is broken down into two steps in the presence of iron(III) chloride. The ionic equations
for these two steps are:
Step 1: 2Fe3+(aq) + 2I–(aq) 2Fe2+(aq) + I2(aq)
Step 2: 2Fe2+(aq) + S2O82–(aq) 2Fe3+(aq) + 2SO42–(aq)
(a) Based on the equations given above, write the ionic equation for the uncatalysed reaction
between Na2S2O8 and KI.
S2O82–(aq) + 2I–(aq) 2SO42–(aq) + I2(aq)

(b) The equations in steps 1 and 2 show that iron forms ions with different oxidation states. Iron
is an example of a transition metal.
(i) Give two other examples of transition metals found in the Periodic Table.

Manganese and copper (Accept any two possible answers.)

(ii) State two other properties of transition metals.

They have high melting and boiling points and form coloured compounds.

(c) Suggest why, unlike iron(II) chloride, calcium chloride cannot be used to catalyse the reaction
between sodium thiosulfate and potassium iodide.

Iron forms compounds with variable oxidation states (+2, +3) but calcium does not (only +2).

(d) Iron is used as a catalyst in many industrial processes. Give an example of these processes.

Haber process for the manufacture of ammonia (Accept any other possible answer.)

© 2013 Marshall Cavendish International (Singapore) Private Limited Worksheet 12 101

(S)CMWB_12.indd 101 8/15/12 2:59 PM


WORK ON IT!
In the table below, circle the question(s) that you have answered incorrectly. Revisit the
relevant section(s) in the textbook to strengthen your understanding of the key concept(s).

Question number(s) Textbook


Learning outcomes Multiple- section(s)
Structured Free-response to revise
choice
(a) Describe the Periodic Table as
an arrangement of the elements 1 – – 16.1
according to proton (atomic) number.
(b) Describe the change in metallic
properties of the elements across a 9 – – 16.2
period of the Periodic Table.
(c) Relate the position of an element in
the Periodic Table to its proton number 3, 4, 5, 8 1(a), 1(b)(i) 3(a) 16.2
and electronic structure.
(d) Describe the relationship between
group number, number of valence 1(b)(ii),
8 – 16.2
electrons and metallic/non-metallic 1(b)(iii)
properties.
(e) Explain the similarities between the
elements in the same group of the – 2 – 16.2
Periodic Table.
(f) Relate the ionic charge of an element
1 1(b)(iv), 2(b) – 16.2
to its group number.
(g) Describe the physical and chemical 1(a), 3(b),
2, 7 3 16.3
properties of Group I elements. 3(c)
(h) Predict the properties of Group I
7 – 3(d) 16.3
elements using the Periodic Table.
(i) Describe the physical and chemical
6 4(a) 2(a)(ii), 2(b) 16.4
properties of Group VII elements.
(j) Predict the properties of Group VII
– 4(b)(i) – 16.4
elements using the Periodic Table.
(k) Describe the Group 0 elements as
– 4(b)(ii) – 16.5
unreactive.
(l) Describe the lack of reactivity of the
noble gases in terms of electronic – – 1(b)(i) 16.5
structure.
(m) Describe the importance of the
noble gases in providing an inert – – 1(b)(ii) 16.5
atmosphere.
(n) Describe the properties of the
– – 4(b), 4(c) 16.6
transition elements.
(o) State that many transition elements
and their compounds are good – – 4(c), 4(d) 16.6
catalysts.
(p) Write chemical equations with state 1(a)(ii),
– 1(b)(v) 8.1
symbols. 1(a)(iii), 2(a)(i)
(q) Write ionic equations with state 1(a)(i),
– – 8.2
symbols. 2(b)(ii), 4(a)

102 Worksheet 12 © 2013 Marshall Cavendish International (Singapore) Private Limited

(S)CMWB_12.indd 102 8/15/12 2:59 PM


Name: ( )

Worksheet 13
Class: Date:

Energy Changes

for Chapter 17
ANSWER IT RIGHT!
Go through the worked example. Use the tips to guide you in understanding and answering
the questions.

When 1.0 mol of propane is burnt in excess oxygen, 2220 kJ of heat is liberated.
C3H8(g) + 5O2(g) 3CO2(g) + 4H2O(g) ∆H = –2220 kJ

(a) Explain why heat energy is liberated when propane burns in oxygen.

Thought Process
The energy change in a chemical reaction depends on whether the energy absorbed during bond
breaking is greater or less than the energy released during bond making.

Answer
Heat energy is liberated because the energy absorbed during bond breaking is less than the energy
released during bond making.

A common mistake in students’ answers is to say that energy is needed to form a bond. The fact is, heat
energy is taken in to break a bond and given out during bond making.

(b) Calculate the heat liberated when 2.5 mol of propane is burnt in excess oxygen.

Thought Process
The amount of heat energy released is directly proportional to the amount of propane used.

Answer
Heat liberated = 2.5 × 2220
= 5550 kJ

(c) Find the mass of propane needed to produce 1665 kJ of heat on burning.

‘Find’ is a general term. It means to calculate, measure, determine and so on.

Answer
Number of moles of propane used = 1665
2220
= 0.75 mol
Mr of propane (C3H8) = (3 × 12) + (8 × 1)
= 44
Mass of propane used = number of moles × molar mass
= 0.75 × 44
= 33 g

© 2013 Marshall Cavendish International (Singapore) Private Limited Worksheet 13 103

(S)CMWB_13.indd 103 8/15/12 3:05 PM


TEST IT!

Section A: Multiple-Choice Questions


Choose the correct answer and write its letter in the brackets provided.

1. Which of the following processes is exothermic?


A Decomposition by heat of zinc carbonate into zinc oxide and carbon dioxide
B Formation of glucose and oxygen during photosynthesis
C Conversion of glucose to carbon dioxide and water during respiration
D Sublimation of dry ice ( C )

2. Which of the following does not involve an endothermic change?


A An iron nail rusting in air
B A few drops of ethanol drying up
C A potted plant making food in sunlight
D A candle melting ( A )

3. A thermometer is placed in water and the temperature is measured at room temperature


(Figure 1). A liquid, X, is then added to the water and the temperature is measured again
(Figure 2).

34 28
32 26
30 24
28 22
26 20
24 18

Figure 1 Figure 2

What conclusion can be drawn from Figures 1 and 2?


A The process of mixing liquid X and water is exothermic.
B The temperature increased by 3.5°C.
C The temperature decreased by 4.0°C.
D The graph that shows the temperature changes after X is added to water should be:

Temperature/°C

room
temperature

Time/min ( D )

104 Worksheet 13 © 2013 Marshall Cavendish International (Singapore) Private Limited

(S)CMWB_13.indd 104 8/15/12 3:05 PM


4. The reaction between hydrogen and oxygen to form water occurs in three stages. In which stages
are the signs of the enthalpy changes correctly shown?
1 2H2(g) + O2(g) 4H(g) + 2O(g) ∆H = positive
2 4H(g) + 2O(g) 2H2O(g) ∆H = positive
3 H2O(g) H2O(l) ∆H = negative
A 1 and 2
B 1 and 3
C 2 and 3
D 1, 2 and 3 ( B )

5. The thermochemical equation for the formation of nitrogen monoxide (NO) in a car engine is
shown below.
1 N (g) + 1 O (g) NO(g) ∆H = +90.5 kJ
2 2
2 2
This implies that .
A the product has more energy than the reactants
B the total energy change for bond making is greater than that for bond breaking
C the formation of the N–O bond in an NO molecule is an endothermic process
D heat energy is released to the surroundings during the reaction ( A )

6. Which of the statements about the hydrogen–oxygen fuel cell is correct?


A The electrolyte is dilute sulfuric acid.
B At the positive electrode (cathode), oxygen is reduced to hydroxide ions.
C At the negative electrode (anode), hydrogen is reduced to water.
D The overall reaction is: H+(aq) + OH–(aq) H2O(l). ( B )

Section B: Structured Questions


Answer the following questions in the spaces provided.

1. (a) (i) The table below shows the heat of combustion of some fuels. The heat of combustion is
the amount of heat produced when 1.0 mol of a fuel is burnt completely in air. Complete
the table by calculating the heat energy liberated when 1.0 g of each fuel is completely
burnt.

Relative
Molecular Heat of
Fuel molecular Heat liberated/(kJ/g)
formula combustion/(kJ/mol)
mass
hydrogen H2 2 –286 –143
methane CH4 16 –890 –56
ethanol C2H6O 46 –1371 –30
butane C4H10 58 –2877 –50

© 2013 Marshall Cavendish International (Singapore) Private Limited Worksheet 13 105

(S)CMWB_13.indd 105 8/15/12 3:05 PM


(ii) Give three advantages of using hydrogen as a fuel.

It is a renewable energy resource. It is pollution free because only water is produced when

hydrogen burns. It gives out the most heat per gram of fuel.

(b) (i) What is a fuel cell?

A fuel cell is a chemical cell in which reactants are continuously supplied to produce electricity.

(ii) In what way is a fuel cell different from an electrolytic cell in terms of energy changes?

In a fuel cell, chemical energy is converted into electrical energy. In an electrolytic cell, electrical

energy is converted into chemical energy.

2. The bond energies of some bonds are given in the table below.

Bond H–H Cl–Cl H–Cl C–C C–H C–Cl


Bond energy/(kJ/mol) 436 243 432 348 413 330

The equation for the reaction between methane (CH4) and chlorine (Cl2) to produce chloroform
(CHCl3) and hydrogen chloride (HCl) is shown below.

H Cl
H C H + 3Cl Cl H C Cl + 3H Cl
H Cl

(a) Calculate the overall enthalpy change of the above reaction.

From the equation above,


Number of moles of C–H bonds broken = 3
Number of moles of Cl–Cl bonds broken = 3
Total energy absorbed for bond breaking = (3 × 413) + (3 × 243)
= 1968 kJ
Number of moles of C–Cl bonds formed = 3
Number of moles of H–Cl bonds formed = 3
Total energy released for bond making = (3 × 330) + (3 × 432)
= 2286 kJ
∆H = Total energy absorbed for bond breaking – total energy released for bond making
= 1968 – 2286
= –318 kJ

106 Worksheet 13 © 2013 Marshall Cavendish International (Singapore) Private Limited

(S)CMWB_13.indd 106 8/15/12 3:05 PM


(b) Hence, deduce whether the reaction is endothermic or exothermic.

The overall enthalpy change of the reaction has a negative value. This indicates that the reaction is

exothermic.

*3. The reaction between A and B to produce C and D is reversible. This means that the reaction can
proceed in both directions: the forward reaction, where A and B react to form C and D; and the
backward reaction, where C and D react to form A and B.
A+B C+D
The energy profile diagram below shows the energy changes as the forward reaction proceeds.

Energy

E3

E2 A+B

E1 C+D

Progress of reaction

(a) What does each of the following energy changes represent?

(i) E2 – E1 : The enthalpy change for the forward reaction

(ii) E3 – E1 : The activation energy for the backward reaction

(iii) E3 – E2 : The activation energy for the forward reaction

(b) Based on the energy profile diagram given, explain whether the forward reaction is
exothermic or endothermic.
It is an exothermic reaction because the energy of the reactants is greater than the energy of the

products.

(c) Based on your answer in (b), deduce whether heat is given out or taken in when C and D react
to form A and B.
Since the forward reaction is exothermic, the backward reaction must be endothermic. Therefore, it is

deduced that heat is taken in during the backward reaction.

© 2013 Marshall Cavendish International (Singapore) Private Limited Worksheet 13 107

(S)CMWB_13.indd 107 8/15/12 3:05 PM


Section C: Free-Response Questions
Answer the following questions.

1. When 1.0 mol of ethanol, CH3CH2OH, is completely burnt in oxygen, 1367 kJ of heat is liberated.
CH3CH2OH(l) + 3O2(g) 2CO2(g) + 3H2O(g) ∆H = –1367 kJ
(a) (i) In terms of bond breaking and bond making, explain why the combustion of ethanol is
exothermic.
The energy released in forming the bonds in CO2 and H2O molecules is greater than the energy

absorbed for breaking the bonds in CH3CH2OH and O2 molecules.

(ii) What is the application of this reaction in the motor industry?


This reaction is used to power cars.

(iii) Sketch the energy profile diagram for the combustion of ethanol.
Energy

Ea
CH3CH2OH +3O2

⌬H = –1367 kJ
2CO2 + 3H2O

Progress of reaction

(b) Calculate the mass of ethanol required to produce 411 kJ of heat.


1 mol of ethanol produces 1367 kJ of heat.
Number of moles of ethanol required to produce 411 kJ of heat = 411
1367
= 0.3 mol
Mr of ethanol (CH3CH2OH) = (2 × 12) + (6 × 1) + 16
= 46
Mass of ethanol required = number of moles × molar mass
= 0.30 × 46
= 13.8 g

(c) When 1.0 mol of propanol, CH3CH2CH2OH, is burnt completely in oxygen, x kJ of heat is
liberated. Would you expect the value of x to be greater or smaller than 1367? Explain your
answer.
The value of x is expected to be greater than 1367. Each molecule of propanol contains more carbon

and hydrogen atoms than ethanol. The combustion of 1.0 mol of propanol thus produces more CO2

and H2O molecules and more heat is liberated.

108 Worksheet 13 © 2013 Marshall Cavendish International (Singapore) Private Limited

(S)CMWB_13.indd 108 8/15/12 3:05 PM


*2. (a) In an experiment, 25.0 cm3 of hydrochloric acid was placed in an insulated container. Aqueous
sodium hydroxide of concentration 1.0 mol/dm3 was then added to the acid, 2.0 cm3 at a
time. The mixture was then stirred and the highest temperature reached was recorded after
each addition. The graph below shows the results of the experiment.

Temperature/°C

35

30

25

20

15

10

0 2 4 6 8 10 12 14 16 18
Volume of NaOH added/cm3

(i) State why the reaction is exothermic.


The temperature of the mixture increased during the experiment.

(ii) From the graph, what was the maximum temperature reached and volume of sodium
hydroxide added?
31.5°C, 11.0 cm3

(iii) Calculate the concentration, in mol/dm3, of hydrochloric acid used in the experiment.

Number of moles of NaOH added = 1.0 × 11


1000
= 0.011 mol
NaOH(aq) + HCl(aq) NaCl(aq) + H2O(l)
From the equation, 1 mol of HCl reacts with 1 mol of NaOH.
Number of moles of HCl = number of moles of NaOH
= 0.011 mol
Volume of HCl = 25.0 cm3
= 0.025 dm3

Concentration of HCl in mol/dm3 = number of moles


3
volume in dm
= 0.011
0.025
= 0.44 mol/dm3

© 2013 Marshall Cavendish International (Singapore) Private Limited Worksheet 13 109

(S)CMWB_13.indd 109 8/15/12 3:05 PM


(b) When 1.0 mol of hydrochloric acid reacts with 1.0 mol of aqueous sodium hydroxide, 57 kJ
of heat energy is given out.
(i) Write the ionic equation for the reaction.
H+(aq) + OH–(aq) H2O(l)

(ii) Deduce the heat energy given out when 100 cm3 of 2.0 mol/dm3 sulfuric acid reacts
with 200 cm3 of 2.0 mol/dm3 aqueous potassium hydroxide. (Hint: Calculate the number
of moles of water formed.)
Number of moles of H2SO4 used = 2.0 × 100
1000
= 0.2 mol
H2SO4(aq) 2H+(aq) + SO42–(aq)
From the equation, 1 mol of H2SO4 produces 2 mol of H+.
Therefore, number of moles of H+ = 2 × 0.2
= 0.4 mol

Number of moles of KOH used = 2.0 × 200


1000
= 0.4 mol
Number of moles of H2O formed = 0.4 mol
Heat energy given out = 0.4 × 57
= 22.8 kJ

WORK ON IT!
In the table below, circle the question(s) that you have answered incorrectly. Revisit the relevant
section(s) in the textbook to strengthen your understanding of the key concept(s).

Question number(s) Textbook


Learning outcomes Multiple- Free- section(s)
Structured to revise
choice response
1(a)(ii), 1(b),
(a) Describe enthalpy change in terms of exothermic 1, 2, 3, 4,
2(b), 3 1(c), 2(a)(i), 17.1
and endothermic changes. 5
2(a)(ii)
(b) Describe bond breaking and bond making as
– – 1(a)(i) 17.2
endothermic and exothermic respectively.
(c) Explain overall enthalpy changes in terms of the 1(a)(i),
– 2 17.2
energy involved in bond breaking and bond making. 2(b)(ii)
(d) Draw energy profile diagrams to represent
exothermic and endothermic reactions and show – 3 1(a)(iii) 17.3
the activation energies of reactions.
(e) Describe hydrogen as a potential fuel, which
reacts with oxygen in a fuel cell to produce 6 1 – 17.4
electricity.
(f) Perform calculations involving the number of
– – 1(b) 9.3
moles, mass and molar mass of a substance.
(g) Perform stoichiometric calculations involving 2(a)(iii),
– – 10.4
concentrations of solutions. 2(b)(ii)
(h) Describe what is meant by neutralisation and write
– – 2(b)(i) 11.2
the ionic equation for the neutralisation reaction.

110 Worksheet 13 © 2013 Marshall Cavendish International (Singapore) Private Limited

(S)CMWB_13.indd 110 8/15/12 3:05 PM


Name: ( )

Worksheet 14
Class: Date:

Speed of Reaction

for Chapter 18
ANSWER IT RIGHT!
Go through the worked example. Use the tips to guide you in understanding and answering
the questions.

(a) Zinc powder was added to excess dilute hydrochloric acid. The volume of hydrogen gas produced
was recorded at regular time intervals. The graph below shows the results of the experiment.

Volume of H2/cm3

80

60

40

20

0
50 100 150 200 Time/s

(i) At what time did the reaction stop? Why?


(ii) Estimate the time required for half the mass of zinc to react with dilute hydrochloric acid.
Explain your answer.

The term ‘estimate’ implies an approximate calculation of the amount or value, based on the information
given (in this case, the graph).

© 2013 Marshall Cavendish International (Singapore) Private Limited Worksheet 14 111

(S)CMWB_14.indd 111 8/15/12 3:09 PM


Thought Process
(i) The horizontal portion of the curve shows that the reaction has stopped.

Answer
(i) 175 s. The reaction stopped because all the zinc metal had reacted.
(ii) Total volume of hydrogen produced = 74 cm3
If half the mass of zinc has reacted, the volume of hydrogen produced will be 37 cm3.
From the graph, the time taken to produce 37 cm3 of hydrogen is 35 s.

(b) The reaction between zinc and dilute hydrochloric acid is catalysed by copper(II) sulfate, CuSO4.
(i) Define the term ‘catalyst’.

‘Define’ requires you to state the meaning of the term.

(ii) Sketch the graphs of the volume of hydrogen produced against time for the reaction between
zinc and dilute hydrochloric acid with and without copper(II) sulfate.

• The term ‘sketch’ implies that your curve need not be accurately drawn on graph paper.
• However, the shape of your curve must be correct. Your curve may need to display some important
values. In this case, both graphs should start from the origin and show the same final volume of
hydrogen.

Answer
(i) A catalyst is a substance which increases the speed of a chemical reaction and remains
chemically unchanged at the end of the reaction.
(ii) Volume of H2/cm3

with CuSO4

without CuSO4

Time/s
0

TEST IT!

Section A: Multiple-Choice Questions


Choose the correct answer and write its letter in the brackets provided.

1. When excess magnesium ribbon is added to dilute hydrochloric acid, the reaction gradually
becomes slower and finally stops. Which of the following statements best explains this
observation?
A The concentration of the hydrochloric acid decreases until it finally becomes zero.
B The magnesium ribbon is slowly being coated with an insoluble layer of magnesium chloride.
C The magnesium ribbon gradually becomes smaller.
D The temperature of the reaction mixture gradually decreases as the reaction proceeds. ( A )

112 Worksheet 14 © 2013 Marshall Cavendish International (Singapore) Private Limited

(S)CMWB_14.indd 112 3/10/16 10:45 AM


2. Excess hydrochloric acid is added to a given mass of calcium carbonate. A graph of volume of
carbon dioxide produced against time is plotted.

Volume of CO2/cm3

40

30

20

10

0 1.0 2.0 3.0 4.0 Time/min

Which of the following statements about this experiment is true?


A At 2.4 minutes, all the calcium carbonate has reacted.
B The reaction is complete after 3.0 minutes.
C The speed of the reaction is fastest at 3.0 minutes.
D The time taken for half the mass of calcium carbonate to react is 1.1 minutes. ( A )

3. Which of the following acids reacts the fastest when 1.0 g of magnesium powder is added to it?
(Assume that all the experiments are carried out at the same temperature.)
A 50.0 cm3 of 1.0 mol/dm3 sulfuric acid
B 50.0 cm3 of 1.0 mol/dm3 hydrochloric acid
C 50.0 cm3 of 1.0 mol/dm3 nitric acid
D 100.0 cm3 of 1.0 mol/dm3 ethanoic acid ( A )

4. The time taken for 5.0 g of marble chips to react completely with 50 cm3 of dilute hydrochloric
acid was 10 minutes. The time taken for the same experiment would be longer than 10 minutes if
.
A the reaction was carried out using smaller pieces of marble chips
B concentrated hydrochloric acid was used
C distilled water was added to the reaction mixture
D a catalyst was added to the reaction mixture ( C )

5. The gases, Q and R, react according to the equation:


piston
Q(g) + R(g) T(g)
The reaction mixture is placed in a container at room
temperature as shown in the diagram. Which of the
following actions can speed up the formation of gas T?
A Adding an inert gas to the mixture mixture of
B Using a bigger container Q and R

C Lowering the piston in the container


D Placing the container in water at 0°C
( C )

© 2013 Marshall Cavendish International (Singapore) Private Limited Worksheet 14 113

(S)CMWB_14.indd 113 8/15/12 3:09 PM


6. Manganese(IV) oxide, MnO2, catalyses the decomposition of hydrogen peroxide to form oxygen
and water. Which of the following statements about manganese(IV) oxide are true?
1 It causes hydrogen peroxide molecules to move faster.
2 It decreases the activation energy of the reaction.
3 It remains chemically unchanged after the reaction.
4 Its mass remains unchanged after the reaction.
A 1 and 2 B 1 and 3
C 3 and 4 D 2, 3 and 4 ( D )

7. Catalysts are commonly used to speed up industrial processes. Which of the following is incorrect?
Process Catalyst
A cracking calcium oxide
B manufacture of ammonia iron
C manufacture of margarine nickel
D removal of air pollutants from exhaust gases platinum ( A )

8. Catalase is an enzyme commonly found in living cells. It increases the rate of decomposition
of hydrogen peroxide found in human bodies into water and oxygen gas. Which of the following
statements about catalase is most likely to be correct?
A It operates effectively at all temperatures.
B It operates most effectively over a certain range of pH.
C It increases the activation energy of the decomposition of hydrogen peroxide.
D It increases the yield of products from the decomposition of hydrogen peroxide. ( B )

9. A substance, X, is added to a reaction mixture. The energy profile diagram below shows the effect
of X on the reaction.

Energy

without X
with X
reactants

products

Progress of reaction

Which of the following observations are not true?


1 The enthalpy change of the reaction decreases.
2 The activation energy of the reaction remains unchanged.
3 The reaction takes a shorter time to complete.
A 1 and 2 B 1 and 3
C 2 and 3 D 1, 2 and 3 ( A )

114 Worksheet 14 © 2013 Marshall Cavendish International (Singapore) Private Limited

(S)CMWB_14.indd 114 8/15/12 3:09 PM


*10. An experiment involving the decomposition of Volume of O2/cm3
10.0 cm3 of hydrogen peroxide of concentration
0.4 mol/dm3 was carried out at 30°C. The results II
are represented by Graph I on the right.
I
The experiment was repeated under different
conditions and the results are represented by
Graph II. Which of the following would produce
Graph II? (The mass of manganese(IV) oxide used in
both experiments was 1.0 g.)
Time/min

Concentration of
Volume of hydrogen
Experiment hydrogen peroxide/ Temperature/°C
peroxide/cm3
(mol/dm3)
A 10.0 0.25 30
B 12.5 0.40 30
C 20.0 0.25 28
D 20.0 0.40 40 ( C )

Section B: Structured Questions


Answer the following questions in the spaces provided.

1. Three experiments were carried out to study the effect of particle size on the speed of reaction
between marble chips (CaCO3) and hydrochloric acid. The graphs below show how the volume of
carbon dioxide gas given off varied with time. In each experiment, a fixed volume of hydrochloric
acid and excess marble chips were used.

Volume of CO2/cm3

I
II

Q
III

0 Time/min

© 2013 Marshall Cavendish International (Singapore) Private Limited Worksheet 14 115

(S)CMWB_14.indd 115 8/15/12 3:09 PM


(a) State two variables that must be kept constant during the experiments.
The temperature at which the reaction occurs and the concentration of hydrochloric acid

(b) At which point, P, Q or R, on Graph I was the reaction most vigorous?

Point P

(c) Why does Graph I become horizontal at point R?

At point R, all the hydrochloric acid has been used up and the reaction has stopped. Thus, the total

volume of carbon dioxide gas produced remains constant.

(d) Which graph represents the experiment using

(i) large marble chips; Graph III

(ii) small marble chips; Graph II

(iii) powdered marble? Graph I

(e) In another experiment, 1.0 g of calcium carbonate powder was reacted with excess hydrochloric
acid at 25°C to produce v cm3 of carbon dioxide. The volume of carbon dioxide produced
against time is represented by Graph X.

Volume of CO2/cm3

2v Y

v X

0 Time/min

Graph Y was obtained when the experiment was repeated under different conditions. Suggest
two changes in the conditions that will produce Graph Y.

The mass of calcium carbonate powder used is 2.0 g. The experiment is carried out at a

temperature higher than 25°C./ Hydrochloric acid of a higher concentration is used.

116 Worksheet 14 © 2013 Marshall Cavendish International (Singapore) Private Limited

(S)CMWB_14.indd 116 8/15/12 3:09 PM


*2. Wine slowly turns sour when it is exposed to air. This is due to the reaction between ethanol in the
wine and oxygen in the air to form ethanoic acid.
ethanol + oxygen ethanoic acid + water
However, when a piece of red-hot platinum wire is suspended over ethanol vapour as shown below,
a sour smell is quickly detected and the wire continues to glow for some time.

red-hot
platinum wire
ethanol vapour
mixed with air

ethanol

(a) Deduce the role of the platinum wire in this experiment. Give a reason for your answer.

The platinum wire acts as a catalyst. It increases the speed of the reaction.

(b) Explain why the platinum wire continues to glow for some time.
The reaction between ethanol and oxygen is an exothermic process. The heat energy given out by the

reaction continues to heat the platinum wire.

(c) Sketch the energy profile diagram for the conversion of ethanol to ethanoic acid with and
without the platinum wire.
Energy
without the platinum wire

with the platinum wire

Ea Ea’
ethanol +
oxygen

ethanoic acid ∆H = negative


+ water

Progress of reaction

(d) Use your answer in (c) to explain the effect of the platinum wire on the reaction between
ethanol and oxygen.
The activation energy of the reaction is lowered. Hence, Ea’ is less than Ea.

The enthalpy change of the reaction, ∆H, remains unchanged.

© 2013 Marshall Cavendish International (Singapore) Private Limited Worksheet 14 117

(S)CMWB_14.indd 117 8/15/12 3:09 PM


(e) In the industrial preparation of alcoholic drinks, sugar or starch is converted into alcohol
with the use of enzymes. During the conversion, the temperature is not allowed to go above
37°C or below 14°C. Give a reason for this condition.

When the temperature is too high, the enzymes will be denatured. When the temperature is too low,

the enzymes will become inactive.

*3. Our stomachs secrete hydrochloric acid. Antacid tablets can reduce excess acidity in our
stomachs. A student carried out six experiments to study the speed of reaction between an antacid
tablet and excess hydrochloric acid.

Volume used in the Temperature at Time taken for the tablet to react
Experiment mixture/cm3 the start of the completely with the acid/s
Acid Water experiment/°C Crushed tablet Whole tablet
1 10 40 20 30 35
2 20 30 22 24 30
3 30 20 28 18 21
4 30 20 22 22 25
5 20 30 20 26 32
6 10 40 29 24 29

(a) How would the student know that the tablet had reacted completely with the acid?
When the tablet had dissolved completely in the acid

(b) Explain why the total volume of the acid–water mixture was kept constant in all the experiments.

The total volume was kept constant so that the concentration of the acid in each experiment would be

proportional to the volume of acid used.

(c) Using the results of the experiments in the table, explain the effect each of the following
actions had on the speed of reaction between the acid and the tablet.
(i) Crushing the tablet

The speed of reaction increased. For each experiment, the time taken for the tablet to react was

always shorter for the crushed tablet than for the whole tablet.

(ii) Increasing the temperature

The speed of reaction increased. Comparing the results of Experiments 2 and 5 (or 3 and 4),

where the concentration of the acid was kept constant, a shorter time was needed for the tablet

to react completely when the temperature was increased.

118 Worksheet 14 © 2013 Marshall Cavendish International (Singapore) Private Limited

(S)CMWB_14.indd 118 8/15/12 3:09 PM


(iii) Increasing the concentration of the acid
The speed of reaction increased. Comparing the results of Experiments 1 and 5, where the

temperature was kept constant, the time taken for the tablet to react completely was shorter

when the volume (and hence concentration) of acid used was higher.

Section C: Free-Response Questions


Answer the following questions.

1. Marble chips (calcium carbonate) react with dilute hydrochloric acid according to the equation:
CaCO3(s) + 2HCl(aq) CaCl2(aq) + H2O(l) + CO2(g)
A conical flask containing marble chips and dilute hydrochloric acid was placed on an electronic
balance. The decrease in the mass of the reaction mixture was recorded at half-minute intervals.
The results of this experiment are shown in the table below.

Time/min Total loss in mass/g


0.0 0.00
0.5 0.13
1.0 0.24
1.5 0.33
2.0 0.40
2.5 0.46
3.0 0.50
3.5 0.50
4.0 0.50

(a) Using the information from the table, calculate the volume, in cm3, of carbon dioxide produced
at room temperature and pressure in 1.5 minutes. (1.0 mol of any gas occupies 24 dm3
at r.t.p.)
Mass of carbon dioxide produced in 1.5 min = total loss in mass from the reaction after 1.5 min
= 0.33 g
Mr of CO2 = 12 + (2 × 16)
= 44
Number of moles of CO2 = mass
molar mass
= 0.33
44
= 0.0075 mol
Volume of CO2 produced = number of moles × 24 000 cm3
= 0.0075 × 24 000
= 180 cm3

© 2013 Marshall Cavendish International (Singapore) Private Limited Worksheet 14 119

(S)CMWB_14.indd 119 8/15/12 3:09 PM


(b) Using the information from the table, show that the speed of the reaction decreased with
time.
The rate of mass loss indicates the speed of reaction, i.e. how quickly the marble chips reacted with
hydrochloric acid.
Mass loss between 0–0.5 minutes = 0.13 – 0.00
= 0.13 g
Mass loss between 2–2.5 minutes = 0.46 – 0.40
= 0.06 g
Mass loss between 2.5–3.0 minutes = 0.50 – 0.46
= 0.04 g
The rate of total mass loss decreased as the reaction proceeded. This shows that the speed of the
reaction decreased with time.

(c) Explain why the speed of the reaction decreased with time.

As the reaction proceeded, the concentration of hydrochloric acid and the amount of marble chips

decreased. This caused the frequency of effective collisions between hydrochloric acid and marble

chips to decrease. Hence, the speed of reaction decreased.

(d) Suggest another method to measure the speed of the reaction.


Measure the volume of carbon dioxide produced at half-minute intervals.

2. When aqueous sodium thiosulfate (Na2S2O3) and hydrochloric acid are mixed, a yellow precipitate
of sulfur is produced. The solution becomes increasingly cloudy as more sulfur is formed.
Na2S2O3(aq) + 2HCl(aq) S(s) + 2NaCl(aq) + SO2(g) + H2O(l)
Four experiments were carried out to study the effect of the concentration of sodium thiosulfate
on the speed of reaction. Only the concentration of sodium thiosulfate was changed for each
experiment.

conical flask

HCl(aq) and Na2S2O3(aq)

white paper with a cross

The time taken for the cross on the paper to disappear from view was recorded.

Experiment A B C D
Time taken/s 110 80 50 40

(a) (i) Write the ionic equation for the reaction between sodium thiosulfate and hydrochloric
acid.
S2O32–(aq) + 2H+(aq) S(s) + SO2(g) + H2O(l)

120 Worksheet 14 © 2013 Marshall Cavendish International (Singapore) Private Limited

(S)CMWB_14.indd 120 8/15/12 3:09 PM


(ii) Explain, in terms of collisions between particles, how the concentration of sodium
thiosulfate affects the speed of reaction.
When the concentration of sodium thiosulfate is increased, there are more S2O32– ions and this

increases the frequency of effective collisions between S2O32– ions and H+ ions. The speed of

reaction thus increases.

(b) Deduce which experiment had used sodium thiosulfate with the highest concentration.
Explain your answer.
Experiment D. It took the shortest reaction time which implies that the concentration of sodium

thiosulfate used was the highest compared to that used in other experiments.

(c) Suggest two conditions that should be kept constant when carrying out the experiments.
The temperature of the reactants and the concentration of hydrochloric acid

*3. Explain each of the observations below in term of collisions between particles.
(a) Nitrogen reacts more vigorously with hydrogen at pressures above 200 atmospheres than at
1 atmosphere.

At high pressures, the nitrogen and hydrogen molecules are closer together. Thus, effective collisions

occur more frequently and this increases the speed of reaction.

(b) Placing a light stick (or glow stick) in hot water makes it glow more brightly.
In hot water, the temperature is higher and reacting particles in the light stick gain energy and move

about with greater speed, thus colliding with one another more frequently. As a result, effective

collisions occur more frequently and the speed of reaction increases, causing the light stick to glow

more brightly.

(c) When a piece of raw liver is dropped into hydrogen peroxide, there is a rapid reaction and
oxygen gas is liberated.
Enzymes present in the liver catalyse the decomposition of hydrogen peroxide to produce oxygen

gas. Enzymes are biological catalysts. The presence of enzymes lowers the activation energy of the

reaction. As a result, more reacting particles possess energy equal to or greater than the activation

energy. Effective collisions occur more frequently and hence the speed of reaction increases.

© 2013 Marshall Cavendish International (Singapore) Private Limited Worksheet 14 121

(S)CMWB_14.indd 121 8/15/12 3:09 PM


WORK ON IT!
In the table below, circle the question(s) that you have answered incorrectly. Revisit the
relevant section(s) in the textbook to strengthen your understanding of the key concept(s).

Question number(s) Textbook


Learning outcomes Multiple- Free- section(s)
Structured to revise
choice response
(a) Explain that reactions can occur at
1, 2 – – 18.1
different speeds.
(b) Describe the different methods that
can be used to investigate the speed – – 1(d) 18.2
of reaction.
(c) Describe how the speed of reaction
can be affected by concentration,
3, 4, 5, 10 1(d), 3(c) – 18.3
pressure, particle size and
temperature.
(d) Explain the effects of these factors in 1(c), 2(a)(ii),
– – 18.3
terms of collisions between particles. 3(a), 3(b)
(e) Suggest a suitable method to
1(a), 1(e),
investigate the effect of a particular – 2(c) 18.3
3(b)
variable on the speed of reaction.
(f) Interpret experimental data from 1(b), 1(c),
1(b), 1(c),
investigations on the speed of 2, 10 1(d), 3(a), 18.3
2(b)
reaction. 3(c)
(g) Define catalysts and describe the
effect of catalysts (including enzymes) 6, 9 2(a) – 18.4
on the speed of reaction.
(h) Explain the effect of catalysts on the
speed of reaction in terms of collisions – – 3(c) 18.4
between particles.
(i) Relate the increase in speed of
reaction to pathways with lower 6, 9 2(c), 2(d) 3(c) 18.4
activation energies.
(j) State that some compounds act
as catalysts in a range of industrial 7 – – 18.4
processes.
(k) State that enzymes are biological
8 2(e) 3(c) 18.4
catalysts.
(l) Write ionic equations with state
– – 2(a)(i) 8.2
symbols.
(m) Perform calculations involving molar
gas volumes and the number of moles – – 1(a) 9.6
of a gas.
(n) Describe enthalpy change in terms of
– 2(b) – 17.1
exothermic and endothermic reactions.
(o) Draw energy profile diagrams to
represent exothermic and endothermic
– 2(c) – 17.3
reactions and show the activation
energies of reactions.

122 Worksheet 14 © 2013 Marshall Cavendish International (Singapore) Private Limited

(S)CMWB_14.indd 122 8/15/12 3:09 PM


Name: ( )

Worksheet 15
Class: Date:

Ammonia

for Chapter 19
ANSWER IT RIGHT!
Go through the worked example. Use the tips to guide you in understanding and answering
the questions.

The reaction between nitrogen and hydrogen to form ammonia is a reversible reaction.
N2(g) + 3H2(g) ⇌ 2NH3(g)
The graph below shows the percentage yield of ammonia at different pressures and temperatures.

Percentage yield of NH3/%

100
200 °C

90

80
400 °C

70

60
500 °C

50

40

30

20

10

0
200 400 600 800 1000
Pressure/atm

(a) What is the percentage yield of ammonia at 400°C and 300 atm?

Answer
50%

© 2013 Marshall Cavendish International (Singapore) Private Limited Worksheet 15 123

(S)CMWB_15.indd 123 8/15/12 3:12 PM


(b) Deduce, from the graph, the optimal conditions for the synthesis of ammonia from nitrogen
and hydrogen. Give a reason for your answer.

‘Deduce’ requires you to make use of the information provided to answer the question.

Thought Process
The graph shows two important facts:
• An increase in temperature decreases the yield of ammonia.
• An increase in pressure increases the yield of ammonia.
Hence, the conditions for obtaining a good yield of ammonia are a low temperature and a high
pressure.

Answer
The optimal conditions for the maximum yield of ammonia are 200°C and 600 atm. The graph
shows that the lower the temperature and the higher the pressure, the higher the yield of ammonia.
The yield of ammonia at 200°C does not increase at pressures above 600 atm.

• Do not get confused over speed of reaction and yield.


• An increase in temperature increases the speed of reaction. However, for some reversible reactions, an
increase in temperature decreases the yield.
• Thus, for such reactions, it is best to use a low temperature to increase the yield and a catalyst to
increase the speed of reaction.

TEST IT!

Section A: Multiple-Choice Questions


Choose the correct answer and write its letter in the brackets provided.

1. When 1 mol of nitrogen reacts with 3 mol of hydrogen under suitable conditions, how many moles
of ammonia will be obtained?
A Exactly 2 mol
B Exactly 4 mol
C Less than 2 mol
D More than 4 mol ( C )

2. Which of the following shows the optimal conditions for manufacturing ammonia in the Haber
process?

Volume ratio of H2 : N2 Temperature/°C Pressure/atm Catalyst


A 1:3 250 450 platinum
B 1:3 450 250 iron
C 3:1 250 450 platinum
D 3:1 450 250 iron ( D )

124 Worksheet 15 © 2013 Marshall Cavendish International (Singapore) Private Limited

(S)CMWB_15.indd 124 8/15/12 3:12 PM


3. Which of the following statements about the Haber process is incorrect?
A A high temperature will increase the yield of ammonia.
B It is not possible to obtain a 100% yield of ammonia.
C Nitrogen is reduced by hydrogen in the process.
D An iron catalyst is used to increase the speed of the reaction. ( A )

4. In the Haber process, the yield of ammonia can be increased by .


A increasing the pressure to 300 atm
B increasing the temperature to 1500°C
C using a mixture containing three parts of nitrogen to one part of hydrogen
D using platinum as a catalyst ( A )

5. Which of the following mixtures produces ammonia when heated?


A NH4NO3 + Al
B NH4NO3 + NaCl
C CO(NH2)2 + HCl
D CH3COONH4 + Ba(OH)2 ( D )

6. Which of the following salts will produce the greatest mass of ammonia when heated with aqueous
sodium hydroxide?
A 0.2 mol (NH4)2SO4
B 0.2 mol (NH4)3PO4
C 0.5 mol NH4Cl
D 0.5 mol NH4NO3 ( B )

Section B: Structured Questions


Answer the following questions in the spaces provided.

1. Ammonia is manufactured industrially by the Haber process. The raw materials used are nitrogen
and hydrogen. The Haber process is a reversible reaction.
(a) What is meant by a ‘reversible reaction’?
A reversible reaction is a reaction that can go both forward and backward at the same time.

(b) Write the chemical equation for the Haber process.


N2(g) + 3H2(g) ⇌ 2NH3(g)

© 2013 Marshall Cavendish International (Singapore) Private Limited Worksheet 15 125

(S)CMWB_15.indd 125 8/15/12 3:12 PM


(c) Briefly describe how ammonia gas can be obtained from aqueous sodium hydroxide and
ammonium dichromate(VI), (NH4)2Cr2O7. Write a balanced chemical equation for the reaction.
Ammonia is displaced from ammonium dichromate(VI) when the salt is heated with sodium hydroxide.

The equation for this reaction is:

(NH4)2Cr2O7(s) + NaOH(aq) NH3(g) + H2O(l) + Na2Cr2O7(aq)

2. Ammonia is widely used in the agricultural industry. It is used for manufacturing fertilisers such as
ammonium nitrate, NH4NO3, and ammonium phosphate, (NH4)3PO4. The fertilisers are a source of
nitrogen, which is required for plants to grow healthily.
(a) Calculate the percentage of nitrogen in ammonium nitrate.

Mr of ammonium nitrate, NH4NO3 = 14 + (4 × 1) + 14 + (3 × 16)


= 80
Mass of nitrogen in 1 mol of NH4NO3 = 2 × 14
= 28 g
mass of nitrogen in 1 mol of NH4NO3
Percentage of nitrogen in ammonium nitrate = × 100%
Mr of NH4NO3
= 28 × 100%
80
= 35%

(b) A farmer added ammonium nitrate to soil that has been recently treated with calcium
hydroxide.
(i) State one purpose of adding calcium hydroxide to the soil.
To neutralise excess acidity / increase the pH of the soil so that crops can grow healthily

(ii) Explain what happens when ammonium nitrate is added to the soil.
Calcium hydroxide will react with ammonium nitrate to form ammonia gas, which then escapes

into the atmosphere. This will cause a loss of nitrogen from the soil.

(iii) Write an equation for your answer in (b)(ii).


2NH4NO3(aq) + Ca(OH)2(aq) 2NH3(g) + 2H2O(l) + Ca(NO3)2(aq)

126 Worksheet 15 © 2013 Marshall Cavendish International (Singapore) Private Limited

(S)CMWB_15.indd 126 8/15/12 3:12 PM


Section C: Free-Response Question
Answer the following question.

1. The diagram below shows the Haber process for manufacturing ammonia.

process X nitrogen
liquid
N2 : H2 = 1 : 3 converter condenser
ammonia

process Y hydrogen

unreacted nitrogen and


hydrogen recycled

(a) (i) Name the processes X and Y for obtaining nitrogen and hydrogen.

Process X: Fractional distillation of liquid air

Process Y: Cracking of petroleum

(ii) Briefly describe how the mixture of nitrogen and hydrogen gases is converted into
ammonia in the converter.

The mixture of gases is compressed to 250 atm. The gases are then heated to 450°C and

passed over finely divided iron, which acts as a catalyst in the reaction. Ammonia gas is formed

as the product.

(iii) Suggest one advantage of recycling unreacted nitrogen and hydrogen in the reaction.
Recycling the unreacted gases allows more gases to react and thus helps to save costs.

(b) (i) What volume of ammonia is expected to be produced when 36 dm3 of nitrogen
reacts with 72 dm3 of hydrogen in the reaction? (All volumes are measured at room
temperature and pressure.)

N2(g) + 3H2(g) ⇌ 2NH3(g)


From the equation, 3 volumes of H2 react with 1 volume of N2.
72 dm3 of H2 will react with 24 dm3 of N2.
However, 36 dm3 of N2 is used. Therefore, H2 is the limiting reactant.

Volume of ammonia produced = 2 × 72


3
= 48 dm3

© 2013 Marshall Cavendish International (Singapore) Private Limited Worksheet 15 127

(S)CMWB_15.indd 127 8/15/12 3:12 PM


(ii) The actual volume of ammonia produced in the reaction is less than the value obtained
in (b)(i). Explain this fact.

The reaction between nitrogen and hydrogen is a reversible process. Thus, some of the

ammonia produced will be converted back into nitrogen and hydrogen.

(iii) Explain, in terms of collisions between molecules, how an increase in pressure would
increase the speed of the reaction.

When the pressure is increased, the number of collisions between nitrogen and hydrogen

molecules per unit volume increases, leading to more effective collisions. Hence, the speed of

the reaction increases.

WORK ON IT!
In the table below, circle the question(s) that you have answered incorrectly. Revisit the
relevant section(s) in the textbook to strengthen your understanding of the key concept(s).

Question number(s) Textbook


Learning outcomes Multiple- Free- section(s)
Structured to revise
choice response

(a) State that some reactions, such as the


1 1(a), 1(b) 1(b)(ii) 19.1
formation of ammonia, are reversible.

(b) Describe the manufacture of ammonia


2 – 1(a), 1(b)(ii) 19.2
from nitrogen and hydrogen.

(c) Describe the operating conditions in


the manufacture of ammonia in the 2, 3, 4 – 1(a)(ii) 19.2
Haber process.

(d) Describe the displacement of ammonia 1(c), 2(b)(ii),


5 – 19.3
from its salts. 2(b)(iii)

(e) Write balanced chemical equations with


– 2(b)(iii) – 8.1
state symbols.

(f) Perform calculations involving the


number of moles, mass and molar 6 – – 9.3
mass of a substance.

(g) Determine the percentage composition


– 2(a) – 9.4
of compounds from given data.

(h) Perform stoichiometric calculations


– – 1(b)(i) 10.3
involving the idea of limiting reactants.

(i) Describe the reasons for controlling


the pH of soil and the methods used to – 2(b)(i) – 11.4
reduce excess acidity in soil.

(j) Describe how the speed of reaction can


be affected by concentration, pressure, – – 1(b)(iii) 18.3
particle size and temperature.

128 Worksheet 15 © 2013 Marshall Cavendish International (Singapore) Private Limited

(S)CMWB_15.indd 128 8/15/12 3:12 PM


Name: ( )

Worksheet 16
Class: Date:

The Atmosphere and Environment

for Chapter 20
ANSWER IT RIGHT!
Go through the worked example. Use the tips to guide you in understanding and answering
the questions.

(a) Every year, a large quantity of coal is burnt as a fuel to produce electricity and heat. When coal
undergoes combustion, carbon dioxide and water vapour are produced. Discuss the effects of
carbon dioxide on the environment.

• ‘Discuss’ implies that a critical account of the issues on a topic is involved.


• In this case, the topic is ‘carbon dioxide’ and the issue to be discussed is ‘effects on the environment’.

Answer
Carbon dioxide is a greenhouse gas, which retains heat in the atmosphere. This produces a
warming effect called the greenhouse effect, which maintains a uniform temperature on the
Earth’s surface. This is important for sustaining life on Earth.

However, human activities like the burning of fossil fuels and deforestation can cause carbon
dioxide to build up rapidly in the atmosphere. This will result in global warming, which may lead to
negative consequences, such as a decrease in crop yields due to droughts.

(b) Coal also contains small quantities of sulfur. Describe the health and environmental problems
caused by sulfur when coal is burnt.

Answer
When sulfur-containing coal is burnt, sulfur dioxide is produced. Sulfur dioxide irritates the eyes
and causes breathing difficulties. It dissolves in rainwater to form acid rain. Acid rain damages
metal bridges and marble buildings. It also reduces the pH of water in rivers and lakes to below 4,
which kills fish and other aquatic life.

Do not confuse sulfur with sulfur dioxide. A common mistake is to state that ‘sulfur dissolves in water to
form acid rain’. In fact, sulfur does not react with water.

TEST IT!

Section A: Multiple-Choice Questions


Choose the correct answer and write its letter in the brackets provided.

1. One of the gases in clean, dry air contains two double covalent bonds in its molecule. What is the
composition (by volume) of this gas?
A 0.03% B 1% C 21% D 78% ( A )

© 2013 Marshall Cavendish International (Singapore) Private Limited Worksheet 16 129

(S)CMWB_16.indd 129 3/10/16 10:16 AM


2. Which air pollutant is correctly matched with its source and effect on the environment?

Air pollutant Source Effect on the environment


A carbon monoxide incomplete combustion of petrol acid rain
B methane decay of vegetable matter global warming
C sulfur dioxide volcanic eruption global warming
D ozone photochemical reaction acid rain
( B )

3. When coal burns, sulfur dioxide is produced. Which of the following pairs of chemicals is used to
remove sulfur dioxide present in flue gases?
A CaSO4 and CaCO3 B CaSO3 and CaSO4
C CaO and CaSO4 D CaCO3 and CaO ( D )

4. Which of the following statements about ozone are correct?


1 The ozone layer filters out some of the harmful ultraviolet radiation from the Sun.
2 Ozone is an air pollutant in the upper layer of the atmosphere.
3 The depletion of the ozone layer is caused by chloroalkanes such as CH2ClCCl3.
4 Compounds such as CFCl3 destroy the ozone layer.
A 1 and 3 B 1 and 4
C 1, 2 and 3 D 2, 3 and 4 ( B )

5. Which of the following can cause an increase in the Earth’s average temperature?
1 Photosynthesis of plants
2 Burning of fuels in power stations
3 Respiration of plants and animals
A 1 and 2 B 1 and 3
C 2 and 3 D 1, 2 and 3 ( C )

Section B: Structured Questions


Answer the following questions in the spaces provided.

1. (a) Name one air pollutant that


(i) reacts with hydrocarbons to form ozone; nitrogen dioxide

(ii) causes the depletion of the ozone layer; chlorofluorocarbons (CFCs)

(iii) reacts with haemoglobin in blood. carbon monoxide

(b) (i) Describe how sulfur dioxide causes the formation of acid rain.
Sulfur dioxide dissolves in water to form sulfurous acid (H2SO3). In the presence of oxygen in the

air, sulfurous acid is slowly oxidised to sulfuric acid (H2SO4).

130 Worksheet 16 © 2013 Marshall Cavendish International (Singapore) Private Limited

(S)CMWB_16.indd 130 8/15/12 3:15 PM


(ii) Write balanced equations for the reactions in (b)(i).
SO2(g) + H2O(l) H2SO3(aq)

2H2SO3(aq) + O2(g) 2H2SO4(aq)

2. Hydrocarbons can get into the air from the incomplete combustion of fuels such as petrol,
kerosene or diesel. The table below shows the concentration of hydrocarbons in the air in a big
city from 1984 to 1995. The concentration is measured in parts per billion (ppb).

Year 1984 1985 1986 1987 1988 1989


Concentration/ppb 1630 1690 1650 1670 1660 1550

Year 1990 1991 1992 1993 1994 1995


Concentration/ppb 1560 1520 1450 1480 1480 1460

(a) Plot a graph based on the data provided in the table.

Concentration of
hydrocarbons/ppb

1700

1600

1500

1400
1984

1985

1986

1987

1988

1989

1990

1991

1992

1993

1994

1995

Year

(b) Cars are now manufactured with catalytic converters to reduce air pollution.
(i) What are the catalysts used in catalytic converters?

Platinum and rhodium

(ii) Based on your graph in (a), deduce the year in which it became compulsory for cars to
be fitted with catalytic converters. Explain your answer.
1989. There was a sharp drop in the concentration of hydrocarbons in the air in 1989.

© 2013 Marshall Cavendish International (Singapore) Private Limited Worksheet 16 131

(S)CMWB_16.indd 131 8/15/12 3:15 PM


(c) Unburnt hydrocarbons contain mainly octane, C8H18. The following reaction occurs when
unburnt hydrocarbons pass through a catalytic converter.
C8H18(g) + xO2(g) yCO2(g) + 9H2O(g)
(i) What are the values of x and y?
x = 12.5 or 25; y = 8
2

(ii) Calculate the volume of carbon dioxide produced at r.t.p. when 285 g of octane is passed
through the catalytic converter.
Mr of C8H18 = (8 × 12) + (18 × 1)
= 114
Number of moles of C8H18 = 285
114
= 2.5 mol
C8H18(g) + 12.5O2(g) 8CO2(g) + 9H2O(g)
From the equation, 1 mol of C8H18 produces 8 mol of CO2.
Number of moles of CO2 produced = 8 × 2.5
= 20 mol
1.0 mol of any gas occupies 24 dm3 at r.t.p.
Volume of CO2 produced = 20 × 24
= 480 dm3

Section C: Free-Response Questions


Answer the following questions.

1. (a) “Ozone is a protector and a pollutant.” Comment on this statement.


It is true that ozone is a protector. In the upper atmosphere, ozone filters out some of the harmful

ultraviolet radiation from the Sun. It is also true that ozone is a pollutant. In the lower atmosphere,

ozone forms photochemical smog which irritates the eyes and lungs and causes breathing difficulties.

(b) The structural formula of Freon-113, a chlorofluorocarbon, is shown below.


Cl F
F C C F
Cl Cl

132 Worksheet 16 © 2013 Marshall Cavendish International (Singapore) Private Limited

(S)CMWB_16.indd 132 8/15/12 3:15 PM


(i) Describe how Freon-113 causes the depletion of the ozone layer.

In the presence of sunlight, Freon-113 decomposes to form chlorine atoms. Chlorine atoms

react with ozone molecules to form chlorine oxide and oxygen, thus destroying the ozone layer.

(ii) Explain why the depletion of the ozone layer is harmful to humans.
As the ozone layer filters some of the harmful UV radiation from the Sun, the depletion of the

ozone layer will cause more UV radiation to reach the Earth’s surface. Too much UV radiation

can cause skin cancer, genetic mutations and eye damage.

2. (a) Explain how the processes in the carbon cycle maintain the level of carbon dioxide in the
atmosphere.

In the carbon cycle, respiration, combustion of fuels, and decay and bacterial decomposition release

carbon dioxide into the atmosphere, while photosynthesis and ocean uptake remove carbon dioxide

from the atmosphere.

(b) How is global warming related to the greenhouse effect?

Greenhouse gases such as carbon dioxide trap some of the infrared radiation from the Sun. Heat

energy is thus retained in the atmosphere. This produces a warming effect called the greenhouse

effect. The rapid build-up of greenhouse gases increases the greenhouse effect. This leads to an

increase in the Earth’s average temperature, a phenomenon known as global warming.

(c) State three effects of global warming.


The rapid evaporation of water from the Earth’s surface; the melting of large quantities of ice in the

North and South Poles, which causes the rising of ocean levels and flooding in low-lying countries;

a decrease in crop yields due to droughts

© 2013 Marshall Cavendish International (Singapore) Private Limited Worksheet 16 133

(S)CMWB_16.indd 133 8/15/12 3:15 PM


(d) Describe the effects of large-scale cutting down of forests on global warming.

Large-scale cutting down of forests reduces the number of trees. As a result, carbon dioxide is

being added to the atmosphere more quickly than photosynthesis can remove the excess gas. This

increased amount of carbon dioxide traps more heat in the Earth’s atmosphere and causes global

warming.

WORK ON IT!
In the table below, circle the question(s) that you have answered incorrectly. Revisit the
relevant section(s) in the textbook to strengthen your understanding of the key concept(s).
Question number(s) Textbook
Learning outcomes Multiple- Free- section(s)
Structured to revise
choice response
(a) Describe the composition by volume of
1 – – 20.1
gases present in dry air.
(b) Name some common air pollutants and
2 1(a) – 20.2
state their sources.
(c) Describe the effects of some air
pollutants on health and the 2 1(a)(ii), 1(b) 1(a) 20.2
environment.
(d) Describe the reactions involved in
catalytic converters to remove – 2(a), 2(b) – 20.3
combustion pollutants.
(e) Describe the reactions used to reduce
the effect of acid rain and in flue gas 3 – – 20.3
desulfurisation.
(f) Discuss the importance of the ozone
4 – 1(a) 20.4
layer.
(g) Discuss the problems involved with the
4 1(a)(ii) 1(b) 20.4
depletion of the ozone layer.
(h) Describe the basic processes of the
– – 2(a) 20.5
carbon cycle.
(i) State that greenhouse gases such as
carbon dioxide and methane may 5 – 2(b), 2(d) 20.5
contribute to global warming.
(j) Discuss the possible effects of an
– – 2(c) 20.5
increase in global warming.
(k) Write balanced chemical equations
– 1(b)(ii), 2(c)(i) – 8.1
with state symbols.
(l) Calculate the volumes of gaseous
reactants and products using the mole – 2(c)(ii) – 10.2
ratio in a chemical equation.

134 Worksheet 16 © 2013 Marshall Cavendish International (Singapore) Private Limited

(S)CMWB_16.indd 134 8/15/12 3:15 PM


Name: ( )

Worksheet 17
Class: Date:

An Introduction to Organic Chemistry

for Chapter 21
ANSWER IT RIGHT!
Go through the worked example. Use the tips to guide you in understanding and answering
the questions.

The table below shows the boiling point ranges of petroleum gas and petrol.

Substance Boiling point range/°C


petroleum gas below 40
petrol 40–75

(a) Explain why petroleum gas and petrol do not boil at a fixed temperature.

Thought Process
Only pure substances have fixed boiling points.

Answer
Petroleum gas and petrol are mixtures of hydrocarbons and not pure substances. Hence, they
boil over a range of temperatures.

(b) State the uses of petroleum gas and petrol.

Answer
Petroleum gas and petrol are used as fuels. Petroleum gas is used as a fuel for cooking and
heating. Petrol is used as a fuel in cars.

(c) Suggest a method to distinguish between petroleum gas and petrol.

• The term ‘suggest’ implies that there is no unique answer to the question.
• There is more than one way to distinguish between petroleum gas and petrol.

Thought Process
• We can use a chemical or physical method to distinguish between two substances.
• As petroleum gas and petrol are chemically similar (both are hydrocarbons), they can be
distinguished by a physical method.

Answer
Petroleum gas and petrol can be distinguished by their boiling points. Petroleum gas has a lower
boiling point range than petrol.

© 2013 Marshall Cavendish International (Singapore) Private Limited Worksheet 17 135

(S)CMWB_17.indd 135 8/15/12 4:16 PM


TEST IT!

Section A: Multiple-Choice Questions


Choose the correct answer and write its letter in the brackets provided.

1. Which of the following statements about a homologous series are correct?


1 All members undergo similar chemical reactions.
2 All members have the same functional group.
3 All members have the same physical properties.
A 1 and 2
B 1 and 3
C 2 and 3
D 1, 2 and 3 ( A )

2. Which of the following pairs of compounds belongs to the same homologous series?
A H H H H H H H
H C C C H H C C C C H
H H H
B H H H H H H H
H C C C H H C C C C H
H H H H H
C H H H O
H C C O H H C C O H
H H H
D H O H O H
H C C O H H C C O C H
H H H ( A )

3. Which of the following statements about natural gas is correct?


A It is obtained by the fractional distillation of crude oil.
B It is a renewable source of energy.
C It consists mainly of methane gas.
D It produces large amounts of soot on combustion. ( C )

136 Worksheet 17 © 2013 Marshall Cavendish International (Singapore) Private Limited

(S)CMWB_17.indd 136 8/15/12 4:16 PM


4. The fractional distillation of a sample of crude oil produces fractions A to D. The boiling point
ranges of the fractions are given below. Which fraction has the largest molecules?

Fraction Approximate boiling point range/°C

A 20–40

B 40–120

C 120–160

D 160–250
( D )
5. Which statement about the fractional distillation of petroleum is correct?
A Liquid petroleum is pumped into the fractionating column.
B The petroleum fraction collected at the top of the column has the largest relative molecular
mass.
C The petroleum fraction collected at the bottom of the column has the lowest range of boiling
points.
D Each petroleum fraction collected is a mixture of compounds. ( D )

6. Which of the following petroleum fractions is correctly matched to its use?


Fraction Use
A bitumen as feedstock for the petrochemical industry
B naphtha for making roads
C lubricating oil for making polishes and waxes
D diesel oil as jet fuel ( C )

Section B: Structured Questions


Answer the following questions in the spaces provided.

1. (a) What is meant by ‘hydrocarbons’?


Hydrocarbons are organic compounds that contain only hydrogen and carbon.

(b) 3.0 g of a hydrocarbon was found to contain 2.4 g of carbon. What is the empirical formula of
the hydrocarbon?
Mass of hydrogen in the hydrocarbon = 3.0 – 2.4
= 0.6 g

Element Carbon Hydrogen


Mass/g 2.4 0.6
Relative atomic mass 12 1
Number of moles/mol 2.4 = 0.2 0.6 = 0.6
12 1
Mole ratio 1 3

∴ The empirical formula of the hydrocarbon is CH3.

© 2013 Marshall Cavendish International (Singapore) Private Limited Worksheet 17 137

(S)CMWB_17.indd 137 8/15/12 4:16 PM


2. Chloromethane is the first member of the homologous series of chloroalkanes. The table below
gives information about chloromethane and the next two members.
(a) Deduce the full structural formula of chlorobutane and predict its physical properties.

Name Formula Boiling point/°C Solubility in water


H
chloromethane H C Cl –23.8 insoluble
H
H H
chloroethane H C C Cl 12.5 insoluble
H H
H H H
chloropropane H C C C Cl 46.6 insoluble
H H H
H H H H
chlorobutane H C C C C Cl > 46.6 insoluble
H H H H

(b) When chloromethane is boiled with sodium hydroxide solution, methanol is obtained.
H H
H C Cl + NaOH H C O H + NaCl
H H
Chloromethane Methanol

Deduce the name of the organic product obtained when chloropropane is boiled with sodium
hydroxide solution. Write an equation for this reaction, showing the full structural formulae of
the organic compounds.
H H H H H H
H C C C Cl + NaOH H C C C O H + NaCl
H H H H H H
Propanol

3. The pie charts below show the energy sources used by countries A and B.
natural gas
hydropower
natural biomass
coal gas coal

nuclear power
crude
oil
hydro- crude
power oil
biomass

Country A Country B

138 Worksheet 17 © 2013 Marshall Cavendish International (Singapore) Private Limited

(S)CMWB_17.indd 138 8/15/12 4:16 PM


(a) Name the renewable and non-renewable sources of energy used by country A.

Renewable sources: Hydropower, biomass

Non-renewable sources: Coal, natural gas, crude oil

(b) Which country, A or B, is a better model for Singapore to follow in terms of the types of energy
sources used? Give two reasons to justify your answer.

Country A. It uses more renewable energy sources (hydropower and biomass) and fewer

non-renewable energy sources (coal, natural gas and crude oil) than country B.

Section C: Free-Response Question


Answer the following question.

1. The diagram below represents a fractionating column where petroleum is separated into various
fractions.

petroleum
vapour Z

(a) (i) Explain how petroleum fractions are separated in the fractionating column.

Petroleum is heated into a vapour in the furnace and passed into the fractionating column.

As the hot vapour rises up the column, it begins to cool and condense. Petroleum fractions

with lower boiling points are collected at the top of the column as gases and those with higher

boiling points are collected at the lower sections of the column.

© 2013 Marshall Cavendish International (Singapore) Private Limited Worksheet 17 139

(S)CMWB_17.indd 139 8/15/12 4:16 PM


(ii) Match X, Y and Z with the petroleum fractions naphtha, kerosene and lubricating oil.

X: Naphtha Y: Kerosene Z: Lubricating oil

(b) (i) Can simple distillation be used to separate petroleum into various fractions? Why?

No. It is difficult to separate mixtures of liquids with boiling points that are close to one another

by simple distillation.

(ii) State the main difference between the apparatus used for fractional distillation and
simple distillation.
A fractionating column is used in fractional distillation but not in simple distillation.

(c) Describe the issues relating to the competing uses of petroleum as an energy source and
chemical feedstock.

About 90% of all the petroleum produced is used as fuel to generate heat and electricity. Another 10%

is used as chemical feedstock for the manufacture of petrochemicals and medicines. Petroleum is a

non-renewable resource and the Earth’s petroleum reserves are finite. With the supply of petroleum

decreasing rapidly, there is a growing need for its conservation.

WORK ON IT!
In the table below, circle the question(s) that you have answered incorrectly. Revisit the
relevant section(s) in the textbook to strengthen your understanding of the key concept(s).
Question number(s) Textbook
Learning outcomes Multiple- Free- section(s)
Structured to revise
choice response
(a) Define a homologous series. 1, 2 1(a), 2 – 21.1
(b) Describe petroleum as a mixture of
5 – – 21.2
hydrocarbons.
(c) Name natural gas and petroleum as
3 3(a) – 21.2
sources of energy.
(d) Describe the fractional distillation of
4, 5 – 1(a)(i), 1(b) 21.2
petroleum.
(e) Name the petroleum fractions and
6 – 1(a)(ii) 21.2
state their uses.
(f) Describe the issues related to the
– 3(b) 1(c) 21.3
competing uses of petroleum.
(g) Determine the empirical and
molecular formulae of a compound – 1(b) – 9.5
from given data.

140 Worksheet 17 © 2013 Marshall Cavendish International (Singapore) Private Limited

(S)CMWB_17.indd 140 3/10/16 10:17 AM


Name: ( )

Worksheet 18
Class: Date:

Alkanes and Alkenes

for Chapter 22
ANSWER IT RIGHT!
Go through the worked example. Use the tips to guide you in understanding and answering
the questions.

(a) What is meant by ‘unsaturated hydrocarbons’?

• When asked to define a term consisting of two words, you have to define both words.
• In this case, ‘hydrocarbons’ as well as ‘unsaturated’ has to be defined.

Answer
Hydrocarbons are organic compounds containing only the elements carbon and hydrogen.
Unsaturated hydrocarbons are hydrocarbons that contain one or more carbon–carbon double bonds.

(b) The structural formulae of six hydrocarbons are shown below.

A B C
CH2 CH2
H H H H H
CH2 CH
H C C C C C H CH2 CH2
CH2 CH
H H H CH2 CH2
CH2

D E F
H H
H H H H H H C H H C H
H C C C C C H H H H H H H
H H H H H H C C C C H H C C C C H
H H H H H H

(i) Classify these hydrocarbons as saturated or unsaturated hydrocarbons.

• ‘Classify’ means putting things in groups based on common characteristics.


• In this case, the common characteristic to consider is whether the hydrocarbons contain the C=C bond.

(ii) Which of these hydrocarbons are isomers?

© 2013 Marshall Cavendish International (Singapore) Private Limited Worksheet 18 141

(S)CMWB_18.indd 141 8/15/12 4:20 PM


Thought Process
(i) Saturated hydrocarbons do not contain C=C bonds, whereas unsaturated hydrocarbons
contain one or more C=C bonds.
(ii) Isomers are compounds with the same molecular formula but different structural formulae.

Answer
(i) Saturated hydrocarbons: C, D and E
Unsaturated hydrocarbons: A, B and F
(ii) A, C and F; D and E

Note that isomers need not belong to the same homologous series. For example, A is an alkene, whereas
C is a cycloalkane.

(c) B is called cyclohexene. It is a hydrocarbon with a cyclic structure. Deduce the molecular and
structural formulae of cyclohexane.

Thought Process
‘Cyclo’ suggests a cyclic structure. ‘Hexane’ refers to a saturated hydrocarbon with six carbon
atoms per molecule.

Answer
Molecular formula: C6H12
Structural formula: CH2
CH2 CH2

CH2 CH2
CH2

TEST IT!

Section A: Multiple-Choice Questions


Choose the correct answer and write its letter in the brackets provided.

1. Which statement about methane and ethane is correct?


A They are liquids at room temperature.
B They are unsaturated hydrocarbons.
C They have the general formula CnH2n.
D They contain carbon and hydrogen only. ( D )

2. Alkanes P and Q have the molecular formulae CxH12 and C14Hy respectively. What are the relative
molecular masses of P and Q?
Relative molecular mass of P Relative molecular mass of Q
A 70 194
B 70 196
C 72 198
D 72 200 ( C )

142 Worksheet 18 © 2013 Marshall Cavendish International (Singapore) Private Limited

(S)CMWB_18.indd 142 8/15/12 4:20 PM


3. Alkenes L and M are isomers. Which statement about L and M is incorrect?
A They have the same empirical formula.
B They have the same molecular formula.
C They contain the same percentage by mass of carbon.
D They have the same boiling point. ( D )

4. Octane (C8H18) and hexadecane (C16H34) are used as fuels. Which statement about these two
alkanes is correct?
A They are soluble in water.
B They have different general formulae.
C Octane is more viscous than hexadecane.
D Octane is more flammable than hexadecane. ( D )

5. Which of the following reactions is correctly matched with the type of reaction?
Reaction Type of reaction
1 C2H4 + H2O C2H5OH addition
2 C2H4 + Cl2 C2H4Cl2 substitution
3 C2H4 + 3O2 2CO2 + 2H2O redox
4 C8H18 C2H4 + C6H14 substitution
A 1 and 2
B 1 and 3
C 1, 3 and 4
D 2, 3 and 4 ( B )

6. Which statement about the reaction of ethene with hydrogen is correct?


A The reaction is catalysed by sunlight.
B The reaction is carried out at 60 atm.
C The product has a lower molecular mass than ethene.
D The product is an alkane. ( D )

7. The structural formula of 2-butene is shown below.


H H H H
H C C C C H
H H
2-Butene reacts with steam to form a product, Z. What is the structural formula of Z?
A H H H H B H H H H
H C C C C H H C C C C O H
H H H H H H H H

C H H H H D H H H H
H C C C C H H C C C C H
H H OH H H OH OH H ( C )

© 2013 Marshall Cavendish International (Singapore) Private Limited Worksheet 18 143

(S)CMWB_18.indd 143 8/15/12 4:20 PM


*8. The alkene, C4H8, reacts with bromine to form the following compound.
H
H C H
H H
H C C C Br
H Br H
What is the structural formula of the alkene?
A H H H H B H H H H
H C C C C H H C C C C H
H H H H

C H D H
H C H H C H
H H H H H
H C C C H H C C C C H
H H H ( C )

Section B: Structured Questions


Answer the following questions in the spaces provided.

1. The molecular formulae of six hydrocarbons are shown below.

C6H14 C7H14 C7H16 C8H16 C9H20 C10H20

(a) Which of these hydrocarbons are alkanes?

C6H14, C7H16, C9H20

(b) Which of the alkanes in (a)

(i) has the highest boiling point;


C9H20

(ii) gives out the least heat (per mole) on complete combustion;

C6H14

(iii) burns with the smokiest flame?


C9H20

144 Worksheet 18 © 2013 Marshall Cavendish International (Singapore) Private Limited

(S)CMWB_18.indd 144 8/15/12 4:20 PM


(c) 20 cm3 of C8H16 was burnt in 300 cm3 of oxygen.
(i) Write a balanced chemical equation for this reaction.

C8H16(l) + 12O2(g) 8CO2(g) + 8H2O(g)

(ii) What is the volume of oxygen that remained unreacted at the end of the reaction?
(All volumes are measured under the same conditions.)
From the above equation, 1 mol of C8H16 reacts with 12 mol of O2.
Volume of oxygen reacted = 12 × volume of C8H16
= 12 × 20
= 240 cm3
Volume of oxygen that remained unreacted = 300 – 240
= 60 cm3

2. (a) (i) Draw the full structural formula of butane.


H H H H
H C C C C H
H H H H

(ii) Draw the full structural formula of the isomer of butane.


H H H
H C C C H
H H
H C H
H

(b) On complete combustion, 1.0 mol of alkane P produces 3.0 mol of carbon dioxide. P reacts
with chlorine under UV light to produce Q and hydrogen chloride. The relative molecular mass
of Q is 78.5.
(i) What is the molecular formula of alkane P? Explain how you arrived at your answer.
The general formula of an alkane is CnH2n+2. Since alkane P contains three carbon atoms per

molecule, the molecular formula of P is C3H(2×3)+2, i.e. C3H8.

(ii) Show that the molecular formula of Q is C3H7Cl.


The equation for the reaction of P, C3H8, with chlorine under UV light is:
UV light
C3H8 + Cl2 C3H7Cl + HCl
Relative molecular mass of C3H7Cl = (3 × 12) + (7 × 1) + 35.5
= 78.5
Thus, the molecular formula of Q is C3H7Cl, as confirmed by the equation and relative molecular
mass given.

© 2013 Marshall Cavendish International (Singapore) Private Limited Worksheet 18 145

(S)CMWB_18.indd 145 8/15/12 4:20 PM


(c) Dichloromethane, CH2Cl2, is a volatile liquid. It is used as a solvent to dissolve a wide range
of organic compounds. In the food industry, it is used to remove caffeine, a stimulant found
in coffee. Dichloromethane undergoes a photochemical substitution reaction with chlorine
gas to form trichloromethane.
(i) Why is the reaction described as a ‘photochemical substitution reaction’?
The reaction takes place only in the presence of ultraviolet light/sunlight and a hydrogen atom in

the dichloromethane molecule is substituted by a chlorine atom.

(ii) Write an equation for this reaction.


UV light
CH2Cl2(l) + Cl2(g) CHCl3(l) + HCl(g)

3. An experiment is carried out in the school laboratory using the apparatus as shown below.

broken porous pot


containing aluminium oxide
mixture of gases

glass wool soaked


in liquid petroleum

heat

water

A petroleum fraction containing a mixture of hydrocarbons, C12H26 and C18H38, is used for the
experiment. The equations for the reactions that take place are as follows:
C12H26 C6H12 + C4H8 + X
C18H38 C10H20 + C8H16 + Y

(a) (i) Name the above process and state its purpose.
Cracking. It breaks down hydrocarbons with long carbon chains into smaller molecules.

(ii) What is the application of this process in the industry?

To produce more useful fractions such as petrol, ethene and hydrogen, which are in high

demand

(b) Identify X and Y in each of the above equations.


X is ethane, C2H6. Y is hydrogen, H2.

146 Worksheet 18 © 2013 Marshall Cavendish International (Singapore) Private Limited

(S)CMWB_18.indd 146 8/15/12 4:20 PM


*4. The flow chart below shows the reactions of a hydrocarbon, L.

+ Cl2(g)
hydrocarbon M C3H7Cl + HCl
UV light

+ H2 200°C, Ni

+ steam
colourless liquid P hydrocarbon L

+ excess O2

colourless gas Q colourless gas R

(a) Identify L and M and draw their full structural formulae.

(i) L: Propene (ii) M: Propane

Full structural formula of L: Full structural formula of M:


H H H H H H
H C C C H H C C C H
H H H H

(b) (i) Write an equation for the formation of P from L, showing the full structural formulae of P
and L.
H H H H H H
H C C C H + H2O H C C C H
H H OH H
L P

(ii) State the conditions for the reaction.

Temperature: 300°C; pressure: 60 atm; catalyst: phosphoric(V) acid

(c) At room temperature, Q is a gas and R is a liquid. Identify Q and R.


Q is carbon dioxide and R is water.

© 2013 Marshall Cavendish International (Singapore) Private Limited Worksheet 18 147

(S)CMWB_18.indd 147 8/15/12 4:20 PM


Section C: Free-Response Questions
Answer the following questions.

1. The table below shows the boiling points (in kelvin) of five hydrocarbons. (The kelvin (K) is the S.I.
unit for temperature.)

Hydrocarbon A B C D E
Number of carbon atoms per molecule 2 3 4 5 6
Boiling point/K 160 250 267 303 337

(a) Plot a graph of boiling point against number of carbon atoms for these hydrocarbons.

Boiling point/K

350

300

250

200

150

100
2 3 4 5 6
Number of carbon atoms per molecule

(b) (i) Four of the five hydrocarbons are alkenes. Which hydrocarbon is not an alkene?

(ii) Explain your answer in (i).


The boiling points of the members of a homologous series gradually increase with the number of

carbon atoms in each molecule. Only B does not fall within the graph, hence it is not an alkene.

148 Worksheet 18 © 2013 Marshall Cavendish International (Singapore) Private Limited

(S)CMWB_18.indd 148 8/15/12 4:20 PM


(c) The boiling point of an alkene is 160 K. Calculate the relative molecular mass of the product
obtained when this alkene reacts with excess aqueous bromine.
From the table given, the alkene is ethene, C2H4.
C2H4 + Br2 C2H4Br2
Relative molecular mass of the product, C2H4Br2 = (2 × 12) + (4 × 1) + (2 × 80)
= 188

(d) The mass of 0.125 mol of an alkene is 7.0 g.


(i) Identify this alkene.

Relative molecular mass of the alkene = 7.0


0.125
= 56
The general formula of an alkene is CnH2n.
12n + 2n = 56
14n = 56
n =4
There are four carbon atoms in each molecule of the alkene.
∴ The alkene is butene.

(ii) What is the boiling point of this alkene?

267 K

2. (a) What is meant by ‘polyunsaturated’ when applied to food products?

It means that the food products contain polyunsaturated fats and oils. The hydrocarbon chains in

these fats and oils contain more than one carbon–carbon double bond.

(b) Describe how margarine is produced.

Margarine is produced by the addition of hydrogen to unsaturated vegetable oil (hydrogenation).

A temperature of 200°C and a nickel catalyst are needed for hydrogenation to occur.

© 2013 Marshall Cavendish International (Singapore) Private Limited Worksheet 18 149

(S)CMWB_18.indd 149 8/15/12 4:20 PM


(c) The structure of a fat molecule is represented as follows:

C17H27 rest of the fat molecule




hydrocarbon
chain

Is the fat molecule saturated or unsaturated? Explain your deduction.


Assuming that the fat molecule is saturated, the formula of its hydrocarbon chain should be

C17H(2×17)+1, that is, C17H35. However, the hydrocarbon chain has the formula C17H27, which is eight

hydrogen atoms fewer than that of a saturated hydrocarbon chain. This implies that the hydrocarbon

chain contains four C=C bonds and the fat molecule is unsaturated.

*3. Dienes are alkenes with two carbon–carbon double bonds. An example of a diene is butadiene,
which has the molecular formula C4H6.
(a) Draw the full structural formula of butadiene.

H H H H
H C C C C H

(b) (i) Compare butadiene and butane in terms of structure and reactivity.
Both butadiene and butane contain only carbon and hydrogen. Butadiene contains

carbon–carbon double bonds, whereas butane contains only carbon–carbon single bonds.

Butane is generally unreactive, whereas butadiene is very reactive.

(ii) Suggest a method to distinguish between butadiene and butane. In each case, describe
what would be observed and write an equation for any reaction that occurs.

Aqueous bromine can be used. When butadiene is added to aqueous bromine, the
reddish-brown bromine solution decolourises immediately.

H H H H H H H H
H C = C C = C H + 2Br2 H C C C C H
Br Br Br Br

H H H H H H H H
(Or H C = C C = C H + Br2 H C C C = C H)
Br Br

When butane is added to aqueous bromine, the reddish-brown solution remains unchanged.

150 Worksheet 18 © 2013 Marshall Cavendish International (Singapore) Private Limited

(S)CMWB_18.indd 150 8/15/12 4:20 PM


(c) Student A predicts that butadiene forms butene during hydrogenation, whereas student B
predicts the product to be butane. Both their predictions are correct.
(i) What is meant by ‘hydrogenation’?
Hydrogenation is the addition of a hydrogen molecule to a carbon–carbon double bond.

(ii) Explain the two different predictions made by students A and B. Write equations to
support your explanation.

Student A’s prediction is based on the partial hydrogenation of butadiene, i.e. only one of the
C=C bonds is hydrogenated.

H H H H H H H H
H C C C C H + H2 H C C C C H
H H
Butene

Student B’s prediction is based on the complete hydrogenation of butadiene, i.e. both C=C
bonds are hydrogenated.

H H H H H H H H
H C C C C H + 2H2 H C C C C H
H H H H
Butane

(d) A 20 cm3 gaseous mixture contains methane and butadiene. On complete combustion,
44 cm3 of carbon dioxide is produced.

(i) Write balanced chemical equations for the complete combustion of methane and
butadiene.
CH4(g) + 2O2(g) CO2(g) + 2H2O(g)

2C4H6(g) + 11O2(g) 8CO2(g) + 6H2O(g)

(ii) Calculate the volumes of methane and butadiene in the gaseous mixture.
Let the volume of CH4 be v cm3.
Thus, volume of C4H6 = (20 – v) cm3
From the equations,
1 volume of CH4 produces 1 volume of CO2 and 1 volume of C4H6 produces 4 volumes
of CO2.
Volume of CO2 produced by methane = v cm3
Volume of CO2 produced by butadiene = 4(20 – v) cm3
Total volume of CO2 produced = 44
v + 4(20 – v) = 44
v + 80 – 4v = 44
3v = 36
v = 12
Volume of CH4 = 12 cm3
Volume of C4H6 = 20 – 12
= 8 cm3
© 2013 Marshall Cavendish International (Singapore) Private Limited Worksheet 18 151

(S)CMWB_18.indd 151 8/15/12 4:20 PM


WORK ON IT!
In the table below, circle the question(s) that you have answered incorrectly. Revisit the
relevant section(s) in the textbook to strengthen your understanding of the key concept(s).

Question number(s) Textbook


Learning outcomes Multiple- Free- section(s)
Structured to revise
choice response
(a) Describe the alkanes as a homologous
series of saturated hydrocarbons with 1, 2 1(a), 2(b)(i) – 22.1
the general formula CnH2n+2.
(b) Name the first four alkanes. – 4(a)(ii) – 22.1
(c) Describe the characteristics of the
– 1(b) – 22.1
alkane homologous series.
(d) Draw the structures of the first four 2(a)(i),
– – 22.1
branched and unbranched alkanes. 4(a)(ii)
(e) Define isomerism and identify isomers. 3 2(a)(ii) – 22.1
(f) Explain why alkanes show a gradation
in their physical properties down the 4 1(b) – 22.2
homologous series.
(g) Describe the chemical properties of 1(c)(i),
– 3(d)(i) 22.2
alkanes. 2(b)(ii), 2(c)
(h) Describe the alkenes as a homologous
1(a), 1(b),
series of unsaturated hydrocarbons with – – 22.3
3(a)
the general formula CnH2n.
(i) Name the first three alkenes. – 4(a)(i) 1(d) 22.3
(j) Describe the characteristics of the
– – 1(b) 22.3
alkene homologous series.
(k) Draw the structures of the first three
7, 8 4(a)(i), 4(b)(i) – 22.3
branched and unbranched alkenes.
(l) Describe the chemical properties 1(c), 3(c),
5, 6, 7 4(b), 4(c) 22.4
of alkenes. 3(d)(i)
(m) Describe the manufacture of alkenes
– 3(a)(i), 3(b) – 22.5
and hydrogen.
(n) Explain the importance of cracking. – 3(a)(ii) – 22.5
(o) Describe the differences between
saturated and unsaturated – – 3(b) 22.6
hydrocarbons.
(p) Define the term ‘polyunsaturated’ when
– – 2(a), 2(c) 22.7
applied to food products.
(q) Describe the manufacture of margarine. – – 2(b) 22.7
(r) Write balanced chemical equations with 1(c)(i),
– 3(d)(i) 8.1
state symbols. 2(c)(ii)
(s) Calculate relative molecular mass or
– – 1(c), 1(d)(i) 9.2
relative formula mass of a substance.
(t) Calculate the volumes of gaseous
reactants and products using the mole – 1(c)(ii) 3(d)(ii) 10.2
ratio in a chemical equation.

152 Worksheet 18 © 2013 Marshall Cavendish International (Singapore) Private Limited

(S)CMWB_18.indd 152 8/15/12 4:20 PM


Name: ( )

Worksheet 19
Class: Date:

Alcohols and Carboxylic Acids

for Chapter 23
ANSWER IT RIGHT!
Go through the worked example. Use the tips to guide you in understanding and answering the
questions.

(a) (i) Outline the steps involved for the conversion of glucose to ethanoic acid. Write equations
for all the reactions involved.

The term ‘outline’ means to give a brief answer without going into details.

(ii) Describe and explain the observation when copper(II) oxide is added to ethanoic acid. Write
an equation for the reaction.

Thought Process
(i) Ethanoic acid is obtained from the oxidation of ethanol, which is produced by the fermentation
of glucose.
(ii) Copper(II) oxide is a base. It reacts with ethanoic acid to form a salt and water.

Answer
(i) Step 1: Fermentation of glucose to form ethanol
Mix a glucose solution with yeast and keep the mixture at about 37°C for a few days.
Ethanol is produced.
yeast
C6H12O6(aq) 2C2H5OH(aq) + 2CO2(g)
37°C

Step 2: Oxidation of ethanol to ethanoic acid


Heat the ethanol obtained in step 1 with a mixture of potassium manganate(VII)
solution and dilute sulfuric acid. Ethanol is oxidised to ethanoic acid.
C2H5OH(aq) + 2[O] CH3COOH(aq) + H2O(l)
(ii) Copper(II) oxide dissolves in ethanoic acid to form a blue solution of copper(II) ethanoate
and water.
CuO(s) + 2CH3COOH(aq) (CH3COO)2Cu(aq) + H2O(l)

• Take note of the charge on the metal ion when you write the formulae of the salts of carboxylic acids.
• For example, copper exists as Cu2+ ions in copper(II) salts. Thus, the formula of copper(II) ethanoate is
(CH3COO)2Cu, not CH3COOCu.

© 2013 Marshall Cavendish International (Singapore) Private Limited Worksheet 19 153

(S)CMWB_19.indd 153 8/15/12 4:24 PM


(b) Determine the relative molecular mass of a carboxylic acid containing six carbon atoms per
molecule.

The term ‘determine’ implies that the required quantity cannot be measured directly, but is obtained
through calculation using a formula.

Thought Process
Use the general formula CnH2n+1COOH to determine the molecular formula of the carboxylic acid
with six carbon atoms per molecule.

Answer
The general formula of a carboxylic acid is CnH2n+1COOH.
Substituting n = 5 into the formula, we have C5H(2×5)+1COOH, that is, C5H11COOH.
Thus, the molecular formula of the acid is C6H12O2.
Relative molecular mass of C6H12O2 = (6 × 12) + (12 × 1) + (2 × 16) = 116

• When counting the number of carbon atoms in a carboxylic acid, take note to include the carbon atom
of the –COOH group.
• In this case, the carboxylic acid with six carbon atoms is C5H11COOH, not C6H13COOH.

(c) 2.22 g of a carboxylic acid required 15.0 cm3 of 2.0 mol/dm3 aqueous sodium hydroxide for
complete reaction. Identify the carboxylic acid and draw its full structural formula.

When asked to draw the full structural formula or displayed formula, you must show all the bonds
between the atoms in the molecule.

Answer 3
Number of moles of NaOH = concentration in mol/dm3 × volume in cm
1000
= 2.0 × 15.0
1000
= 0.03 mol

CnH2n+1COOH(aq) + NaOH(aq) CnH2n+1COONa(aq) + H2O(l)

From the equation, 1 mol of CnH2n+1COOH reacts with 1 mol of NaOH.


Number of moles of CnH2n+1COOH = number of moles of NaOH
= 0.03 mol
Mr of CnH2n+1COOH = mass
number of moles
= 2.22
0.03
= 74
12n + 2n + 1 + 12 + (2 × 16) + 1 = 74
14n = 28
n=2
The molecular formula of the acid is C2H5COOH.
Thus, the carboxylic acid is propanoic acid. Its full structural formula is:
H H O
H C C C O H
H H

154 Worksheet 19 © 2013 Marshall Cavendish International (Singapore) Private Limited

(S)CMWB_19.indd 154 8/15/12 4:24 PM


TEST IT!

Section A: Multiple-Choice Questions


Choose the correct answer and write its letter in the brackets provided.

1. Which organic compound does not belong to the same homologous series as the following
compound?
H OH H
H C C C H
H H H
A CH3OH B C5H11OH
C C6H11OH D C7H15OH ( C )

2. Which of the following compounds will be produced when wine is left exposed to the air for a period
of time?
A O B O
H C O H CH3 C O H
C O D H
C2H5 C O H CH3 C O H
H ( B )

3. Which of the following reactions does not produce carbon dioxide?


A Fermentation of glucose
B Combustion of ethanol
C Oxidation of propanol
D Reaction of ethanoic acid with sodium carbonate ( C )

4. The oxidation of ethanol produces ethanoic acid and water. In this reaction, one molecule of
ethanol .
1 loses one hydrogen atom
2 loses two hydrogen atoms
3 gains one oxygen atom
4 produces two molecules of water
A 1 and 2 B 1 and 4
C 2 and 3 D 2, 3 and 4 ( C )

5. A carboxylic acid, Z, has a relative molecular mass of 60. What is the next member after Z in the
homologous series?
A Methanoic acid B Ethanoic acid
C Propanoic acid D Butanoic acid ( C )

© 2013 Marshall Cavendish International (Singapore) Private Limited Worksheet 19 155

(S)CMWB_19.indd 155 8/15/12 4:24 PM


6. When compound P is heated with acidified potassium manganate(VII), compound Q is obtained.
The full structural formula of Q is shown below.
H H H O
H C C C C O H
H H H
Q
What are P and Q?
P Q
A propanol propanoic acid
B propanol butanoic acid
C butanol propanoic acid
D butanol butanoic acid ( D )

7. Compound X reacts with aqueous sodium carbonate to form a product with the molecular formula
HCO2Na. What is X?
A Methanol B Ethanol
C Methanoic acid D Ethanoic acid ( C )

8. The properties of an organic compound, R, are shown below.


• It reacts with aqueous sodium hydroxide.
• It reacts with sodium carbonate.
• It has no reaction with bromine solution.

What could be the structure of R?


A H H H
H C C C O H
H H H

B H H O
H C C C O H
H H

C H H H O
H C C C C O H
H

D H O H H
H C C O C C H
H H H
( B )

156 Worksheet 19 © 2013 Marshall Cavendish International (Singapore) Private Limited

(S)CMWB_19.indd 156 8/15/12 4:24 PM


Section B: Structured Questions
Answer the following questions in the spaces provided.

1. The structural formulae of six organic compounds are shown below.

A B C
H H H H H H H O H
H C O C C H H C C C O H H C C O C H
H H H H H H H H

D E F
H H O H H H O H H H O H
H C C C O H H C C C C O H H C C C C O C H
H H H H H H H

(a) Name the organic compounds B, C, D and E.

(i) B: Propanol (ii) C: Methyl ethanoate

(iii) D: Propanoic acid (iv) E: Butanoic acid

(b) Identify two pairs of isomers.

A and B; C and D

(c) Suggest a chemical test to distinguish between C and F.


Add aqueous bromine to each compound. C will have no reaction with aqueous bromine but F will

decolourise aqueous bromine.

(d) (i) Outline a method to obtain E in the laboratory.


Add a mixture of potassium manganate(VII) solution and dilute sulfuric acid to butanol and heat

the mixture. The organic compound produced is E (butanoic acid).

(ii) Write an equation for the reaction in (d)(i), showing the full structural formulae of all the
organic compounds.
H H H H H H H O
H C C C C O H + 2[O] heat H C C C C O H + H2O
H H H H H H H

© 2013 Marshall Cavendish International (Singapore) Private Limited Worksheet 19 157

(S)CMWB_19.indd 157 8/15/12 4:24 PM


2. (a) From the molecular formulae of the organic compounds below, draw the structural formulae
of these compounds and name them.

(i) CH4O (an alcohol) (iii) C2H4O (an ester)


H O H
H C O H esterification H C O C H
H H
methanol methyl methanoate

(ii) CH2O2 (a carboxylic acid) (iv) HCO2Na (a salt)

O + NaOH(aq) O
H C O H H C ONa
methanoic acid sodium methanoate

(b) (i) Is ethanoic acid a strong or weak acid? Explain your answer.
It is a weak acid as it ionises only partially to give hydrogen ions in aqueous solution.

(ii) Write an equation for the reaction between calcium carbonate and ethanoic acid.
CaCO3(s) + 2CH3COOH(aq) (CH3COO)2Ca(aq) + H2O(l) + CO2(g)

3. The flow chart below shows some reactions involving ethanol.

red solution

litmus
solution
yeast +X
colourless solution W ethanol colourless solution Y
dilute H2SO4,
heat concentrated
+ C3H7OH
H2SO4, heat

colourless solution Z

(a) Identify each of the following:

(i) W: Glucose (ii) X: Potassium manganate(VII) solution

(iii) Y: Ethanoic acid (iv) Z: Propyl ethanoate

(b) Draw the full structural formula of Z.


H H H O H
H C C C O C C H
H H H H

158 Worksheet 19 © 2013 Marshall Cavendish International (Singapore) Private Limited

(S)CMWB_19.indd 158 8/15/12 4:24 PM


4. (a) The structure of an organic compound, Q, is shown below.

H H H O H H
H C C C O C C C H
H H H H H

(i) Name the compound Q.


Propyl propanoate

(ii) Describe how you would synthesise Q in the school laboratory.

Mix propanol with propanoic acid and add a few drops of concentrated sulfuric acid to the

mixture. Heat the mixture in a water bath. Then pour the reaction mixture into a beaker of water.

Q (propyl propanoate) formed will float on top of the water.

(iii) Write an equation for the reaction in (a)(ii).


C3H7OH(aq) + C2H5COOH(aq) C2H5COOC3H7(aq) + H2O(l)

(b) State two commercial uses of esters.


They are used as solvents for cosmetics, perfumes and glues, and in the preparation of artificial food

flavourings.

Section C: Free-Response Questions


Answer the following questions.

1. (a) Ethanol can be produced by the hydration of ethene or the fermentation of sugar. Complete
the table below.

Reaction Hydration of ethene Fermentation of sugar

Ethene is obtained from the Sugar is obtained from


Source of raw materials
cracking of petroleum. sugarcane.

Catalyst for the reaction Phosphoric(V) acid, H3PO4 Yeast

Temperature and pressure 300°C, 60 atm 37°C, atmospheric pressure

Speed of reaction Fast Slow

© 2013 Marshall Cavendish International (Singapore) Private Limited Worksheet 19 159

(S)CMWB_19.indd 159 8/15/12 4:24 PM


(b) State one difference between the physical properties of ethene and ethanol.
Ethene is a gas at room temperature, but ethanol is a liquid at room temperature.
(Or: Ethene is insoluble in water but ethanol is soluble in water.)
(c) Perfumes and deodorants often contain large amounts of ethanol. Suggest two reasons why
ethanol is used in these products.

Ethanol is a good solvent (it dissolves many substances that are not soluble in water).

Ethanol is volatile (it evaporates easily at room temperature).

*2. Gasohol is used as car fuel in some countries such as Brazil, Canada and Thailand. It is a mixture
of liquid gasoline and ethanol. Gasoline consists mainly of octane, C8H18.
(a) Write equations for the complete combustion of ethanol and octane.
C2H5OH(aq) + 3O2(g) 2CO2(g) + 3H2O(g)

2C8H18(l) + 25O2(g) 16CO2(g) + 18H2O(g)

(b) Suggest a chemical test to distinguish between ethanol and octane.

Add acidified potassium manganate(VII) to each compound and heat. The solution containing

ethanol will turn from purple to colourless. Octane will have no reaction with acidified potassium

manganate(VII).

(c) A sample of gasohol vapour in the car engine contains 25% ethanol and 75% octane. What
is the volume of carbon dioxide produced on the complete combustion of 20 dm3 of gasohol
vapour?
Volume of ethanol in the sample = 25% × 20
= 5 dm3
Volume of octane in the sample = 75% × 20
= 15 dm3
From the equations in (a), 1 mol of C2H5OH produces 2 mol of CO2. 1 mol of C8H18 produces 8 mol
of CO2.
Volume of CO2 produced by C2H5OH = 2 × 5
= 10 dm3
Volume of CO2 produced by C8H18 = 8 × 15
= 120 dm3
Total volume of CO2 = 10 + 120
= 130 dm3

160 Worksheet 19 © 2013 Marshall Cavendish International (Singapore) Private Limited

(S)CMWB_19.indd 160 8/15/12 4:24 PM


*3. The table below gives information about the first four members of the carboxylic acid homologous
series.

Carboxylic acid Melting point/°C Boiling point/°C


methanoic acid 8 101
ethanoic acid 17 118
propanoic acid –21 141
butanoic acid –8 164

(a) State the trend in the melting and boiling points of the carboxylic acids down the series, if any.
The change in the melting points of the acids does not show a regular pattern while the boiling points

increase gradually down the series.

(b) (i) What is the relative molecular mass of pentanoic acid?


The molecular formula of pentanoic acid is C4H9COOH, i.e. C5H10O2.
Relative molecular mass of pentanoic acid = (5 × 12) + (10 × 1) + (2 × 16)
= 102

(ii) Predict the boiling point of pentanoic acid.


Increase in boiling point between carboxylic acids with three and four carbon atoms
= 164 – 141
= 23°C
Expected boiling point of the carboxylic acid = 164 + 23
= 187°C

(c) A sample of a carboxylic acid was analysed and found to contain 0.24 g of carbon, 0.04 g of
hydrogen and 0.16 g of oxygen.
(i) What is the empirical formula of this carboxylic acid?

Element C H O
Mass/g 0.24 0.04 0.16
Relative atomic mass 12 1 16
0.24 = 0.02 0.04 = 0.04 0.16 = 0.01
Number of moles/mol
12 1 16
Mole ratio 2 4 1

The empirical formula of the carboxylic acid is C2H4O.

(ii) Deduce the name and molecular formula of the carboxylic acid.
Butanoic acid, C4H8O2 (or C3H7COOH)

© 2013 Marshall Cavendish International (Singapore) Private Limited Worksheet 19 161

(S)CMWB_19.indd 161 8/15/12 4:24 PM


WORK ON IT!
In the table below, circle the question(s) that you have answered incorrectly. Revisit the relevant
section(s) in the textbook to strengthen your understanding of the key concept(s).

Question number(s) Textbook


Learning outcomes Multiple- Free- section(s)
Structured to revise
choice response
(a) Describe alcohols as a homologous series
1 – 1(b) 23.1
containing the –OH group.
(b) Name and draw the structures of the first
– 1(a)(i), 2(a)(i) – 23.1
four unbranched alcohols.
(c) Describe the combustion and oxidation 1(d), 3(a)(ii),
2, 3, 4, 6 2(a), 2(b) 23.2
reactions of alcohols. 3(a)(iii)
(d) Describe the formation of ethanol from the
catalytic addition of steam to ethene and the 3 3(a)(i) 1(a) 23.3
fermentation of glucose.
(e) State some uses of ethanol. – – 1(c) 23.4
(f) Describe carboxylic acids as a homologous
5 – 3(a), 3(b) 23.5
series containing the –COOH group.
(g) Name and draw the structures of the first 1(a)(iii), 1(a)(iv),
– 3(c)(ii) 23.5
four unbranched carboxylic acids. 2(a)(ii)
(h) Describe carboxylic acids as weak acids. – 2(b)(i) – 23.6
(i) Describe the reactions of carboxylic acids 2(a)(iv),
3, 7, 8 – 23.6
with reactive metals, carbonates and bases. 2(b)(ii)
(j) Describe the formation of ethanoic acid from
6 1(d), 3(a)(iii) – 23.7
the oxidation of ethanol.
(k) Describe the formation of esters from the 1(a)(ii), 2(a)(iii),
– – 23.8
reaction of carboxylic acids with alcohols. 3(a)(iv), 3(b), 4(a)
(l) State some uses of esters. – 4(b) – 23.8
(m) Write balanced chemical equations with
– 2(b)(ii) 2(a) 8.1
state symbols.
(n) Calculate relative molecular mass or relative
– – 3(b)(i) 9.2
formula mass of a substance.
(o) Determine the empirical and molecular
– – 3(c) 9.5
formulae of a compound from given data.
(p) Calculate the volumes of gaseous reactants
and products using the mole ratio in a – – 2(c) 10.2
chemical equation.
(q) Describe the properties of acids and their
– 3(a)(iii) – 11.1
reactions with metals, bases and carbonates.
(r) Define isomerism and identify isomers. – 1(b) – 22.1
(s) Describe the chemical properties of alkanes. – – 2(a), 2(b) 22.2
(t) Describe the alkenes as a homologous
series of unsaturated hydrocarbons with the – – 1(b) 22.3
general formula CnH2n.
(u) Describe the differences between saturated
– 1(c) – 22.6
and unsaturated hydrocarbons.

162 Worksheet 19 © 2013 Marshall Cavendish International (Singapore) Private Limited

(S)CMWB_19.indd 162 8/15/12 4:24 PM


Name: ( )

Worksheet 20
Class: Date:

Macromolecules

for Chapter 24
ANSWER IT RIGHT!
Go through the worked example. Use the tips to guide you in understanding and answering
the questions.

(a) An example of biodegradable plastic is poly(lactic acid) (PLA). It is produced from renewable
resources such as potato peels. The monomer of poly(lactic acid) is lactic acid. Potato peels are
first converted to glucose, which is then converted to lactic acid. The structural formula of lactic
acid is shown below.
CH3 O
HO C C OH
H
(i) What is the chemical reaction that occurs when lactic acid is converted to poly(lactic acid)?
(ii) Write an equation to show the polymerisation of lactic acid.
(iii) Suggest one advantage and one disadvantage of using biodegradable plastics.

Thought Process
(i) • Lactic acid does not contain any C=C bond. Therefore, it cannot undergo addition
polymerisation.
• Lactic acid contains two functional groups, the hydroxyl group (–OH group) and the carboxyl
group (–COOH group).
• Hence, lactic acid can undergo condensation polymerisation to form a polyester.

Answer
(i) Condensation polymerisation
(ii)
CH3 O CH3 O removal of H2O
molecules
HO C C OH + HO C C OH
H H
CH3 O CH3 O
O C C O C C
H H

(iii) Advantage: Disposal of biodegradable plastics does not cause land pollution as these plastics
can be decomposed by bacteria.
Disadvantage: These plastics are not durable as they decompose easily.

© 2013 Marshall Cavendish International (Singapore) Private Limited Worksheet 20 163

(S)CMWB_20.indd 163 3/10/16 10:18 AM


(b) (i) Nylon is used to make fishing lines. State two other uses of nylon.
(ii) State one disadvantage of using nylon for making fishing lines.
(iii) Nylon was invented as a substitute for silk, which was in short supply during World War II.
“Nylon is a better material for manufacturing items than silk.” Comment on this statement.

• ‘Comment’ is meant as an open-ended instruction.


• You need to recall or infer points of interest relevant to the context of the question.
• In this case, your answer could discuss whether nylon, a non-biodegradable material, is always better.

Thought Process
(iii) Nylon is a synthetic fibre that is non-biodegradable, while silk is a natural material that is
biodegradable.

Answer
(i) Nylon can be used in making parachutes and sleeping bags.
(ii) Nylon is non-biodegradable. Thus, fishing lines made of nylon can cause pollution, as they do
not decompose when thrown away.

• Do not simply write ‘nylon fishing lines cause pollution’ or ‘nylon fishing lines are dangerous to sea life’.
• You need to be more precise by describing nylon as being ‘non-biodegradable’.

(iii) Nylon is strong and can be drawn into long, thin strands without breaking. Thus, it is a better
material than silk for making items such as ropes and nets. However, in some instances,
biodegradable materials such as silk are more useful. For example, fishing lines often break
and get lost in the sea. If they are made of nylon, which is non-biodegradable, they may pollute
the sea as they do not decompose.

TEST IT!

Section A: Multiple-Choice Questions


Choose the correct answer and write its letter in the brackets provided.

1. A section of the structure of a polymer is shown below.


F Cl F Cl
C C C C
Cl H Cl H
Which of the following shows the monomer of this polymer?
A Cl H B Cl F
C C C C
F Cl H H
C Cl F D F Cl
C C C C
Cl H F H ( A )

164 Worksheet 20 © 2013 Marshall Cavendish International (Singapore) Private Limited

(S)CMWB_20.indd 164 8/27/12 8:20 PM


2. Which of the following shows the correct polymer that can be obtained from the given monomer?
Monomer Polymer
A H H H H H H
H C C H C C C C
H H H H H H

B H Cl Cl H Cl H
C C C C C C
Cl H H Cl H Cl

C CH3 Cl CH3 H CH3 H


C C C C C C
H H Cl H Cl H

D H H H H
H O C C O H O C C O
H H H H ( B )

3. Which of the following monomers reacts with H O O H to form a synthetic fibre?


A O B H
C6H5 C O H CH3(CH2)5 N H

C O O D H H
H O C C6H4 C O H H N (CH2)6 N H ( C )

4. A section of the structure of a macromolecule is shown below.


O O H H O O H H
C C N N C C N N
Which of the statements about this macromolecule is correct?
A It is formed by addition polymerisation.
H O O H H
B The repeat unit is N C C N N .
C Its monomers are HOOC COOH and H O O H.
D Its monomers are HOOC COOH and NH2 NH2. ( D )

© 2013 Marshall Cavendish International (Singapore) Private Limited Worksheet 20 165

(S)CMWB_20.indd 165 8/27/12 8:20 PM


5. A polymer is made from the monomers shown below.
H H O O
H N (CH2)4 N H H O C (CH2)4 C O H

Which of the following is produced when the monomers react?


A Hydrogen gas
B Ammonia gas
C A polyester
D Water ( D )

6. Which of the following materials are non-biodegradable?


1 Poly(propene)
2 Nylon
3 Terylene
4 Wool
A 1 and 2
B 2 and 4
C 1, 2 and 3
D 1, 3 and 4 ( C )

Section B: Structured Questions


Answer the following questions in the spaces provided.

1. Teflon is used to coat the surfaces of non-stick frying pans. It is a macromolecule formed by the
addition polymerisation of tetrafluoroethene. The formula of tetrafluoroethene is given below.
F F
C C
F F
(a) Explain the term ‘addition polymerisation’ using Teflon as an example.

Addition polymerisation occurs when unsaturated monomer units, i.e. tetrafluoroethene, join together

to form a long-chain molecule, i.e. Teflon, as the only product without the loss of any molecules or

atoms.

(b) Draw part of the structure of the Teflon polymer, showing three repeat units.
F F F F F F
C C C C C C
F F F F F F

166 Worksheet 20 © 2013 Marshall Cavendish International (Singapore) Private Limited

(S)CMWB_20.indd 166 8/27/12 8:20 PM


(c) Calculate the percentage of carbon in a Teflon molecule.
The repeat unit of Teflon is:
F F
C C
F F
Relative molecular mass of a repeat unit (C2F4) = (2 × 12) + (4 × 19)
= 100
Percentage of carbon in the repeat unit = 2 × 12 × 100
100
= 24%
Percentage of carbon in Teflon = 24%

2. (a) Deduce the structural formulae and names of the monomers for the following polymers.

Polymer Monomer
Structural formula:

H H H H H H
C C C C C C

Cl H Cl H Cl H

Poly(chloroethene)
Name: Chloroethene

Structural formula:
H C6H5 H C6H5 H C6H5

C C C C C C
H H
H H H H
Polystyrene
Styrene
Name:

(b) State two uses of poly(ethene).

For making plastic bags/ toys/ buckets/ clingfilm

(c) ,(i) Explain why nylon is classified as a polyamide and Terylene, a polyester.
Nylon is a polyamide because it contains many amide linkages, CONH .

Terylene is a polyester because it contains many ester linkages, COO .

(ii) Nylon and Terylene are examples of synthetic fibres. State two uses of synthetic fibres.

For making clothes/ sleeping bags/ curtains/ parachutes/ fishing lines

© 2013 Marshall Cavendish International (Singapore) Private Limited Worksheet 20 167

(S)CMWB_20.indd 167 8/27/12 8:20 PM


3. Poly(carbonates) are used in the manufacture of compact discs. The structure of a poly(carbonate)
is shown below.
O O O
C O O C O O C O O

(a) Draw the repeat unit of a poly(carbonate).


O
C O O

(b) One of the two monomers used to prepare the poly(carbonate) is phosgene. Its structural
formula is:
O
Cl C Cl

Draw the structure of the other monomer.

H O O H

(c) (i) What type of reaction is used to make poly(carbonates)?


Condensation polymerisation

(ii) What is the inorganic side-product in this reaction?

Hydrogen chloride

*4. (a) Two monomer molecules are shown below. They are used to make a polymer, Y.
O O
H O C C O H H O O H

(i) Draw the structure of Y, showing two repeat units.


O O O O
C C O O C C O O

(ii) Name a synthetic polymer that has a similar structure as Y. Give a reason for your
answer.

Terylene. Both polymers are polyesters.

168 Worksheet 20 © 2013 Marshall Cavendish International (Singapore) Private Limited

(S)CMWB_20.indd 168 8/27/12 8:20 PM


(b) When a macromolecule, Z, is boiled with dilute hydrochloric acid, it breaks down to form two
organic products. The structural formulae of the two products are shown below.
O O H H
H O C C O H H N N H

(i) Deduce whether Z is an addition polymer or a condensation polymer.


Condensation polymer

(ii) Draw the structure of Z, showing two repeat units.


O O H H O O H H
C C N N C C N N

Section C: Free-Response Question


Answer the following question.

1. (a) (i) Describe the formation of poly(propene).


The monomer for making poly(propene) is propene. Propene undergoes addition polymerisation

under high temperature and pressure and in the presence of a catalyst to form poly(propene).

(ii) Write an equation to show the reaction described in (i).


CH3 H CH3 H
nC C C C
H H H H n

Propene Poly(propene)

(b) Poly(propene) is used for making plastic bottles. State two advantages and one disadvantage
of using plastic bottles instead of glass bottles.
Plastic bottles are lighter than glass bottles and they do not break easily. However, they are

non-biodegradable and thus cause land pollution when disposed of.

© 2013 Marshall Cavendish International (Singapore) Private Limited Worksheet 20 169

(S)CMWB_20.indd 169 8/27/12 8:20 PM


(c) An addition polymer contains 2000 monomer units which are alkene molecules. The relative
molecular mass of the polymer is 84 000. Identify the monomer used.

Relative molecular mass of the monomer = 84 000


2000
= 42
The general formula of an alkene is CnH2n.
Relative molecular mass of CnH2n = 42
12n + (2 × 1)n = 42
14n = 42
n =3
The monomer is propene, C3H6.

WORK ON IT!
In the table below, circle the question(s) that you have answered incorrectly. Revisit the
relevant section(s) in the textbook to strengthen your understanding of the key concept(s).

Question number(s) Textbook


Learning outcomes Multiple- Free- section(s)
Structured to revise
choice response
(a) Describe the structure of
– 1(a) – 24.1
macromolecules.
(b) Describe the addition polymerisation of
– 1(a) 1(a) 24.2
ethene to form poly(ethene).
(c) Deduce the structure of a polymer from 1(b), 2(a),
its monomer and vice versa. 3(a), 3(b),
1, 2 – 24.2
4(a)(i),
4(b)(ii)
(d) State some uses of poly(ethene). – 2(b) – 24.2
(e) Describe the formation of nylon
and Terylene by condensation 3, 4, 5 3(c), 4(b) – 24.3
polymerisation.
(f) Describe nylon as a polyamide and 2(c)(i),
– – 24.3
Terylene, a polyester. 4(a)(ii)
(g) State some uses of synthetic fibres
– 2(c)(ii) – 24.3
such as nylon and Terylene.
(h) Describe the problem of pollution
6 – 1(b) 24.4
caused by the disposal of plastics.
(i) Calculate the relative molecular
mass or relative formula mass of a – – 1(c) 9.2
substance.
(j) Determine the percentage composition
– 1(c) – 9.4
of compounds from given data.

170 Worksheet 20 © 2013 Marshall Cavendish International (Singapore) Private Limited

(S)CMWB_20.indd 170 8/27/12 8:20 PM


Name: ( )

Trial Examination
Class: Date:

Trial Examination
PAPER 1
Answer all questions.

1. An isotope of element Z forms an ion, Z3–. The Z3– ion has 18 electrons and 17 neutrons.
What is element Z?
A Argon
B Nitrogen
C Oxygen
D Phosphorus ( )

2. Benzoic acid occurs naturally in some plants. It is widely used in the food industry as a food
preservative. The melting and boiling points of benzoic acid are shown below.
• Melting point: 122°C
• Boiling point: 249°C
Which of the following occurs when benzoic acid at 2000°C is cooled to 100°C?
Distance between particles Energy of particles
A increases increases
B increases decreases
C decreases decreases
D decreases increases ( )

3. A student is asked to confirm the presence of Ca2+ and I– ions in a salt solution.
Which of the following observations is incorrect?
Test Observation
A add ammonia solution no precipitate
B add sodium hydroxide solution white precipitate, insoluble in excess
C add barium chloride solution yellow precipitate
D add dilute sulfuric acid white precipitate ( )

4. A substance, X, has the following properties:


• Melting point: 3550°C
• Does not conduct electricity
Which of the following substances is likely to be X?
A Diamond
B Graphite
C Copper
D Poly(ethene) ( )

© 2013 Marshall Cavendish International (Singapore) Private Limited Paper 1 171

(S)CMWB_TrialExPaper.indd 171 8/27/12 8:21 PM


5. The diagram below shows the electrons present in the outer shell of element R.
What is the chemical formula of the sulfate of R?

A R(SO4)2
B R2SO4
C R2(SO4)3
D R3(SO4)2 ( )

6. The positions of elements X and Y in the Periodic Table are shown below.

X
Y

Which of the following is true about the two elements?


A X has a higher melting point than Y.
B X reacts more vigorously with water than Y.
C The carbonates of X and Y are insoluble in water.
D The chlorides of X and Y are covalent molecules. ( )

7. Which of the following equations is not balanced?


A 4Na + O2 2Na2O
B Al(OH)3 + 3HCl AlCl3 + 3H2O
C Fe2O3 + 2CO 2Fe + 3CO2
D C6H12O6 2C2H5OH + 2CO2 ( )

8. The thermite reaction is highly exothermic. It is thus used for welding steel in railway lines. The
equation for the thermite reaction is shown below.
2Al(s) + Fe2O3(s) Al2O3(s) + 2Fe(s)
What is the mass of iron(III) oxide, Fe2O3, needed to produce 3.0 mol of iron, Fe?
A 60 g
B 120 g
C 180 g
D 240 g ( )

9. Which of the following statements is true for all metals?


A They are solids at room temperature and pressure.
B They have high melting points.
C They are denser than water.
D They are conductors of electricity. ( )

172 Paper 1 © 2013 Marshall Cavendish International (Singapore) Private Limited

(S)CMWB_TrialExPaper.indd 172 8/27/12 8:21 PM


10. Which of the following changes has a positive ∆H value?
A 2Na(s) + 2H2O(l) 2NaOH(aq) + H2(g)
B NaOH(aq) + HCOOH(aq) HCOONa(aq) + H2O(l)
C 2HBr(g) H2(g) + Br2(g)
D H2O(l) H2O(s) ( )

11. A steel spoon is electroplated with copper in an electrolytic cell.


Which of the following statements is correct?
A The electrolytic cell contains molten copper(II) sulfate.
B The anode is pure copper.
C Oxygen gas is liberated at the anode.
D Copper dissolves to form Cu2+ ions at the cathode. ( )

12. Alkaline cells convert chemical energy to electrical energy. They are commonly used in portable
audio devices. The reactions that occur at the electrodes of an alkaline battery are shown below.
Electrode X: Zn(s) + 2OH–(aq) ZnO(s) + H2O(l) + 2e–
Electrode Y: 2MnO2(s) + H2O(l) + 2e– Mn2O3(s) + 2OH–(aq)
Which of the following statements is correct?
A Electrical energy is produced by the neutralisation reaction.
B Zinc acts as the oxidising agent in the alkaline cell.
C Oxidation occurs at electrode X.
D The oxidation number of manganese decreases from +5 to +3. ( )

13. An experiment was carried out to study the reaction between calcium carbonate (in excess) and
10 cm3 of 0.2 mol/dm3 hydrochloric acid at 25°C. Graph I shows the results of the experiment.

Volume of CO2/cm3

II

Time/min

Which set of conditions will produce Graph II?

Volume of Concentration of
Temperature/°C
HCl/cm3 HCl/(mol/dm3)
A 20 0.2 20
B 20 0.1 25
C 30 0.2 30
D 10 0.1 30
( )

© 2013 Marshall Cavendish International (Singapore) Private Limited Paper 1 173

(S)CMWB_TrialExPaper.indd 173 8/27/12 8:21 PM


14. Which reactant has undergone a reduction reaction to form the product?
Reactant Product
A SO2 SO3
B HCl Cl2
C H+ H2
D CuO CuSO4 ( )

15. The enzyme, catalase, is used in the rubber industry to convert latex to foam rubber. It can also
be used as a catalyst in the decomposition of hydrogen peroxide.
catalase
2H2O2(aq) 2H2O(l) + O2(g)

The energy profile diagram for the catalysed and uncatalysed decomposition of hydrogen peroxide
is shown below.
Energy

E4
E3

2H2O2(aq)
E2

2H2O(l) + 02(g)
E1

Progress of reaction

Which of the following represents the activation energy in both reactions?

Catalysed reaction Uncatalysed reaction


A E3 – E1 E4 – E1
B E3 – E2 E4 – E2
C E4 – E1 E3 – E1
D E4 – E2 E3 – E2 ( )

16. The reaction between lead(II) acetate and potassium chromate produces a yellow precipitate of
lead(II) chromate. Which of the following is true about the solubilities of the salts?
Lead(II) acetate Potassium chromate Lead(II) chromate
A soluble soluble soluble
B soluble soluble insoluble
C insoluble insoluble insoluble
D insoluble insoluble soluble ( )

17. Which process is used to obtain hydrogen for the manufacture of ammonia?
A Cracking of petroleum
B Fractional distillation of liquid air
C Fractional distillation of petroleum
D Haber process ( )

174 Paper 1 © 2013 Marshall Cavendish International (Singapore) Private Limited

(S)CMWB_TrialExPaper.indd 174 8/27/12 8:21 PM


18. Tin is below iron and above copper in the reactivity series. Which of these statements correctly
describes iron, tin and copper?
1 Their sulfates are soluble in water.
2 Their oxides can be reduced by carbon on heating.
3 Their oxides can be reduced by hydrogen on heating.
A 1 and 2
B 1 and 3
C 2 and 3
D 1, 2 and 3 ( )

19. 10.0 cm3 of 1.0 mol/dm3 ethanoic acid, CH3COOH, is slowly added to 20.0 cm3 of 1.0 mol/dm3
sodium hydroxide, NaOH, containing methyl orange. The reaction mixture is constantly stirred.

glass rod
10.0 cm3 of
1.0 mol/dm3 CH3COOH

20.0 cm3 of
1.0 mol/dm3 NaOH
and methyl orange

Which of the following statements is true about this experiment?


A The pH of the reaction mixture is less than 7.
B The colour of the indicator remains unchanged.
C A precipitate of sodium ethanoate is formed.
D 0.02 mol of water is produced. ( )

20. 0.02 mol of the hydroxide of a metal, M, requires 15.00 cm3 of 2.0 mol/dm3 sulfuric acid for
complete neutralisation.
What is the chemical formula of the salt produced?
A MSO4
B M(SO4)3
C M2SO4
D M2(SO4)3 ( )

21. Which of the following pairs of reactants is correctly matched with the colour change that occurs
during the reaction?
Reactants Colour change
A chlorine gas and potassium iodide solution brown to colourless
B ethanol and acidifed potassium manganate(VII) colourless to purple
C zinc metal and copper(II) sulfate solution blue to colourless
D iron metal and dilute sulfuric acid colourless to yellow ( )

© 2013 Marshall Cavendish International (Singapore) Private Limited Paper 1 175

(S)CMWB_TrialExPaper.indd 175 8/27/12 8:21 PM


22. The symbols of two elements, X and Y, are shown below.
23 16
11 X 8Y

When X reacts with Y, compound Z is formed. What is the molar mass of compound Z?
A 39 g/mol
B 55 g/mol
C 62 g/mol
D 94 g/mol ( )

23. Which of the following contains at least two compounds?


A Brass
B Diamond
C Petrol
D Ammonium nitrate ( )

24. A mixture containing aqueous sodium chloride and aqueous copper(II) sulfate is distilled.

water out

distillation flask

NaCl(aq) and CuSO4(aq)

water in

distillate

Which of the following is correct?

Temperature of
Thermometer
solution in the Distillate
reading/°C
distillation flask/°C
A 100 100 aqueous sodium chloride
B > 100 100 aqueous copper(II) sulfate
C > 100 100 water
D > 100 > 100 no distillate ( )

176 Paper 1 © 2013 Marshall Cavendish International (Singapore) Private Limited

(S)CMWB_TrialExPaper.indd 176 8/27/12 8:21 PM


25. A solid mixture contains 1.0 mol of hydrated iron(II) sulfate, FeSO4.7H2O, and 0.5 mol of hydrated
nickel(II) sulfate, NiSO4.6H2O.
How many moles of ions are found in the solution when the solid mixture is completely dissolved
in water?
A 2
B 3
C 4
D 10 ( )

26. Four experiments are carried out to determine the conditions that cause the rusting of iron.

NaCl(aq) NaCl(aq)

silver iron lead iron

Beaker 1 Beaker 2

NaCl(aq) NaCl(aq)

copper iron zinc iron

Beaker 3 Beaker 4

In which beaker will the rusting of iron be observed?


A Beaker 1 only
B Beaker 4 only
C Beakers 2 and 4
D Beakers 1, 2 and 3 ( )

27. Which of these reactions does not occur in the blast furnace during the extraction of iron?
A C + CO2 2CO
B CaCO3 CaO + CO2
C CaO + SiO3 CaSiO4
D Fe2O3 + 3CO 2Fe + 3CO2 ( )

© 2013 Marshall Cavendish International (Singapore) Private Limited Paper 1 177

(S)CMWB_TrialExPaper.indd 177 8/27/12 8:21 PM


28. The mass of magnesium used in the experiment below is 1.0 g.

magnesium gas Q

glass wool
soaked in
water
heat

water

What is the mass of the product formed and the identity of gas Q?
Mass of product Gas Q
A more than 1.0 g hydrogen
B more than 1.0 g oxygen
C less than 1.0 g hydrogen
D less than 1.0 g oxygen ( )

29. L, M and N are Period 3 elements. They form different types of oxides as shown below.

Element Type of oxide


L basic
M amphoteric
N acidic

Which of the following shows the correct order of the elements in terms of increasing proton
number?
A L, M, N
B L, N, M
C N, M, L
D N, L, M ( )

30. Which of the following reactions can be used to prepare lead(II) chloride?
A Pb(s) + 2HCl(aq) PbCl2(s) + H2(g)
B Pb(NO3)2(aq) + 2HCl(aq) PbCl2(s) + 2HNO3(aq)
C PbCO3(s) + 2HCl(aq) PbCl2(s) + H2O(l) + CO2(g)
D PbO(s) + 2HCl(aq) PbCl2(s) + H2O(l) ( )

31. When compound Q is heated, a colourless gas is liberated and a white residue is obtained. The
colourless gas forms a white precipitate with limewater.
Which of the following could be compound Q?
A Calcium nitrate
B Calcium carbonate
C Copper(II) carbonate
D Copper(II) nitrate ( )

178 Paper 1 © 2013 Marshall Cavendish International (Singapore) Private Limited

(S)CMWB_TrialExPaper.indd 178 8/27/12 8:21 PM


32. Consider the set-up of a simple cell as shown below.

magnesium iron

NaCl(aq)

Which statement about this simple cell is correct?


A The iron electrode is the negative terminal.
B The electrons flow from the iron electrode to the magnesium electrode.
C The solution turns green after some time.
D The magnesium electrode decreases in size. ( )

33. Which of the following statements about methane is correct?


1 It is a colourless, odourless gas.
2 It is a greenhouse gas.
3 It causes the depletion of the ozone layer.

A 1 and 2
B 1 and 3
C 2 and 3
D 1, 2 and 3 ( )

34. A given volume of propane produces 13.2 g of carbon dioxide on complete combustion. What is
the mass of water produced in the reaction?
A 7.2 g
B 13.2 g
C 21.6 g
D 28.8 g ( )

35. Ethanoic acid is a 1 acid. It is prepared by the 2 of ethanol. The reaction between ethanoic
acid and methanol produces an ester called 3 . Esters are used as 4 .
1 2 3 4
A strong oxidation methyl ethanoate flavourings
B strong reduction ethyl methanoate fuels
C weak oxidation methyl ethanoate flavourings
D weak reduction ethyl methanoate fuels ( )

© 2013 Marshall Cavendish International (Singapore) Private Limited Paper 1 179

(S)CMWB_TrialExPaper.indd 179 8/27/12 8:21 PM


36. Which of the following pairs of organic compounds are not isomers?
A H O O
H C C O H H C O CH3
H

B H H H H H H
H C C C O H H C O C C H
H H H H H H

C H H H H H H
H C C C C H H C C C H
H H H
H C H
H

D H H H H H
H C C C C H H H C H H
H H H H H C C C H
H H C H H
H ( )

37. Aspirin is used to relieve pain. The structural formula of aspirin is shown below.

O OH
C
H C O CH3
C C C
C C O
H C H
H
Which of the following statements about aspirin are correct?
1 It contains an ester group.
2 It reacts with sodium carbonate solution.
3 It reacts with acidified potassium manganate(VII) on heating.
A 1 and 2
B 1 and 3
C 2 and 3
D 1, 2 and 3 ( )

180 Paper 1 © 2013 Marshall Cavendish International (Singapore) Private Limited

(S)CMWB_TrialExPaper.indd 180 8/27/12 8:21 PM


38. An organic compound, X, is found in coconut oil. It has the molecular formula C17H31COOH. What
can be deduced about X?
A It is an alcohol.
B It is an ester.
C It is a saturated carboxylic acid.
D It is an unsaturated carboxylic acid. ( )

39. A student carried out some tests on compound Y. The structural formula of Y is shown below.
H H H H H
H C C C C C O H
H H H
Which of the following observations is incorrect?
A Aqueous bromine was decolourised when Y was added to it.
B Effervescence was observed when sodium carbonate was reacted with Y.
C The colour of acidified potassium manganate(VII) changed from purple to colourless when it
was heated with Y.
D A sweet, fruity smell was detected when Y was heated with a mixture of methanoic acid and
concentrated sulfuric acid. ( )

40. Which of the following statements about nylon is correct?


A It is obtained by addition polymerisation.
B The monomers of nylon are HOOC– –COOH and HO– –OH.
C Each repeat unit in the nylon polymer contains two nitrogen atoms.
D The combustion of nylon produces hydrogen chloride gas. ( )

© 2013 Marshall Cavendish International (Singapore) Private Limited Paper 1 181

(S)CMWB_TrialExPaper.indd 181 8/27/12 8:21 PM


PAPER 2
Section A
Answer all questions in this section.

1. The diagram below shows the positions of eight elements in the Periodic Table. (Note: The letters
do not represent the chemical symbols of the elements.)

H
A E
B D F
C G

(a) State the group that H belongs to in the Periodic Table. [1]

(b) Write the electronic configuration of


(i) A; [1]
(ii) D. [1]

(c) State the element that


(i) gains three electrons when it forms an ion; [1]
(ii) forms compounds with oxidation states +2 and +3; [1]
(iii) has four valence electrons. [1]

(d) Write the chemical formula of the compound formed when A and F combine. [1]
[Total: 7 marks]

2. Since 1992, it has been compulsory for all new petrol-fuelled vehicles in Singapore to be fitted
with catalytic converters in the car exhaust systems.
(a) The diagram below shows three air pollutants that are passed through a catalytic converter.
They are converted to three other gases. Identify these gases. [2]

• unburnt hydrocarbons • gas P


catalytic
• nitrogen dioxide • gas Q
converter
• carbon monoxide • gas R

(b) In terms of oxidation and reduction, state and explain the reactions that occur when unburnt
hydrocarbons pass through the catalytic converter. [2]

(c) (i) Write an equation for the reaction between nitrogen dioxide and carbon monoxide in the
catalytic converter. [1]
(ii) Calculate the total volume of gaseous products obtained when 10 cm3 of nitrogen dioxide
reacts with 50 cm3 of carbon monoxide. [2]

(d) (i) Name another air pollutant that may also be found in car exhaust fumes. [1]
(ii) Write an equation to show how this substance is formed in car engines. [1]
[Total: 9 marks]

182 Paper 2 © 2013 Marshall Cavendish International (Singapore) Private Limited

(S)CMWB_TrialExPaper.indd 182 8/27/12 8:21 PM


3. An alkene, L, has a relative molecular mass of 42.
(a) What is its
(i) molecular formula; [2]
(ii) empirical formula; [1]
(iii) structural formula? [1]

(b) M is a polymer produced from L.


(i) Draw the structure of M, showing three repeat units. [2]
(ii) Name the type of chemical reaction that is used to produce M. [1]
(iii) Suggest a use for M. [1]
[Total: 8 marks]

4. The diagram below shows the arrangement of outer electrons in a compound consisting of three
elements.

(a) State whether each of the following statements about this compound is true or false. [2]
(i) It could be hydrogen cyanide, HCN.
(ii) It is a covalent compound.
(iii) It can conduct electricity.
(iv) It is an unsaturated hydrocarbon.

(b) Draw a similar diagram to show the arrangement of outer electrons in ethene, C2H4. [3]
[Total: 5 marks]

5. Ammonia is obtained by the Haber process.


(a) State the raw materials and conditions required for the Haber process. [3]

(b) The graph of percentage yield of ammonia against temperature at pressures of 300 atm,
400 atm and 500 atm is shown below.

Yield of ammonia/%

100

500 atm
400 atm
300 atm

0 Temperature/°C
400 500 600 700 800

(i) What is the effect of pressure on the yield of ammonia? [1]


(ii) The graph above shows that a lower temperature gives a higher yield of ammonia.
However, a lower temperature is not used in the Haber process. Explain why. [1]

© 2013 Marshall Cavendish International (Singapore) Private Limited Paper 2 183

(S)CMWB_TrialExPaper.indd 183 3/10/16 10:21 AM


(c) (i) A factory manufactures ammonia to make fertilisers. When 2.8 tonnes of nitrogen were
used in the manufacturing process, 2.55 tonnes of ammonia were produced. Calculate
the percentage yield of ammonia. (1.0 tonne = 106 g) [2]
(ii) Suggest why the percentage yield of ammonia calculated in (c)(i) is not 100%. [1]

(d) Calcium hydroxide is sometimes used in agriculture to neutralise excess acidity in the soil.
However, it may react with nitrogenous fertilisers such as ammonium nitrate and release
ammonia gas into the atmosphere.

Write an equation, with state symbols, for the reaction between calcium hydroxide and
ammonium nitrate. [2]
[Total: 10 marks]

6. A reaction scheme is shown below.

white powder heat white residue Y colourless


+
X (when cold) gas

+ HNO3(aq) + limewater

white precipitate Z, + NH3(aq) colourless white


soluble in excess
solution precipitate
NH3(aq)

(a) Identify X, Y and Z. [2]

(b) Outline the steps for the preparation of X, starting from Y. [3]
[Total: 5 marks]

7. The diagram below shows six elements, which are found in Period 3 of the Periodic Table. (The
letters A–F in the diagram do not represent the chemical symbols of the elements.)
23 28 31 32 32 37
11 A 14 B 15 C 14 D 15 E 17 F

(a) (i) Which of these elements have similar chemical properties? [1]
(ii) State a reason for your answer in (a)(i). [1]

(b) Deduce the number of protons, neutrons and electrons in the ion formed from F. [2]

(c) Which element


(i) forms an oxide with a giant molecular structure; [1]
(ii) is made up of a lattice of positive ions in a ‘sea of electrons’? [1]
[Total: 6 marks]

184 Paper 2 © 2013 Marshall Cavendish International (Singapore) Private Limited

(S)CMWB_TrialExPaper.indd 184 8/27/12 8:21 PM


Section B
Answer three questions in this section.

The last question is in the form of an either/or question. Answer only one of the alternatives.

8. Aluminium is used extensively for the construction of aircraft because of its low density and
resistance to corrosion.

Aluminium is extracted industrially from bauxite, an ore which contains aluminium oxide. The
diagram below shows the electrolytic cell used for the manufacture of aluminium. The electrolyte
is a mixture of molten aluminium oxide and cryolite, Na3AlF6. The function of cryolite is to lower the
melting point of aluminium oxide so that electrolysis can be carried out at a lower temperature.
carbon anode
carbon cathode

molten aluminium oxide


and cryolite
molten aluminium
tapping
hole

(a) State, with ionic half-equations, whether oxidation or reduction occurs at the
(i) carbon anode; [2]
(ii) carbon cathode. [2]

(b) Explain why


(i) the anodes in the electrolytic cell have to be replaced from time to time; [2]
(ii) the melting point of aluminium oxide has to be lowered. [2]

(c) During the extraction of aluminium from aluminium oxide, 128 tonnes of oxygen were
produced. Calculate the mass, in tonnes, of aluminium produced. (1 tonne = 106 g) [2]
[Total: 10 marks]

9. (a) Honeycombs in a beehive are made of beeswax, which is commercially used to make wax
paper, candles, cosmetics and polishing materials such as shoe polish and furniture polish.
Beeswax is a mixture of esters (about 74%), hydrocarbons (about 20%) and other substances
(about 6%). The esters are made from long-chain carboxylic acids and alcohols.

An ester found in beeswax has the following structural formula:


O
CH3(CH2)14 C O (CH2)29CH3

(i) Write the chemical formulae of the organic compounds that can be used to form this
ester. State the conditions for the reaction. [2]
(ii) Student A predicts that beeswax will melt at a constant temperature of about 62°C.
Student B predicts that it will melt at a temperature range of 62–66°C, while student C
predicts that it will melt at a temperature range of 1000–1200°C.

Which student is correct? Give reasons for your answer. [3]

© 2013 Marshall Cavendish International (Singapore) Private Limited Paper 2 185

(S)CMWB_TrialExPaper.indd 185 3/16/16 11:19 AM


(b) Amides are a homologous series of organic compounds containing nitrogen atoms. With the
exception of methanamide, amides are solids at room temperature and pressure. The table
below shows some information on the first two amides.

Name Chemical formula Full structural formula


O H
methanamide HCONH2
H C N H
H O H
ethanamide CH3CONH2 H C C N H
H

When ethanamide is boiled with aqueous sodium hydroxide, the following reaction occurs:

H O H H O
H C C N H + NaOH H C C ONa + NH3
H H
(i) An amide with four carbon atoms is boiled with excess aqueous sodium hydroxide.
Name the amide and organic product formed. Write an equation for the reaction, showing
the full structural formulae of the organic compounds. [3]
O H
(ii) Name another organic compound that contains the C N group and state one of its
uses. [2]
[Total: 10 marks]
EITHER

10. (a) In Experiment 1, 0.1 mol/dm3 aqueous sodium hydroxide was added slowly from a burette to
25.0 cm3 of hydrochloric acid in a conical flask. The changes in pH during the titration were
recorded by a pH probe attached to a computer as shown in Figure 1.
pH

14

12

10
8
6

0 10 20 30 40
Volume of NaOH added/cm3

Figure 1

186 Paper 2 © 2013 Marshall Cavendish International (Singapore) Private Limited

(S)CMWB_TrialExPaper.indd 186 8/27/12 8:21 PM


Experiment 2 was then carried out using 0.05 mol/dm3 aqueous sodium hydroxide instead.
(i) Using the information in Figure 1, calculate the concentration, in g/dm3, of the
hydrochloric acid in the conical flask. [4]
(ii) What is the volume of aqueous sodium hydroxide required to reach the end-point in
Experiment 2? [1]

(b) Phosphoric acid, H3PO4, contains three hydrogen atoms. It forms three types of sodium salts,
NaH2PO4, Na2HPO4 and Na3PO4, upon reaction with aqueous sodium hydroxide.
A titration was carried out by adding 0.1 mol/dm3 aqueous sodium hydroxide slowly to
25.0 cm3 of 0.1 mol/dm3 phosphoric acid. The graph in Figure 2 shows the changes in pH
during the titration.
pH

14

12

10

25 50 75
Volume of NaOH added/cm3
Figure 2
(i) Find the mole ratio of the acid and base used when 25.0 cm3 of 0.1 mol/dm3 aqueous
sodium hydroxide is added to 25.0 cm3 of 0.1 mol/dm3 phosphoric acid. [2]
(ii) Using your answer in (b)(i), write an equation for the reaction between the acid and
base used. [1]
(iii) The pH value of a salt formed during the titration can be obtained from the mid-point
of the vertical part of the titration curve. Hence, estimate the pH values of the salts,
NaH2PO4 and Na2HPO4, from Figure 2. [2]
[Total: 10 marks]
OR

10. (a) The primary source of tin is the ore, cassiterite, which contains tin dioxide. Cassiterite is
mixed with coke and heated to a high temperature of about 1360°C in the furnace.

Predict three reactions that occur in the furnace. Write equations, including state symbols,
for these reactions. [3]

© 2013 Marshall Cavendish International (Singapore) Private Limited Paper 2 187

(S)CMWB_TrialExPaper.indd 187 8/27/12 8:21 PM


(b) The diagram below shows the set-up used to investigate the reduction of three metal oxides
with hydrogen.

metal oxide
hydrogen
gas burning
dry hydrogen

The table shows the appearance of each metal oxide before the experiment.

Colour of oxide at
Experiment Metal oxide
room temperature
1 lead(IV) oxide brown
2 aluminium oxide white
3 copper(II) oxide black

(i) Predict the colour change that would be observed in each experiment. [3]
(ii) The following table shows the mass of lead(IV) oxide and lead at the start and end of the
experiment.

Mass of lead(IV) oxide at the start of the experiment/g 4.78

Mass of lead at the end of the experiment/g 4.14

Show that the empirical formula of the oxide used is PbO2. [2]
(iii) Describe a chemical test to distinguish between aluminium oxide and copper(II) oxide.
Give a reason for your answer. [2]
[Total: 10 marks]

188 Paper 2 © 2013 Marshall Cavendish International (Singapore) Private Limited

(S)CMWB_TrialExPaper.indd 188 8/27/12 8:21 PM


Answers to Trial Examination
Paper 1
1. D 6. A 11. B 16. B 21. C 26. D 31. B 36. D
2. C 7. C 12. C 17. A 22. C 27. C 32. D 37. A
3. C 8. D 13. A 18. D 23. C 28. A 33. A 38. D
4. A 9. D 14. C 19. B 24. C 29. A 34. A 39. B
5. C 10. C 15. B 20. D 25. B 30. B 35. C 40. C

Paper 2
Section A
1. (a) Group VIII/ 0

(b) (i) 2, 1
(ii) 2, 8, 4

(c) (i) E
(ii) C
(iii) D
(iv) A2F

2. (a) Water vapour, nitrogen, carbon dioxide


[Three correct: 2 marks; two correct: 1 mark]

(b) Unburnt hydrocarbons react with oxygen and are oxidised to carbon dioxide. [1] Oxygen is
reduced to form water. [1]

(c) (i) 2NO2(g) + 4CO(g) N2(g) + 4CO2(g)


(ii) The limiting reagent is nitrogen dioxide.
From the equation 2NO2(g) + 4CO(g) N2(g) + 4CO2(g),
2 mol of NO2 react with 4 mol of CO to produce 1 mol of N2 and 4 mol of CO2.
∴ Volume of N2 produced = 10 = 5 cm3
2
4
Volume of CO2 produced = × 10 = 20 cm3 [1]
2
Total volume of gaseous products = 5 + 20 = 25 cm3 [1]

(d) (i) Sulfur dioxide


(ii) S(s) + O2(g) SO2(g)

3. (a) (i) The general formula of an alkene is CnH2n.


Mr of L (CnH2n) = 42
12n + 2n = 42
n = 3 [1]
Therefore, the molecular formula of L is C3H6. [1]

© 2013 Marshall Cavendish International (Singapore) Private Limited Answers to Trial Examination 189

(S)CMWB_Answer.indd 189 8/27/12 8:21 PM


(ii) CH2
(iii) H H H
H C C C H
H

(b) (i) CH3 H CH3 H CH3 H


C C C C C C
H H H H H H
[Correct repeat unit: 1 mark; three repeat units: 1 mark]
(ii) Addition polymerisation
(iii) For making plastic bags

4. (a) (i) True


(ii) True
(iii) False
(iv) False
[Four correct: 2 marks; two correct: 1 mark]

(b)

H H

C C

H H

Arrangement of C and H atoms is correct. [1]


Number of electrons shared between carbon atoms (C=C) is correct. [1]
Number of electrons shared between carbon and hydrogen atoms (C–H) is correct. [1]

5. (a) Raw materials: nitrogen and hydrogen [1]


Conditions: temperature of 450°C, pressure of 250 atm, iron catalyst
[Three correct: 2 marks; two correct: 1 mark]

(b) (i) At constant temperature, the higher the pressure, the greater the yield of ammonia.
(ii) A lower temperature will result in a slower reaction, which is not desirable for the
large-scale production of ammonia.

(c) (i) N2(g) + 3H2(g) 2NH3(g)


Mr of NH3 = 14 + (3 × 1) = 17
Mr of N2 = 2 × 14 = 28
From the equation,
Theoretical yield of ammonia = 2.8 × (2 × 17) = 3.4 tonnes [1]
28
Percentage yield = actual yield × 100% = 2.55 × 100% = 75% [1]
theoretical yield 3.4
(ii) The reaction between nitrogen and hydrogen is reversible. Hence, it is not possible to
obtain 100% yield of ammonia.

190 Answers to Trial Examination © 2013 Marshall Cavendish International (Singapore) Private Limited

(S)CMWB_Answer.indd 190 8/27/12 8:21 PM


(d) Ca(OH)2(aq) + 2NH4NO3(aq) [1] 2NH3(g) + Ca(NO3)2(aq) + 2H2O(l) [1]

6. (a) X is zinc carbonate; Y is zinc oxide; Z is zinc hydroxide.


[Three correct: 2 marks; two correct: 1 mark]

(b) Step 1: Dissolve Y (zinc oxide) in dilute nitric acid. [1]


Step 2: Filter to remove excess Y. Collect the filtrate. [1]
Step 3: Add excess aqueous sodium carbonate. Filter to obtain residue X
(zinc carbonate). [1]

7. (a) (i) Elements C and E (or B and D)


(ii) They are isotopes.

(b) 17 protons, 20 neutrons, 18 electrons


[Three correct: 2 marks; two correct: 1 mark]

(c) (i) B or D
(ii) A

Section B
8. (a) (i) Oxidation occurs at the anode. The oxide ion is oxidised to oxygen. [1]
2O2–(l) O2(g) + 4e– [1]
(ii) Reduction occurs at the cathode. The aluminium ion is reduced to aluminium. [1]
Al3+(l) + 3e– Al(l) [1]

(b) (i) The oxygen gas liberated during electrolysis reacts with the carbon anodes to form
carbon dioxide. [1] The anodes gradually erode and thus have to be replaced. [1]
(ii) Heat energy is required to melt the electrolyte. [1] If the melting point is lowered, less
heat energy will be needed to maintain the electrolyte in the molten state. [1]

(c) 2Al2O3(l) 4Al(l) + 3O2(g)


Mr of O2 = 2 × 16 = 32
6
Number of moles of O2 = 128 × 10 = 4 × 106 mol
32
From the equation,
Number of moles of Al = 4 × 4 × 106 = 16 × 106 mol [1]
3 3
Ar of Al = 27
Mass of Al = 16 × 106 × 27 = 144 × 106 g = 144 tonnes [1]
3
9. (a) (i) CH3(CH2)14COOH and CH3(CH2)29OH. [1] Add a few drops of concentrated sulfuric acid to
a mixture of the carboxylic acid and alcohol and warm. [1]
(ii) Student B is correct. [1] Beeswax is a mixture of esters and hydrocarbons and will
melt over a range of temperatures. [1] Esters and hydrocarbons are simple covalent
compounds and melt at low temperatures. [1]

(b) (i) H H H O H H H H O
H C C C C N H + NaOH H C C C C ONa + NH3 [1]
H H H H H H
Butanamide [1] Sodium butanoate [1]

© 2013 Marshall Cavendish International (Singapore) Private Limited Answers to Trial Examination 191

(S)CMWB_Answer.indd 191 8/27/12 8:21 PM


(ii) The organic compound is nylon. [1] It is used to make fishing nets/ clothing/ curtains/
parachutes/ sleeping bags. [Any one use: 1 mark]

EITHER

10. (a) (i) Volume of sodium hydroxide required for the titration = 15.00 cm3 [1]
Number of moles of NaOH = 0.1 × 15.00 = 0.0015 mol
1000
NaOH + HCl NaCl + H2O
From the equation,
Number of moles of HCl = 1 × 0.0015 = 0.0015 mol [1]
Concentration of HCl in mol/dm3 = number of moles
volume in dm3
= 0.0015 × 1000 = 0.06 mol/dm3 [1]
25.0
Mr of HCl = 1 + 35.5 = 36.5
Concentration of HCl in g/dm3 = 0.06 × 36.5 = 2.19 g/dm3 [1]
(ii) Volume of NaOH required to neutralise 0.0015 mol of HCl in Experiment 2
= 0.0015
0.05
= 0.03 dm3
= 30.00 cm3 [1]

(b) (i) Number of moles of NaOH = 0.1 × 25.0 = 0.0025 mol


1000
Number of moles of H3PO4 = 0.1 × 25.0 = 0.0025 mol
1000
Mole ratio of NaOH : H3PO4 = 0.0025 : 0.0025 [1] = 1 : 1 [1]
(ii) NaOH(aq) + H3PO4(aq) NaH2PO4(aq) + H2O(l)
(iii) The pH of NaH2PO4 is about 4.0 (mid of pH range 2.4–5.6). [1]
The pH of Na2HPO4 is about 9.2 (mid of pH range 7.6–10.8). [1]

OR

10. (a) Carbon is oxidised to form carbon dioxide: C(s) + O2(g) CO2(g) [1]
Carbon dioxide reacts with more carbon to form carbon monoxide: C(s) + CO2(g) 2CO(g) [1]
Tin dioxide is reduced by carbon monoxide to form tin: SnO2(s) + 2CO(g) Sn(l) + 2CO2(g) [1]

(b) (i) Lead(IV) oxide turns grey. [1] Aluminium oxide remains white. [1] Copper(II) oxide turns
reddish-brown. [1]
(ii) Mass of oxygen = 4.78 – 4.14 = 0.64 g
Number of moles of lead in lead(IV) oxide = 4.14 = 0.02 mol
24
Number of moles of oxygen in lead(IV) oxide = 0.64 = 0.04 mol
16
Mole ratio of Pb : O = 0.02 : 0.04 = 1 : 2 [1]
Empirical formula of the oxide used in the experiment is PbO2. [1]
(iii) Add aqueous sodium hydroxide to aluminium oxide and copper(II) oxide separately and
heat. Aluminium oxide is an amphoteric oxide, which is soluble in aqueous sodium
hydroxide. [1] Copper(II) oxide is a basic oxide and thus will not dissolve in aqueous
sodium hydroxide. [1]

192 Answers to Trial Examination © 2013 Marshall Cavendish International (Singapore) Private Limited

(S)CMWB_Answer.indd 192 8/27/12 8:21 PM


(S)CMWB_IFC&IBC(TE).indd 1
Acknowledgements

Front cover crystals © Yurok/Dreamstime.com


Private Limited

Reprinted 2017

All rights reserved.

Printed in Malaysia
First published 2007
Second edition 2013

ISBN 978-981-01-1702-3
addressed to the Publisher.
Website: www.mceducation.com
E-mail: tmesales@mceducation.com
© 2002 Times Media Private Limited
© 2001 Oxford University Press Pte Ltd

Customer Service Hotline: (65) 6213 9444

welcome any information which would enable us to contact the copyright holders/owners involved.
© 2014 Marshall Cavendish Education Pte Ltd

Published by Marshall Cavendish Education


Times Centre, 1 New Industrial Road, Singapore 536196

Level Theory Workbook by Oxford University Press Pte Ltd

of the copyright owner. Any requests for permission should be

The publisher would like to acknowledge the following who have kindly given permission for use of copyright material:
© 2003, 2007, 2013 Marshall Cavendish International (Singapore)

photocopying, recording or otherwise, without the prior permission


or transmitted, in any form or by any means, electronic, mechanical,

While every effort has been made to contact copyright holders of material reproduced, we have been unsuccessful in some
Original edition first published in 2001 as Chemistry Matters for GCE ‘O’

Marshall Cavendish is a registered trademark of Times Publishing Limited.


No part of this publication may be reproduced, stored in a retrieval system

instances. To these copyright holders, we offer our sincere apologies and hope they will take our liberty in good faith. We would
The Periodic Table of Elements
Period Group →

I II III IV V VI VII 0
1 4

1 H He
Hydrogen Helium
1 2
7 9 11 12 14 16 19 20

2 Li Be B C N O F Ne
Lithium Beryllium Boron Carbon Nitrogen Oxygen Fluorine Neon
3 4 5 6 7 8 9 10
23 24 27 28 31 32 35.5 40

3 Na Mg Al Si P S Cl Ar
Sodium Magnesium Aluminium Silicon Phosphorus Sulfur Chlorine Argon
11 12 13 14 15 16 17 18
39 40 45 48 51 52 55 56 59 59 64 65 70 73 75 79 80 84

4 K Ca Sc Ti V Cr Mn Fe Co Ni Cu Zn Ga Ge As Se Br Kr
Potassium Calcium Scandium Titanium Vanadium Chromium Manganese Iron Cobalt Nickel Copper Zinc Gallium Germanium Arsenic Selenium Bromine Krypton
19 20 21 22 23 24 25 26 27 28 29 30 31 32 33 34 35 36
85 88 89 91 93 96 99 101 103 106 108 112 115 119 122 128 127 131

5 Rb Sr Y Zr Nb Mo Tc Ru Rh Pd Ag Cd In Sn Sb Te I Xe
Rubidium Strontium Yttrium Zirconium Niobium Molybdenum Technetium Ruthenium Rhodium Palladium Silver Cadmium Indium Tin Antimony Tellurium Iodine Xenon
37 38 39 40 41 42 43 44 45 46 47 48 49 50 51 52 53 54
133 137 139 * 178 181 184 185 190 192 195 197 201 204 207 209

6 Cs Ba La Hf Ta W Re Os Ir Pt Au Hg Tl Pb Bi Po At Rn
Caesium Barium Lanthanum Hafnium Tantalum Tungsten Rhenium Osmium Iridium Platinum Gold Mercury Thallium Lead Bismuth Polonium Astatine Radon
55 56 57 72 73 74 75 76 77 78 79 80 81 82 83 84 85 86
226 227 †
7 Fr Ra Ac
Francium Radium Actinium
87 88 89
140 141 144 150 152 157 159 162 165 167 169 173 175
*58-71 Lanthanoid series
†90-103 Actinoid series Ce Pr Nd Pm Sm Eu Gd Tb Dy Ho Er Tm Yb Lu
Cerium Preseodymium Neodymium Promethium Samarium Europium Gadolinium Terbium Dysprosium Holmium Erbium Thulium Ytterbium Lutetium
58 59 60 61 62 63 64 65 66 67 68 69 70 71
a a = relative atomic mass 232 238

Key X X = atomic symbol Th Pa U Np Pu Am Cm Bk Cf Es Fm Md No Lr


Thorium Protactinium Uranium Neptunium Plutonium Americium Curium Berkelium Californium Einsteinium Fermium Mendelevium Nobelium Lawrencium
b = proton (atomic) number
b 90 91 92 93 94 95 96 97 98 99 100 101 102 103

The volume of one mole of any gas is 24 dm3 at room temperature and pressure.
3/10/16 9:23 AM
2nd Edition

Chemistry Matters for GCE 'O' Level Workbook (2nd Edition)


is the companion workbook to the revised best-selling Chemistry Matters for
GCE ‘O’ Level textbook. It is written in line with the 2013 GCE Ordinary Level
Chemistry syllabus from the Ministry of Education, Singapore. This workbook 2nd Edition

2nd Edition
is designed to promote learning with understanding. Its comprehensive range
of exercises builds foundational knowledge and develops critical higher-order
thinking skills. Coupled with tips on answering techniques and diagnostic self-
tests, this workbook will help students tackle questions with success and achieve
excellence in Chemistry.

Strengthens conceptual understanding


• Graded multiple-choice, structured and free-response questions reinforce learning

Workbook (Teacher’s Edition)


while building confidence and competency.

Develops independent learning and critical thinking skills


• Every worksheet ends with a diagnostic checklist that encourages students to
evaluate their learning and take steps to secure their understanding.
• A variety of context-based and data-based questions challenge students to train
up their higher-order thinking skills.

Sharpens interpretive and answering skills


• Tips to guide students’ understanding of the questions they might be asked help
them to hit the mark with their answers.
• Accompanying worked examples of sample questions ensure students articulate
their answers effectively.

Builds exam confidence

John Sadler • Emily Sadler


Tan Yin Toon • Chen Ling Kwong
• A Trial Examination gives further practice and boosts exam confidence by helping
students familiarise themselves with the exam format.

Other components
• Practical Book
• Textbook

Tan Yin Toon • Chen Ling Kwong


John Sadler • Emily Sadler
ISBN 978-981-01-1702-3

Workbook
Teacher's Edition
(S)CMWB(TE)_Cover.indd 1 9/11/12 11:22 AM

You might also like